Sins of omission

The other day, I got a request for an interview: a reporter was writing a story about Ken Miller. I was happy to do so — this was clearly going to be a friendly piece about Miller, and I thought it was good that he get some more press. I talked on the phone with this fellow for 20 minutes or so, and I told him what I thought: Miller is a smart guy, a great speaker, a hardworking asset to the people opposing creationism, and I also said that his efforts to squeeze religion into science were ill-founded and badly argued. I said, "It's an effort to reconcile a legitimate discipline with foolishness."

Guess what the only quote to make it into the article was?

Yeah, it turned out to be a crappy atheist-bashing article. It wasn't enough to talk about Miller's good work and the respect he gets from others — no, it had to be turned into a fight, with poor Miller unable to win because he's being "attacked by Darwin-hating fundies and leftie atheists alike," and the New Atheists are the primary villains of the piece. The more complex story I tried to tell got discarded, and only one short sentence made it to the final result. I must have been a major disappointment to the reporter, since I didn't give him much in the way of vicious attack-dog quotes.

He also got a little bit from Jerry Coyne. Again, it's clear but temperate stuff. The story really does not have anything to justify the claim that we're out to get Miller, or that the New Atheists are somehow in symbiosis with fundagelical loons.

"By discussing science and religion together and asserting that science more or less points you to evidence for God, he blurs the boundaries between science and faith," says Coyne, "boundaries which I think have to be absolutely maintained if we're going to have a rational country and we're going to judge things based on evidence rather than superstition."

I agree completely with that — Miller does blur the lines in very silly ways. The article even reiterates Miller's notorious explanation from his book, Finding Darwin's God, and obliviously confirms Coyne's point by approvingly citing the way Miller mingles nonsense with science.

But the cell biologist also makes explicitly scientific arguments: maintaining, for instance, that quantum indeterminacy -- the ultimately unpredictable outcome of physical events -- could allow God to intervene in subtle, undetectable ways.

This sort of sly intervention, he argues, is vital to the Creator's project: if God were to re-grow limbs for amputees, for instance -- if God were to perform the sort of miracles demanded by atheists as proof of his existence -- the consequences would be disastrous.

"Suppose that it was common knowledge that if you were a righteous person and of great faith and prayed deeply, all of a sudden, your limb would grow back," he says. "That would reduce God to a kind of supranatural force . . . and by pushing the button labeled 'prayer,' you could accomplish anything you wanted. What would that do to moral independence?"

That is not a scientific argument in any way—I guess the reporter was fooled by the flinging about of "quantum". All that is is tired old post hoc theological apologetics without a hint of evidence to back it up.

Nowhere anywhere in the article is any reasonable support for the notion of a god, nor especially of any peculiarly Catholic deity. Of course there isn't, because he doesn't have any.

What he does do, again, is try to throw atheists under the bus. It's more bullshit about how science has to compromise with the public's version of spiritual superstition, rather than remaining true to the evidence.

But Miller rejects any suggestion that the science in his work suffers when he brings in the spiritual. And he argues that the New Atheists, in their forceful rejection of God, are doing damage, in their own right, to a scientific brand already under assault.

Indeed, Miller argues that the creationists and New Atheists are in an odd sort of symbiosis -- reinforcing each others' extreme views of the incompatibility of science and religion.

Well, fuck that noise.

The New Atheists are as much a force in opposition to creationism as is Ken Miller; more so, I would argue, because we don't make fuzzy, muddled compromises with absurd medieval humbug. Even if he disagrees on that last point, his constant efforts to belittle the atheists on his side in this struggle, to repeatedly argue that they are a detriment to science education, is getting tiresome. Miller wants to turn the pro-evolution movement into a stalking horse for Catholicism, while his godless colleagues have repeatedly stated that we want no endorsement of religion or atheism in science education. The only one doing damage to the "brand of science" is the guy with pitiful idea that god is noodling about at the quantum level in ways that are completely undetectable — he wants to claim that he has an invisible dragon in his garage, and what's more, that that claim is scientific.

Remind me, next time I'm asked about Ken Miller, that I shouldn't bother to say anything appreciative. It will be ignored and won't be reciprocated. And I'm not going to endorse his crusade to taint science with supernaturalism.

More like this

No good deed will go unpunished.

By Nerd of Redhead, OM (not verified) on 04 Mar 2010 #permalink

I know. But at least now if any reporter wants to hear my views on Miller, I'll be sure to give them lots of juicy biting quotes.

With even the best Christians (Xians need not apply) it's vital that, until you have established a relationship of personal trust, you simply cannot trust them to do the right and simple human thing - not when they have the "Good Lord" with them to absolve or even bless them if they fuck you over.

Sad but true. There's some biologically-based reason for this, I'm guessing.

By Sioux Laris (not verified) on 04 Mar 2010 #permalink

So god acts on the quantum level? Is he the hidden variables Boym or does he actively change the wavefunction of the particles by constantly observing them?

I demand quantum miracles now. Perhaps I should begin praying to the statue of St. Albert the Great.

By Shawn Wilkinson (not verified) on 04 Mar 2010 #permalink

Why not try "I would love to share my honest views about Ken Miller, but interviewers always mislead me about exactly what they are writing about. So rather than have my opinion oversimplified solely to help sell a biased news article, let me just say that I believe Ken Miller to be a member of the New Kids on the Block, whoa-oh ah oh-oh." Then just hang up on them.

Remind me, next time I'm asked about Ken Miller, that I shouldn't bother to say anything appreciative.

But Miller is J*hn Kw*k's buddy. You're undermining all that Kw*k stands for by not appreciating Miller.

By 'Tis Himself, OM (not verified) on 04 Mar 2010 #permalink

This sort of sly intervention, he argues, is vital to the Creator's project: if God were to re-grow limbs for amputees, for instance -- if God were to perform the sort of miracles demanded by atheists as proof of his existence -- the consequences would be disastrous.

To continue on and find out why it would be disatrous:

"Suppose that it was common knowledge that if you were a righteous person and of great faith and prayed deeply, all of a sudden, your limb would grow back," he says. "That would reduce God to a kind of supranatural force . . . and by pushing the button labeled 'prayer,' you could accomplish anything you wanted. What would that do to moral independence?"

Oh, for fuck's sake! That's pushing idiocy to a new frontier. Religidiots already claim god can regrow limbs; now all of a sudden, it would be a bad thing if a god actually answered prayers and did the job. Uh huh. If god isn't a supranatural force, then what the hell is it? Apparently some supposedly benign force that can't actually do anything for real, surfing on quantum waves?

This is just trying to twist the old chestnut of free will - why did it happen? goddidit. Why didn't it happen? goddidit. Going by the bible, the last fucking thing god wants is moral independence. What a crock of shit.

Saying he was a member of NKotB would be far too flattering.

How about if I say he was a long-lost Jonas Brother?

I suggest using a stream of profanity when they ask next time, but be sure to add near the end that you're not talking about the subject of their article, but the reporter that called you last time and selectively quoted you in order to manufacture controversy. There is no reason anyone should put up with this.

By ckitching (not verified) on 04 Mar 2010 #permalink

As a former physics student, I fucking hate that kind of abuse of quantum mechanics.

(How much? I finally registered with MT to comment. :P)

By Zombified (not verified) on 04 Mar 2010 #permalink

#4:

I demand quantum miracles now. Perhaps I should begin praying to the statue of St. Albert the Great.

St. Albert won't grant you quantum miracles, Shawn. You're better off praying to St. Edwin, though his miracles can really kind of go either way.

I'd recommend St. Werner, but you can't know for sure how those would turn out.

By tfoss1983 (not verified) on 04 Mar 2010 #permalink

PZ Myers
"Saying he was a member of NKotB would be far too flattering."

But how often do you get the chance to sing a New Kids' tune into the phone as a polite way to say "Fuck off"?

Shame on you PZ....Censor yourself!

Courtesy the self-appointed communications experts of the internet

Miller wants to turn the pro-evolution movement into a stalking horse for Catholicism, while his godless colleagues have repeatedly stated that we want no endorsement of religion or atheism in science education.

Wait, what? When has Ken Miller used pro-evolution advocacy to specifically promote Catholicism, or introduce religion into the science curriculum?

On the one hand they argue that the reason why god is playing a game of hide for 2000 years and go seek is that by this means he can test our faith. As Ken Miller stated, if god was too overt, there would be no challenge to having faith, no way to prove that you were faithful. Or something like that.

So, faith is great and important, but then they use any chance they get to warp evidence to "prove" god's existence.

Faith-heads, just pick one:

(1) science can prove god's existence. we need to get busy and prove it.

(2) god can't be proven or disproved, so we'll stop meddling with scientific claims and just stick to the message that faith is irrational but we think it is good anyway.

By idiotiddidit#5116d (not verified) on 04 Mar 2010 #permalink

Hmm, before a couple of days ago I'd have simply agreed with your unhappiness about this on principle; now, having experienced what having your words taken out of context and used against you feels like first-hand (thank you for that, pissant Intersection twerps), I'm far more understanding.

By WowbaggerOM (not verified) on 04 Mar 2010 #permalink

Frustrating, but time and again someone, somewhere is wrong about science on the internet.

It leads me to a general question:
Why not leave science to the scientists?

Is it possible to educate the public with sound bites, blogs, or journalists summarizing the newest pr?

Maybe there is an argument for going back to the ivory tower. Any field of science has been around long enough now where the out-of-field scientist (let alone a non-scientist) will not and cannot appreciate what a field is doing (and why) without some serious study (and then good luck).

Perhaps refocus science education: teach science to the college students who take our science classes. Let them develop the expertise over time based on their genuine interest in the topic as revealed by them registering for our courses and then not dropping them.

If 60% of Ohioans (e.g.) believe the earth is less than 6000 years old, is a mission to educate people on current science the right strategy? Some large segment of humanity is simple not swayed by evidence and does not use reason to determine whether or not to believe in a claim.

Global warming is an example: No good comes from public debate on the topic because it seems ill-informed. Even just characterizing what the science knows is often inaccurate and spun to suit the author's purpose (far less of this goes on in the peer-reviewed literature, and if it does go on, it's only temporary, as evidence eventually wins).

I use global warming as my example-- I am wholly ignorant of the field and the science behind it. Yet, I defer completely to the mainstream expert consensus that man-made global warming is a problem.

Appeal to authority is not always a fallacy, and perhaps we should adopt this model rather than the one where anyone with an internet connection has an opinion no more or less forceful than the expert who'd devoted his/her life to studying the topic (and has likely contributed new scientific knowledge to the field).

The journalist Myers cites here does a dis-service to science. I say, why let this journalist have a voice?

Not talking censorship, but rather that we consider the source and seriously discount a non-expert view on any area of real science.

jmo

By https://me.yah… (not verified) on 04 Mar 2010 #permalink

A few fanboi words in defense of Ken:

When Ken takes on creationists in Public debates, he comes prepared with a knowledge of their arguments, as well as a knowledge of the science.

Most scientists only have the latter, and are reduced to wondering what the heck the creationist is on about.

Ken kicks arse and takes names in that forum, and at least with respect to a current US audience, he's all the more effective for being a christian.

And I guess you all know that, and I respect the intellectual honestly of not letting him get away with talking carp just because he's doing good as well.

But he's also filling a void there. Dawkins doesn't debate creationists, because he feels it lends them undue credibility. And for all Peasey's eloquence and intelligence, he will only be commanding in a debate once his voice breaks, and in terms of being up with the creationists arguments, Ken is the better of the two.

By Bored Wombat (not verified) on 04 Mar 2010 #permalink

That is not a scientific argument in any way—I guess the reporter was fooled by the flinging about of "quantum". All that is is tired old post hoc theological apologetics without a hint of evidence to back it up.

Have you ever noticed that the entire goal of the apologist isn't to convince someone of the correctness of their position. Rather, they only argue that their position can't be disproved. Thus, like a well-oiled gymnast, they twist and contort their assertions to fit in the snaking spaces between the jigsaw pieces of known evidence. It's always a god of the gaps argument. Sagan's invisible dragon is the perfect analogy.

But I can't understand why they go to all the trouble. Even if theism cannot be totally disproved, it doesn't matter because that's not a rational standard. When you believe in the existence of something for which no evidence exists (when you take something on "faith"), then you are irrational. Period. Occam's Razor cuts out needless assumptions. Hume made it clear why we must always accept the "lesser miracle." The party who bears the burden of proof is clear. Theists have failed to make their case for millennia. Now, in 2010, you don't get to mumble something about quantum magic tricks and claim that you've made a reasonable, sane, sensible, rational argument for believing in an invisible but benevolent super-ghost who needs to hide from us for our own good.

It's just insane to me that an intelligent man like Miller chooses to resort to such stupidity rather than give up his bad faith assumptions. Man, religion really does turn people's brains to mush.

@15:

The answer was much easier back in the Bronze Age. You don't believe in God? Well, do you believe in the edge of my sword?

Now that we live in an era of human intellect where it takes more than the threat of violence or the wielding of power to convince people of ones correctness, we have the whole slew of cognitive dissonance exhibited by people. God is in this gap. Oh shit, that gap doesn't exist? Well, he's actually here. Shit, stop filling gaps! It is obvious that God was never in a gap but can simply never be found.

What we're dealing is a religion whose origins were founded during an era of "it is obvious that gods exist" attempting to remain in an era when the obvious isn't so anymore.

By Shawn Wilkinson (not verified) on 04 Mar 2010 #permalink

Ken kicks arse and takes names in that forum, and at least with respect to a current US audience, he's all the more effective for being a christian.

that reminds me, I should wear my unicorn hat and "my little pony" shirt when I lecture to groups of young girls.

using a lie to talk about an unrelated message is...

dishonest.

this is what we accuse Ken of being.

intellectually dishonest.

I CAN direct you to the many arguments that have clearly shown this to be the case.

doubt you're really interested though, so long as this tactic appeases the faithheads, right?

idealism be damned.

Gotta love the journalistic quote-mining that they used on PZ. Why, it almost makes me think that "the creationists and [journalists] are in an odd sort of symbiosis -- reinforcing each others' [propensity to obscure facts via sneaky bullshit tactics]."

Bored Wombat @ 18:

Ken kicks arse and takes names in that forum, and at least with respect to a current US audience, he's all the more effective for being a christian.

Really? So it's okay for him to spew utter nonsense such as this?:

This sort of sly intervention, he argues, is vital to the Creator's project: if God were to re-grow limbs for amputees, for instance -- if God were to perform the sort of miracles demanded by atheists as proof of his existence -- the consequences would be disastrous. "Suppose that it was common knowledge that if you were a righteous person and of great faith and prayed deeply, all of a sudden, your limb would grow back," he says. "That would reduce God to a kind of supranatural force . . . and by pushing the button labeled 'prayer,' you could accomplish anything you wanted. What would that do to moral independence?"

Bullshit to your fanboi take. He's doing harm, just as all religious people do, by doing everything he can to keep people mentally tied to a construct which allows people to justify their pettiness, nastiness and evils towards others.

Even if he disagrees on that last point, his constant efforts to belittle the atheists on his side in this struggle, to repeatedly argue that they are a detriment to science education, is getting tiresome.

Obvious question: If this reporter misrepresented and oversimplified your view, isn't it possible he did the same to Miller's?

I mean, clearly Miller thinks that New Atheist (hatethetermbutwhatever) anti-accommodationism is ill-founded and badly argued and counterproductive, just like you and Coyne think about his efforts to marry evolution and faith. But for all we know, he spent most of his interview time praising you guys as smart and hardworking assets to the cause too, and that got discarded in favor of the Look At The Scientists Fight! theme.

I'd check in with him before concluding that he's escalating the rhetoric, at least.

By Anton Mates (not verified) on 04 Mar 2010 #permalink

From smiting the Egyptians to hiding in a misunderstanding of quantum mechanics. How art Thou fallen, O Light-bringer!

By Blake Stacey (not verified) on 04 Mar 2010 #permalink

"God's work is indistinguishable from randomness" never fails to have me laughing hysterically. Surely that's not the work of the biblical god then, so what god is it? Miller is obviously a heretic - anyone for a good ol' christian barbeque?

By MadScientist (not verified) on 04 Mar 2010 #permalink

If PZ was selectively quoted, is it not possible that Miller was, too? I understand that Miller has been in the biz for a long time and the quotes may well be a good representation of what he wanted to say, but if PZ is unhappy with the way he was quoted, Miller should be given some benefit of the doubt.

By surprises aplenty (not verified) on 04 Mar 2010 #permalink

Maybe it's neither here nor there, but what does a smart scientist like Ken Miller say about the problem of infinite regression? Who, or what, created this "thing" that acts through quantum indeterminacy in our physical world? Has he ever answered such a question?

I've asked why infinite regression isn't the "deal-breaker" for religion before, in my earliest days as an atheist, but lately it seems like nobody even uses this argument anymore. Michael Ruse dismissed it out of hand recently in a video on RD.net, without even offering a reason. He just scoffed at it, and that was it. No explanation whatsoever. (Incidentally, it was infinite regression, as well as Richard's view that "Complex things come from simpler things" that convinced me there was no God. It was also the clincher in a recent debate with my brother.)

But what does Miller say? Can't somebody just sit this intelligent man down, grab him (lightly but firmly) by his shoulders, give him one good, brisk (but loving) shake, and then look him straight in his Catholic peepers and say "What is your major malfunction here, dude?"

I can't be living on such a planet. It's impossible. This guy Miller, in particular, just astounds me. My mother is Catholic, too, but she never went to college and her ignorance is thereby excused, to a much larger degree.

Hello? Anybody home? Paging Doctor Miller... Dr. Miller, please report back to sanity...

By SaintStephen (not verified) on 04 Mar 2010 #permalink

God is the big casino in the sky? Tilting the odds in his favor...weird.

And God said let there be pachinko, and you will know my works. Bally 15:10.

Next time reporter calls:
Ken Miller...Miller?..Miller! Oh the beer guy, lite beer good; the others meh. Now let me tell you all about yeast...

By kantalope (not verified) on 04 Mar 2010 #permalink

Miller's approach to science seems very much parallel to the catlicking priests', - claiming moral superiority while sodomising choir boys.

What I can't get through my thick skull is that when you know better, why the hell not just be quiet about the superstition?
No need to defend nonsense that is used as a social glue. Just sleep through the sermon, as that avoids any grey matter to be tortured and killed.
But, guess Miller wants to be seen as a defender of the faith, for some obscure reason.
Honesty and decency would be more honourable, but when you are infected . . .

And I'm not going to endorse his crusade to taint science with supernaturalism.

Is Miller really trying to do that though? Rather I got the impression that he's trying to argue that the natural doesn't kill his supernatural. And really that's not such a bad thing.

Also, Only A Theory was a fantastic read. It was actually a lecture he gave for that which got me looking deeper into evolution and the ID controversy.

I always want to say something along the lines of "well, if your God doesn't do miracles, or interact with the world in any detectable way, what the Hell exactly does it do? What use is it as an explanation of anything?" Followed up with a hefty "What's more likely, that an invisible, undetectable, all-knowing, all-powerful god exists, or that you are simply wrong?" Someone like Ken should at least have the guts to stand up and say that yes, they may well be wrong, and in fact that balance of empirical evidence would suggest that they are.

By Your Mighty Overload (not verified) on 04 Mar 2010 #permalink

The balance of evidence being, I should add, exactly zero for their hypothesis.

By Your Mighty Overload (not verified) on 04 Mar 2010 #permalink

Himself,

The balance of evidence being, I should add, exactly zero for their hypothesis.

Ah, but that's using science as a way of knowing.

One has to use another way of knowing to properly evaluate the evidence.

By John Morales (not verified) on 04 Mar 2010 #permalink

D'oh. My previous was to Your Mighty Overload.

Sorry!

By John Morales (not verified) on 04 Mar 2010 #permalink

Hmmm... It really is a tough one. I don't like accommodationism because it's dishonest if you don't actually see room for accommodation. But realistically, what's the alternative? Since we can't actually disprove God, shouldn't we just let the religious babies have their bottles?

If you tell the average (religious) slob who is neither educated in science nor remotely interested in it that he can have either Yahweh/Allah or science but not both he'll pick Yahweh/Allah every time, won't he?

And then you get anti-evolution, anti-global warming, anti-vaccines, general hostility to science and scientists and decreased science funding.

Isn't there something to the old saying about catching more flies with honey than with vinegar? Oh, Jebus, I feel a bit ill just writing that, but it's probably true.

By ambulocetacean (not verified) on 04 Mar 2010 #permalink

About the possibility of Miller being misquoted.. am I wrong or are almost all of the quotes meant to reference his position not actually quotes but descriptions form the reporter? In other words, no sign that they're even words being taken out of context but perhaps outright fabrications meant to support the atheist scientist vs religious scientist war theme?

The bullshit about god not regrowing limbs because if he actually answered prayers (hello, I thought that was the WHOLE FUCKING POINT) then he'd somehow mess something up seems to be an actual quote, but.. well, that's kind of par for the course for the religionist. A person on Miller's level who doesn't deny the science itself has to leave a loophole like this, he has to redefine his religion as being only that which can never be detected or investigated in any way outside of imagining things.

I'm not familiar enough with Miller's writing to comment on particulars of it, leave it to the lots of others here that are.

If PZ was selectively quoted, is it not possible that Miller was, too?

PZ just wrote 9 paragraphs about how exactly he was selectively quoted.You would have to provide some evidence that and how Miller was, otherwise that comment doesn't really help with anything.

By Rorschach (not verified) on 04 Mar 2010 #permalink

A couple of comments:

Based on several years of observation, Ken Miller is hands down the best pro-evolution debater in North America. It really isn't even close.

Again, based on several years of observation, PZ is easily the most entertaining and trenchant critic of creationists in North America.

Both are invaluable resources for me personally, and I value them for different reasons. It's a shame that we keep having this tactical debate with respect to how atheists should comport themselves.

I personally think that Ken Miller has no business complaining about the godless's desire not to be marginalized, ignored or pilloried unfairly. I'm a theist, but it is no sweat off my back if some atheists happen to be vocal in their non-belief. I don't see how it helps the scientific enterprise one bit to discourage constitutionally-protected speech.

On the flip side, I also think that many of you here are clueless when you attempt to argue that Miller is intellectually dishonest simply because he doesn't share some of your premises. The Ken Miller I know is not your enemy, because he would never attempt to put his version of 'Darwin's God' into the public schools.

By the way, it is likely every year more people read what Miller and his colleague Joe Levine have written about evolution than any other author, living or dead---and there is nothing in their texts at all about religion one way or the other.

By Scott Hatfield, OM (not verified) on 04 Mar 2010 #permalink

Add me to the "benefit of the doubt" camp; it's unfair to complain about the sin of omission and then of being thrown under a bus in the same entry.

Otherwise, I respect Miller's right to push his God Of The Gaps as far back as he has: I still think he's wrong. Same as everyone else here I guess :)

Scott @40,

By the way, it is likely every year more people read what Miller and his colleague Joe Levine have written about evolution than any other author, living or dead---and there is nothing in their texts at all about religion one way or the other.

Nothing at all?

According to the linked article, "Miller's effort to bridge science and the Abrahamic faiths of Christianity, Judaism, and Islam began, in earnest, with the publication of Finding Darwin's God: A Scientist's Search for Common Ground Between God and Evolution (1999)."

By John Morales (not verified) on 04 Mar 2010 #permalink

Anytime "quantumism" enters into the discussion you know its pure hokum. Reliance on "quantumism" has been the refuge for anti-science spiritualist charlatans like Deepak Chopra and Robert Lanza for decades. Now it seems theists like Miller have taken up the cudgel. You would think that these pseudo-physicists would be embarrassed by this ridiculous charade, but I guess it gives them some credibility to those who know nothing about either quantum mechanics or biology.

Scott, the thing is Miller does share our premises, except when he's talking about religion.

@17: "The journalist Myers cites here does a dis-service to science."

Yeah, not to mention a disservice to journalism.

By Pete Moulton (not verified) on 04 Mar 2010 #permalink

@ Rorschah 39

"If PZ was selectively quoted, is it not possible that Miller was, too?
PZ just wrote 9 paragraphs about how exactly he was selectively quoted.You would have to provide some evidence that and how Miller was, otherwise that comment doesn't really help with anything."

Nonesense, nothing needs verifying. PZ's write up comprehensively falsifies the proposition "this article is trustworthy and we can reach conclusions based on it".

"Remind me, next time I'm asked about Ken Miller, that I shouldn't bother to say anything appreciative. It will be ignored and won't be reciprocated."

PZ condemns Miller on insufficient evidence: the reporting of Miller's words by someone we know to be an unreliable witness.

By https://www.go… (not verified) on 04 Mar 2010 #permalink

This sort of sly intervention, he argues, is vital to the Creator's project: if God were to re-grow limbs for amputees, for instance -- if God were to perform the sort of miracles demanded by atheists as proof of his existence -- the consequences would be disastrous.

"Suppose that it was common knowledge that if you were a righteous person and of great faith and prayed deeply, all of a sudden, your limb would grow back," he says. "That would reduce God to a kind of supranatural force . . . and by pushing the button labeled 'prayer,' you could accomplish anything you wanted. What would that do to moral independence?"

Wait... did he just offer up the Babel Fish argument in all seriousness?

Can we ask these guys why it's not OK for their God to make people's limbs grow back now, but it was OK for their God to kill millions of people back then?

Why is miraculous healing a threat to moral independence but the death of all the first born of Egypt is just peachy?

If their God cares about moral independence, where does "hardening Pharaoh's heart" fit in?

I'm also visualising a doctor saying "Well, I could treat you, but then you'd just take it for granted that you could get treatment and you'd lose your moral independence."

Miller's message to the sick: God loves you, fuck off and die.

Damn, now the Intersection will whine about me too :)

By Stephen Wells (not verified) on 04 Mar 2010 #permalink

Damn.
This is why I hope to never be anything more than a scientific peon (assuming I manage to actually get the degree and get elevated to that level.)
I don't know how you celeb scientists stay sane with all this bullshit "journalism" buzzing around, waiting to pounce on anything you say.
(Since when was journalism supposed to be sensationalised drama baiting instead of... well, news?)

This sort of sly intervention, he argues, is vital to the Creator's project: if God were to re-grow limbs for amputees, for instance -- if God were to perform the sort of miracles demanded by atheists as proof of his existence -- the consequences would be disastrous. "Suppose that it was common knowledge that if you were a righteous person and of great faith and prayed deeply, all of a sudden, your limb would grow back," he says. "That would reduce God to a kind of supranatural force . . . and by pushing the button labeled 'prayer,' you could accomplish anything you wanted. What would that do to moral independence?"

Either there's an interventionist god who selectively answers prayers from non-amputees or there's a noninterventionist god who ignores prayers completely. Since Miller is a Catholic and Catholics believe in the "power of prayer" then he's supposedly pushing for the first option. So I fail to understand what Miller's argument is. Does his god hate amputees so greatly that it ignores their prayers but listens and responds positively to other prayers?

By 'Tis Himself, OM (not verified) on 04 Mar 2010 #permalink

OK, so it sounds like Ken's suggesting I look for evidence for God in the semiconductors I study--a God of the bandgaps. Oh, look, they vary with temperature and doping, but nope, no random electrons that can't be explained by thermal agitation.

I'm sorry, but ol' Ken need to crack a text on quantum mechanics. Quantum indeterminacy is an awfully small drawer to put a deity in.

And 'Tis Himself is right. If the deity intervenes--ever--then the question is why he doesn't intervene to stop the occasional genocide, move the epicenter of an earthquake. An interventionist deity must by definition be one heartless SOB.

This isn't just crappy science. It's crappy theology and fuzzy thinking.

By a_ray_in_dilbe… (not verified) on 05 Mar 2010 #permalink

Ken Miller isn't fighting Creationism. He can't; he's an Old Earth Creationist. He's merely fighting Young Earth Creationism. In order to fight Creationism, Miller must be tossed under the bus.

P.S.
To anyone who would argue with my assessment of Miller: I welcome you to distinguish his position from that of Old Earth Creationism.

Re: growing back limbs.

I heard a theist (I think it was Dinesh D'Souza) answer a question about why god doesn't grow back limbs in this way:

It's known that people have a default happiness setting. In other words, a year or two after winning the lottery or losing a loved one, people tend to revert back to being just as happy or miserable as they were before those major events.

By implication, people are just as happy without their limbs.

I believe my mouth actually dropped open.

By Kathy Orlinsky (not verified) on 05 Mar 2010 #permalink

I'm not sure who I'm addressing this to... too lazy to read up and figure it out...

To the people asking why there is an issue with Miller at all:

I don't think the problem is that he believes in god, sure, it seems a bit silly that someone who works day to day in a rational and evidence based world would find themselves able to hold onto an idea with no evidence, but whatever floats your goat.
The problem comes when you start defining quantum level events as god, or the actions of god. We could easily claim that miotic division is the action of god or the draining of my sink when the plug is removed, but there is nothing in that swirling, glugging torrent that demands or even suggests an overlord.

Belief in god: fine, if you like that sort of thing.
Scientific enquiry: definitely fine.
Scientific enquiry with a liberal sprinkling of godflakes: NAUGHTY STEP!

The more complex story I tried to tell got discarded, and only one short sentence made it to the final result. I must have been a major disappointment to the reporter, since I didn't give him much in the way of vicious attack-dog quotes.

Point number 1: Journalists already know the story they are going to write when they contact you. Always ask what the general outline/framework is. Figure out from that the best phrases you can come up with for making the points you want to make.
Point number 2: Journalists can't handle complex narratives. They are particularly terrible at context. Avoid both of these. Come into your interviews with, at most, three points you want to make. Remember, there are always layers of editors to also fuck it up, and to ask for a story that thrives more on conflict so it will sell more copies.
Point 3: Find a way to answer every question you are asked with the points you are trying to make. You are merely filler and color for the story they are going to tell. Find a nice way to fit your point into the story, and keep going back to it.

By MAJeff, OM (not verified) on 05 Mar 2010 #permalink

Kathy Orlinsky #53

That's the sort of sloppy thinking we've come to expect from D'Souza.

* Do people actually do have default happiness settings? I'd like to see a citation for that.

* What's to say that a major trauma, like losing a limb, doesn't reset the default?

* Perhaps the long-term default setting remains more or less constant, but the short-term setting can vary widely. Why doesn't D'Souza's god worry about the short-term effects of losing a limb?

By 'Tis Himself, OM (not verified) on 05 Mar 2010 #permalink

In my work life, I'm a sociologist who studies the intersection of social movements and news media. The points from my previous post come out of that sort of work.

If you want to change journalism, it isn't going to come anytime soon. The narrative has been established: "mean new atheists" vs. "very nice religious scientists." It fits so well into our idea of ourselves as a "moderate" and "pragmatic" nation...after all, folks like Miller and whatshisname at NIH can be presented as occupying the happy middle, which is--by definition--the right position to occupy (the entire field of journalism is basically Broderism writ large...objectivity norms, both sides, focus on the middle, blah blah blah).

Changing the narrative is going to be a long slough. It can be done. One of my colleagues has done some work on how the Rhode Island Coalition Against Domestic Violence was able, in working with journalists as well as civic leaders, to shift the discourse on domestic violence and murder away from the constant focus on neighbors who are shocked because "he seemed like such a nice young man" and toward a narrative that tends to focus more on institutional responses and the contextual problem of domestic violence. It took about 6-7 years for the shift to take place.

J-school trains for incompetence, and what PZ is experiencing is exactly what they are trained to do. The journo had a story. PZ gave good quote. The editors were likely happy that it was a story that had nice conflict, presented both sides, and used the right words. The publisher sees controversy, and thus more eyes and more money. It's a win for everyone in the news organization.

By MAJeff, OM (not verified) on 05 Mar 2010 #permalink

SaintStephen #20 infinite regression

I think this argument falls flat when trying to reason with most believers (even casual ones). I'll say why BUT do not think I in any way am making scientific statements here - I am only talking about how I think ordinary people approach the question.

1. most believers are imprinted with "god always was and always will be" - it is something they have been conditioned to accept a priori - and they do. It comes with the whole magic package - and probably is the LEAST problematic problem for them.

2. questioning things like "why is there evil?" or "why does god hate amputees?", "why are there so many varied beliefs?", "how do you know that?", "kid gets sick - you rush her to church or to hospital?", or even models of thought that clearly show how if faith beliefs are allowed they would interfere with getting to secular truths they ALREADY accept - these get them thinking more about how rational or irrational they are in their faith.

3. cosmic issues are way above most of us - way beyond our education and experiences - the science surrounding these things sounds like magic - no less magical than "god always was and always will be". It is a non-starter for most because the "then how did that get there?" regresses back to magic (again the science sounds like magic to them - and a matter of faith for the scientists); they feel justified in just accepting "god always was and always will be".

Bottom line: it is too complicated and too abstract for most people.. they see the scientific answers no more valid than their "god always was and always will be"

By ConcernedJoe (not verified) on 05 Mar 2010 #permalink

sorry that was SaintStephen #29 infinite regression

By ConcernedJoe (not verified) on 05 Mar 2010 #permalink

Reading Deltoid lately has led me to agree with the commenters there who suggest that people should always ask for the reporter's permission to record the interview. If it's given, record it. If not, don't do the interview.

It might not be a bad idea for someone to set up a site where scientists can report on their experiences with individual journalists and their coverage of science and scientists, linking to their articles. For each journalist, praise and criticism could be included, and perhaps there could be some kind of rating system.... That way, when someone's contacted, she can get information in one place and not have to search through google. Might have helped to expose Leake a lot sooner.

Beyond the science/religion issue, I didn't care for Miller's jingoistic presentation of science as this lone maverick enterprise rather than a collaborative and accumulative activity; and I really didn't like his "welfare queens" analogy. Infuriating and wrong.

"How about if I say he was a long-lost Jonas Brother?"--PZ What about the Jonas Father? I'm sure Disney would appreciate that new spin.

By myownschadenfreude (not verified) on 05 Mar 2010 #permalink

#56: D'Souza's God apparently stopped caring around the time of the Neolithic agricultural revolution. At least that's the impression I get from listening to him speak.

Ken Miller at least makes his attempts in a rational (if unbelievable) manner.
I have hope for him, that he may one day see that indeed, everything he knows and has known can have existed independent of any deity.

For men such as Ken Ham,on the other hand, I have no such hope.

PZ: Journalist missquote solution:

When interviewing Managing Partners at law firms they frequently agree to speak with me on the condition that I email any them any quotation I intend to use for their approval.

You can't control context but you can control how you are quoted.

Q.E.D

my comment to the article:

Prof. Miller is an excellent scientist when he sticks to his chosen field of expertise. Unfortunately, he also engages in ridiculous apologetics theology. His quantum god claim is merely a "god of the gaps argument", the same kind of argument used by young earth creationists who incorrectly claim their are no "transitional fossils". These arguments are not scientific theories, they are piss poor apologetics.

If Prof. Miller is a good catholic he believes in transubstantiation, the magical turning of wine and wafers into the blood and body of christ. That's fine, there just isn't any scientific evidence for that belief - quantum or otherwise. The "New Atheist" crowd (of which I am a member) merely point out that Prof. Miller is doing theology not science when he talks about god. We tend to be annoyed when people like Prof. Miller use their position as eminent scientists to make an argument from authority and lend the mantle of scientific rigour to nonsense theological claims.

In theory, the shit-for-brains reporter might have done the same thing to Miller that he did to you.

The only way someone could say that about quantum mechanics is to not understand quantum mechanics. If god changes things at the quantum level, in such small ways that it is undetectable from the random distribution, then it is no change at all.

It reminds me of the astrologer who, when pushed, claimed that the planetary effects are so subtle that even advanced statistics can't tell them from random chance. Yet, the Babylonians six thousand years ago (at the beginning of the universe?) could record the rules by which the planets affect our lives?

If the outcome LOOKS like a random distribution (even under the most sophisticated statistical tests) then it IS a random distribution.

WCorvi,
I agree, but a small nit. Nobody really knows how to define "random". Kolmogorov spent his entire life trying and the closest he came was an algorithmic definition--e.g. if the number of bits needed to define a series is equal to the number of bits in the series, then the series is random. Unfortunately, this only works for finite series, and there's some question about whether a finite series is ever truly random.

This (and various other paradoxes) is what caused deFinetti to give up entirely on any sort of measure theoretic definition of probability. The result was modern Bayesian probability.

By a_ray_in_dilbe… (not verified) on 05 Mar 2010 #permalink

Yes, I know the reporter probably left out any moderating commentary Miller might have made. But the stuff he did leave in — that atheists are doing harm, that they're just like the fundies they oppose, and all that madness about quantum indeterminacy — are typical Miller.

That's the story reporters apparently want whenever they talk about the guy. I'm just saying that in the future I'll feed their appetite by clearly saying that that is all crazy talk, and that what Miller does is lard good science with bullshit.

Oh, and MAJeff, I did as you say. I got the reporter's goal, which he said was to do a laudatory piece on a well-known regional figure, and almost all of what I said was about what a good guy Miller was, and how he'd been such a hero to science in the Dover trial, in getting science to the public, etc. Not much I can do when the reporter fails to tell me that the real focus of the story was on the atheist/christian battle, and how dear abused Ken was the reasonable moderate caught in the middle.

This sort of sly intervention, he argues, is vital to the Creator's project: if God were to re-grow limbs for amputees, for instance -- if God were to perform the sort of miracles demanded by atheists as proof of his existence -- the consequences would be disastrous.

"Suppose that it was common knowledge that if you were a righteous person and of great faith and prayed deeply, all of a sudden, your limb would grow back," he says. "That would reduce God to a kind of supranatural force . . . and by pushing the button labeled 'prayer,' you could accomplish anything you wanted. What would that do to moral independence?"

This is just pure bullshit, and it's a little surprising to me that the author postulates this example as if it's not already happening. I wrote on my blog several months ago about Lourdes and the reports of miracle healings taking place there. And despite the obvious paradox of having to accept on a very conscious level that god appears to be capricious and completely random in whom he decides is worthy of healing, it has not stemmed or even remotely slowed the stream of the steady faithful that flock there every day. The religious don't put anywhere near the sort of thought into reconciling the randomness of so-called "miracles" and faith criteria. It doesn't even occur to them to question it, as they've been taught their whole live not to... religion has a built in defense against such though processes.

Such a statement just shows that this author understands religion as poorly as he understands science and atheism.

By Celtic_Evolution (not verified) on 05 Mar 2010 #permalink

typos in #72 included for you, completely free of charge. Courtesy, as always, of the chimp...

By Celtic_Evolution (not verified) on 05 Mar 2010 #permalink

Not much I can do when the reporter fails to tell me that the real focus of the story was on the atheist/christian battle, and how dear abused Ken was the reasonable moderate caught in the middle.

No, there isn't. I think, at this point, it should be sort of a working assumption that this is the basic story the media want to tell.

Working with news media suck. No matter what one says, the second it leaves your mouth you have no control over it. They can selectively quote to their heart's content. The problem is when dumb-ass narratives become institutionalized as "the way things are." Right now, the institutional narrative is "those poor put-upon moderates." Just as "Al Gore" became synonymous with "liar" in the 2000 campaign, "new atheist" has become synonymous with "big old meanie persecuting moderates." The stock characters are already in position. All that's needed is a quote; there's likely already a space set aside for it in the partially written text.

What it's going to take at this point, is long-term sustained activity to change the story itself. That may actually take new journalists (and editors, and publishers....)

By MAJeff, OM (not verified) on 05 Mar 2010 #permalink

What it's going to take at this point, is long-term sustained activity to change the story itself. That may actually take new journalists (and editors, and publishers....)

That may be a near impossible task... with the glut of media outlets that exist today, between print, online, periodical, and TV / News, too many "journalists" (and I put that in scare quotes intentionally. as the definition of 'journalist' has become so blurred these days) feel like in order to make a name for themselves, they must put forth attention-grabbing, hot button stories filled with conflict and tension. For too many (but not all, certainly), integrity takes a back seat to notoriety. Why do you think Fox news is so goddam successful? That may as well be their friggin motto.

Conflict sells to the lowest common denominator, short attention span average consumer of today. Just turn on the fucking TV for fuck's sake... anyone else sick of voyeuristic, contrived conflict put forth by about every major network on about every given day?

By Celtic_Evolution (not verified) on 05 Mar 2010 #permalink

What is the point of an undetectable god that works at the quantum level? Also, why couldn't the world work if the all-powerful diety in charge of it performed miracles for those he favored? Firstly, the entire Bible assumes that is how the world work as does the Catholic Church. Sure, over time they've changed what type of miracles are to be expected, but in the Gospels Jesus healed the blind and deaf and turned water into wine. Does Miller reject those miracles? How can one be Catholic while rejecting miracles? The fundamental Catholic doctrine is that their god reanimated his son's corpse to overcome death for all mankind!!

There is no way if Ken Miller actually laid out his religious beliefs that any objective observer would label him Catholic. Unfortunately he is afraid to shed the label himself.

The whole quantum divine will is a wonderful example of what happens when a strong rational mind decides to rationalize rather than think rationally.

You don't get that kind of sophisticate self-delusion from folks who collect angel figurines.

By a_ray_in_dilbe… (not verified) on 05 Mar 2010 #permalink

When you believe in the existence of something for which no evidence exists (when you take something on "faith"), then you are irrational.

string theory comes to mind when I read that statement.
Question whats a "new athiest" is there an old testament atheist and you guys are the new testament version?

By broboxley (not verified) on 05 Mar 2010 #permalink

Re #57:

As someone who worked as a reporter a while, and spent a little while (just a year--did fine, didn't much like it, switched to Biology) in a J-school, yeah, that's all about right.

It's the nature of a certain standard approach to news: find the conflict, that's the nut of the thing, the notion is, it keeps people reading. Colourful quotes add a little zing and personality to that, and in 300 words, you've got yourself something that might get page one, tells people why this is interesting, why they should care.

By the way, you can actually do good stuff in that framework. But it depends on the nature of the story, obviously. And the real problem is: it's really not very friendly to expanding people's grasp of more complex issues and phenomena, obviously. They'll only get more complex stuff and shift their sense of what the conflict actually is in a sort of accretive fashion, at best, from many stories, if all they're absorbing is the daily news.

So, given this, re changing the narrative, my take on it is somethin' like: if they want conflict, fine, let 'em have it, but open their eyes to the larger one, and get 'em to see there really is merit to the position of those who see this as a matter of simple intellectual honesty versus its simpering absence. And the priorities of science and of responsible public intellectuals have to be: get it right, first, above all else. Say what you know, how well you know it, and call people out when you catch them doing otherwise.

That, in my ever so humble opinion, is the problem with Miller. Fuck all this talk of playing smart politics to get people on side in the fucking 'culture wars'--the whole trouble with it is it simply runs too counter to professional ethics and, again, the simple requirement for intellectual honesty. And get them to understand: a responsible scientist just doesn't say the stupid airy fairy shit Miller does about quantum bibble babble, and likewise, a responsible scientist just doesn't let that shit pass without appropriately critical comment.

Get them to see that as the conflict. Get them to understand that's where you're coming from, and that Miller, for all he's trying to sound nice and reasonable, is being nothing of the fucking sort. He's simply being a dishonest panderer. And a responsible intellectual--almost regardless of their discipline--shouldn't be playing that game just to make nice. You've got a responsibility to call it like it is. He's betraying that. You've got to point it out. You can't be apologizing excessively if folk find it a bit painful.

So, saying it again: I figure you want to get it through their heads: look, that's the real conflict, here. People shy from writing about that one because thinking about it actually bites a bit more than this stupid sideshow about who's more 'nice'. But the reporter, also a professional, is likewise being pretty much a wimp if s/he lacks the fucking cojones to open that one up properly, and wants to stick to that safe, popular narrative.

I'm pretty sure they'll print that. So I don't think PZ's instinct here is wrong at all. Give 'em a quote that slaps 'em in the face with it, and they'll print copy that slaps people in the face with it.

Regardless of how you do it, anyway, get the reporter to respect that attitude of the responsible thinker (as they really fucking should), get them to grasp that that's the conflict, and once they can work around that in their rather limited daily news format, you're many, many steps ahead.

Colgate Twins post on this in 3. . .2 . . . 1. . .

By Jerry Coyne (not verified) on 05 Mar 2010 #permalink

I don't want to point out the obvious here but wouldn't it be a good idea to simply record all your interviews (you do have an ipod touch!) and if you are taken out of context by the reporter just post an mp3 of the full, in context, interview.

John Morales (#42):

I was referring to Miller and Levine's high school textbook. "Finding Darwin's God" is Miller's personal take on the evo/creo wars. If you read it, you probably wouldn't have a problem with 98 percent. The 2 percent, with the quantum dodge, isn't science but a careful reading of the text shows Miller isn't presenting it as such.

Nomad (#44): I agree. But so what? I share certain premises with Donald Trump, but I don't see myself adopting his management style any time in the future. My point is that the premise by which people accuse Miller of hypocrisy or intellectual dishonesty doesn't come into play.

Q.E.D. (#67): I don't think Miller uses his position as a scientist to make an argument from authority with respect to Catholicism or any other faith. That seems as nonsensical to me as a fundy claiming that PZ uses his position as a scientist to privilege his atheist views. I don't believe either one is the case.

You'd have to be pretty stupid to claim that simply because so-and-so is a scientist, any claim they make is a scientific claim. I realize pretty stupid people exist, but I much prefer a world in which scientists with divergent personal views (like Miller and Myers) can have a go at each other without either claiming their viewpoint is privileged by their standing as scientists.

athywren (#54): Your position could be seen as suggesting that a scientist who is privately a theist has no business publicly discussing how they ATTEMPT to reconcile apparent conflicts between science and religion. I don't really think that's what you meant to suggest. Miller doesn't really put quantum indeterminacy forward as a scientific claim, as it is obvious from a careful reading of his text. It does the enterprise of science no harm if individuals defend their personal beliefs with arguments that can not be adjudicated by science, as long as they don't pretend the claims they are making are scientific claims. Miller doesn't claim that his quantum indeterminacy argument is scientific, so I don't see the point.

sudonim2(#52): I will rise to your challenge.

In the strictest of possible senses, Miller certainly is a creationist in that he believes in a creator. In the same sense, Dobzhansky, speaking more broadly, identified himself as 'both an evolutionist and a creationist.' Fair enough.

But this is NOT the sense that most of us refer to creationists, who propose not to accept the mechanism of evolution, but instead insist that we substitute a religious doctrine of 'special creation' as an explanation for life's diversity.

Properly speaking, old earth creationists (OEC) are still concerned with harmonizing the findings of science with something like a literal understanding of the Bible. They still want to substitute a creative act of a supernatural being for a well-established explanation based on natural causes. They do not believe that the latter is compatible with the existence of the former. These folk, like Hugh Ross of Reasons To Believe, will interpret the word used in Genesis ('yom') which is often translated as 'day' to mean much longer units of time, a usage which is permissible in the original Hebrew. They are still hung up on the idea that, in some way, the Biblical account is without error when properly understood.

Ken Miller doesn't believe in any of that. He does not treat Genesis as a science text, he doesn't agonize on the sequence of events described in Genesis not lining up with the fossil record, and he certainly accepts evolution as the mechanism for the generation of diversity, and does not at any time require divine intervention.

Bottom line: Miller's views are distinguishable from those typically labeled 'creationists' on the basis of how he does science, not on the basis of how he interprets scripture. That is poles apart from the way virtually all self-described 'creationsts' behave.

By Scott Hatfield, OM (not verified) on 05 Mar 2010 #permalink

Suppose that it was common knowledge that if you were a righteous person and of great faith and prayed deeply, all of a sudden, your limb would grow back," he says. "That would reduce God to a kind of supranatural force . . . and by pushing the button labeled 'prayer,' you could accomplish anything you wanted. What would that do to moral independence?

The problem is, according to the bible, "pushing the button called prayer" is supposed to accomplish anything you want. Supporting scripture:

This is the confidence we have in approaching God: that if we ask anything according to his will, he hears us. And if we know that he hears us - whatever we ask - we know that we have what we asked of him" (1 John 5:14-15)
"And I will do whatever you ask in my name, so that the Son may bring glory to the Father. You may ask me for anything in my name, and I will do it" (John 14:13-14).

These are promises made by Christ himself. Overwhelming, indisputable evidence tells us that these promises are false, and therefore Xians ignore them, or make up lame excuses about "moral independence."

If you ignore the last two chapters, Finding Darwin's God is a word-for-word better-argued, more accessible read against creationism* than The Greatest Show on Earth, in my opinion. So it's always heartbreaking to know that those two last chapters exist - it's like watching the Matrix sequels where they fuck everything up, when they should have left well alone.

*However, TGSOE is a better book for evolution.

it's like watching the Matrix sequels where they fuck everything up, when they should have left well alone.

I don't know what you're talking about. Those never happened. You hear me? Never. Happened.

By Celtic_Evolution (not verified) on 05 Mar 2010 #permalink

To Scott Hatfield, OM, re: #82

I would be interested to know how one can find 98% of a book acceptable and scientific, and 2% unacceptable and non-scientific, especially when the god argument in question is loaded with scientific terms. It would seem an intentional exercise on Miller's part to make the god argument look science-y.

Why should we have to carefully parse Miller's text to realize that this one tidbit in an otherwise reasonable book is unreasonable? And what else can we conclude, but that he was trying to make it look reasonable?

Anyway, as for PZ's dilemma, I see no one has mentioned the option of not saying anything unfriendly, especially for a supposedly 'laudable' and positive article -- giving the axe-grinders nothing to grind. Maybe a reporter would try to push you in that direction, but couldn't you just innocently say 'but I thought this was supposed to be a nice article, why mention that?' *bats eyelashes*

But I can understand why holding one's tongue might seem disingenuous. And then you'd have to make sure that everyone did that; sounds impossible. Reporters could just go trolling for the negative.

I do like the option of telling reporters you think they're going to twist your words, and done. I guess it depends on how interested PZ is in getting his name in the papers.

#86

I was getting ready to write the same thing in response to Scott Hatfield, more or less...

The last two chapters serve as summation and conclusion and are dreadful... and as a casual reader, how do I reconcile the first 90% of the book with a conclusion that is seemingly contradictory? How do I accept his approach to the science he uses in the first part when it seems clear the he is willing to accept the supernatural without much reservation?

By Celtic_Evolution (not verified) on 05 Mar 2010 #permalink

@78

A "new atheist" is one the Christians aren't allowed to burn at the stake.

Hmmm, I think the better response would be that when a reporter asks you about Ken Miller -- because you KNOW they are going to spin it this way -- just emphasize what great work he does, what an asset he is, and that "Though we might disagree on certain issues, this does not diminish his stature as a scientist and a campaigner for better science education." And then refuse to elaborate on the issue on which you disagree.

Don't give them shit, and then the only article htey'll have to write is, "Well, we think these New Atheists are really mean and nasty, but we don't have any quotes at all to demonstrate that. but they are atheists! They must be meanies!"

By James Sweet (not verified) on 05 Mar 2010 #permalink

No.

That sounds like intellectual cowardice. So if you're a Catholic with stupid ideas, you don't get even mild criticism because that might make the critic look like a meanie, and you're still free to say some really nasty things about atheists?

Nah, I'm not going to play that game.

Scott Hatfield @ 82

"You'd have to be pretty stupid to claim that simply because so-and-so is a scientist, any claim they make is a scientific claim. . ."

You don't think that a biologist mixing quantum physics nonsense (not his field of expertise) and theology (not his field of expertise FWIW)isn't purposefully dressing up crappy apologetics as science?

seriously?

maybe you should read his *Templeton* essay where he purposefully conflates science with religion and throws in quantum mechanics and Einstein for good measure.

http://www.templeton.org/belief/essays/miller.pdf

No.

That sounds like intellectual cowardice. So if you're a Catholic with stupid ideas, you don't get even mild criticism because that might make the critic look like a meanie, and you're still free to say some really nasty things about atheists?

Nah, I'm not going to play that game.

Yes, this.

I'd rather be taken out of context and thought of as a nasty old atheist asshole and know that I was honest and forthright when giving my opinion, than acquiesce to the Milquetoast Mafia with vapid, empty platitudes that don't reflect my actual thoughts.

I'll refuse the interview altogether before taking that tack...

By Celtic_Evolution (not verified) on 05 Mar 2010 #permalink

St. Albert won't grant you quantum miracles, Shawn. You're better off praying to St. Edwin, though his miracles can really kind of go either way.

I'd recommend St. Werner, but you can't know for sure how those would turn out.

LOL!

Except it's Erwin, not Edwin.

Oh, and MAJeff, I did as you say. I got the reporter's goal, which he said was to do a laudatory piece on a well-known regional figure, and almost all of what I said was about what a good guy Miller was, and how he'd been such a hero to science in the Dover trial, in getting science to the public, etc. Not much I can do when the reporter fails to tell me that the real focus of the story was on the atheist/christian battle, and how dear abused Ken was the reasonable moderate caught in the middle.

So, the reporter lied. Right?

By David Marjanović (not verified) on 05 Mar 2010 #permalink

Insignificant creature steps out on a limb

A modern scientist that holds a view that there is an active (at any level) god has to reek of intellectual dishonesty and inane (perhaps insane) arguments.

While I can respect the work of such scientists - it is hard to respect a person like Miller who can push such apologetics as the quantum god ploy.

A scientist MUST reject "magic did it". It is the main rule of science. There is no getting around that - no how no way honestly.

A scientist can never accept that lack of knowledge or understanding equals goddidit. A scientist must seek natural answers for all issues of concern. Simply put: there cannot be any "god" (supernatural) in science. Science seeks answers that are natural regarding the relationships, stimuli, reactions, hows, and predictions. Why questions are wrapped up in those natural things and no more for science.

So how does the prime directive of science get reconciled to god belief? My bold answer: it does NOT because it cannot! Miller sounds silly, immature, illogical, disingenuous, or downright dishonest when his great mind tries, as does Collins, or any of the theist scientists.

I respect the work of Miller - but I am finding it hard to respect the man. Why? By way of a small example - if he does not think god performs miracles (except maybe random undetectably small ones!?!?) then why does he not attack the "Lourdes committee" and his RCC and not us atheists who on the face value point of no real miracles agree 100% with him?

Apologetics - work of ignorant, stupid, dishonest and/or disillusion people. Miller, Collins, the likes of them are not ignorant or stupid. Soooooo......

By ConcernedJoe (not verified) on 05 Mar 2010 #permalink

PZ: The New Atheists are as much a force in opposition to creationism as is Ken Miller; more so, I would argue, because we don't make fuzzy, muddled compromises with absurd medieval humbug.

Probably more effective in combination than either would be apart. If you haven't seen it yet, you might want to look at this piece, which Hemant Mehta pointed out. The advantages of ecologic diversity should be readily apparent to an evolutionary biologist. =)

What MAJeff @ #55 & #74 said, with a little amplification: when a journalist calls you, not only have they already written the story, they've already decided what you're going to say. The interview's purpose is to get you to say approximately what they already pencilled you in to say, by whatever means necessary.

I had this done to me a few years back, when I was interviewed on the phone by a journalist, with a company press officer also on the phone. I didn't say what they guy wanted me to say. In fact, I said the exact opposite when asked a direct question. The journalist wrote me as saying what he wanted anyway, with quotes around it.

As soon as the article appeared, I had an angry company vice president on the phone wanting to know why the fuck I'd told the guy that (good job I had the press guy on the phone with me).

Oh, and the subject of the article? Not what the journalist had told me in advance of the interview.

By Brain Hertz (not verified) on 05 Mar 2010 #permalink

I want to clarify one thing - and I am not back tracking.

I do not have a problem with Miller or Collins or whatever great going to church every Sunday, or Saturday, or Friday. Or with them taking communion reverently. Or for thinking J or Mo or Abe or B-baby was great and inspirational. I have no problem and they need not defend themselves to anyone let alone me - if they just leave it at this (excuse a way of saying) "this stuff floats my boat" -- I say great whatever helps give you peace and happiness as long as you don't dictate it on me. No problem... and I would never disrespect or think ill of that.

But Miller, Collins, others.. bring that very personal thing into their professional work. Whether it is very much or just a little they do. And it is wrong.

Atheist attitudes in science are MORE than OK - they are REQUIRED!! You may not be a Atheist - but when you put on a lab coat or take up a pen as as a scientist you better act like one!

By ConcernedJoe (not verified) on 05 Mar 2010 #permalink

Wait--are Celtic Evolution and the Rev Big Dumb Chimp related in some way? Hairy leprechauns!

By recovering catholic (not verified) on 05 Mar 2010 #permalink

Wait--are Celtic Evolution and the Rev Big Dumb Chimp related in some way? Hairy leprechauns!

Heh... not hardly... I just like display proper reverence to the good Reverend for my typos, as he is the King Of Typos. His very presence online makes my fingers slap haphazardly at the wrong keys.

By Celtic_Evolution (not verified) on 05 Mar 2010 #permalink

The Rev. and I both navigate the choppy waters of the IT field for a living, are both shutterbugs, and of course share an affinity for any and all discussions involving pirates, bacon, and lesbians.

But alas, no relation otherwise.

By Celtic_Evolution (not verified) on 05 Mar 2010 #permalink

Atheist attitudes in science are MORE than OK - they are REQUIRED!! You may not be a Atheist - but when you put on a lab coat or take up a pen as as a scientist you better act like one!

1. They don't allow eating in labs generally
2. They equally dont allow babies in labs

Makes acting like an atheist somewhat difficult

tytalus (#86) writes:

"I would be interested to know how one can find 98% of a book acceptable and scientific, and 2% unacceptable and non-scientific, especially when the god argument in question is loaded with scientific terms. It would seem an intentional exercise on Miller's part to make the god argument look science-y."

Ummmm....you do realize that 'Finding Darwin's God' is not a science textbook, but a personal response by Miller to the evo/creo wars? There is no requirement that it be vetted as 100 percent science. The so-called 'God argument' is not presented as a proof or demonstration of God's existence, but how the possible existence of something like the God Miller believes in could be compatible with the present state of scientific knowledge. It's an exercise in philosophy, not science. Miller does not pretend otherwise.

I mean, there are significant chunks of Dr. Dawkins's recent works that are not scientific, either. Consider his wonderful puncturing of Fred Hoyle's misunderstanding of evolution, when he remarks that 'God is the ultimate 747.' Pithy, clever, and I found it personally helpful,but certainly not a scientific claim. Just because Dawkins's books contain exercises in reasoning about metaphysical matters, and are thus not science, doesn't mean that Dawkins has somehow sold science down the river. The same is true with Miller's arguments in this respect. Why impose some sort of 'seamless garment' argument to his popularizations, and not others?

By Scott Hatfield, OM (not verified) on 05 Mar 2010 #permalink

His very presence online makes my fingers slap haphazardly at the wrong keys.

It's all a part of my master plan

By Rev. BigDumbChimp (not verified) on 05 Mar 2010 #permalink

You know.. Most of the scientists are looking or some "other" thing that can explain why quantum effects work on a small scale, but not a large, but there is one guy, and a few researchers, examining the prospect that its about "how much" matter you have, and suggesting there is a threshold at which gravity prevents drastic fluctuations. This seems to me to be a reasonable guess. But, more to the point, it would be damn funny for all these, "God is in the quantum!", BS. It would be, "God is in the quantum, but he is so weak that gravity prevents him doing anything to objects larger than X atoms in size.", which I rather suspect is *not* the outcome these fools are looking for. lol

Scott Hatfield, OM #102

Point taken...

By Celtic_Evolution (not verified) on 05 Mar 2010 #permalink

It's all a part of my master plan

Waht matser plan? we Havve on idea what ou're talknig about.

EwanR 101 -- wow you got me there -- when I clean my screen off I'll try to rebutt :-)

Hi Scott ... your #102..

because Miller and the like actually do place god into science in their musings. To say another way Dawkins in my readings is presenting a counter to "goddidit" not to science. Where Miller is in essence presenting a counter to a fundamental rule of science. We is a scientist arguing against science. Dawkins by example in my view is presenting a counter to those that would inject god into what should be an atheist enterprise.

I am not saying a scientist must be atheist - please read above comment #97. That sentence just for the record - not for you Scott.

By ConcernedJoe (not verified) on 05 Mar 2010 #permalink

My 106 was an unintentional blockquote fail. Should that be considered appropriate, or ironic?

My 106 was an unintentional blockquote fail. Should that be considered appropriate, or ironic?

Both.

By Celtic_Evolution (not verified) on 05 Mar 2010 #permalink

discussions involving pirates, bacon, and lesbians? Count me in

By broboxley (not verified) on 05 Mar 2010 #permalink

PZ #71 wrote:

I'm just saying that in the future I'll feed their appetite by clearly saying that that is all crazy talk, and that what Miller does is lard good science with bullshit.

No, please don't "feed their appetite" by giving them what they want and turning into the mean, raving atheist, the scientific equivalent of Madaleine Murray O'Hare. That would make them so happy. Instead, sit down and figure out a series of short, punchy, reasonable, polite, but devastating criticisms of the Moderate Middle, and trot them blandly out when you think you're being wheeled into position as the Bad Guy. Focus on concepts like honesty, consistency, rigor, and clarity, and use just those words.

And oh, yeah. At some point, say "it really just comes down to love." And pause as if this is a weighty point. It's gold. If the reporter puts it in, the reader is left to figure out what the heck that means, and thus they mentally leave the Mean Atheist Strawman behind.

Scot Hatfield OM #102 wrote:

The so-called 'God argument' is not presented as a proof or demonstration of God's existence, but how the possible existence of something like the God Miller believes in could be compatible with the present state of scientific knowledge. It's an exercise in philosophy, not science. Miller does not pretend otherwise.

But that's the problem. Miller is pretending that it should be an exercise in philosophy, instead of an argument in science.

I've been misquoted by journalists more than once. Asking for a correction or retraction is a futile endeavor.

By 'Tis Himself, OM (not verified) on 05 Mar 2010 #permalink

> Miller doesn't claim that his quantum indeterminacy argument is scientific, so I don't see the point.

The point is that quantum theory actually is a respectable scientific theory. Every time some woo-meister invokes a quantum effect, or quantum indeterminacy, or somesuch to explain, say, faith healing or mind-reading, science suffers. People get the idea that quantum theory is sort of woo-woo and before you know it they're all "More things in Heaven and earth, Horatio..." and we're off to the races.

And all that goes double when an otherwise respectable scientist goes quantum-woo on us.

By InfraredEyes (not verified) on 05 Mar 2010 #permalink

Q.E.D. (#91):

You are incorrect. Miller does not conflate science with religion in his Templeton essay. What he points out is that there is a similarity between the two in that certain propositions are accepted as true without actually being demonstrated.

The difference matters. An effective critique of Miller would not focus on the (false) claim that he is conflating two different things, but how the very real differences between science and religion shape the sort of claims each can make.

By Scott Hatfield, OM (not verified) on 05 Mar 2010 #permalink

@PZ: I think it OUGHT to go without saying. "Live by the sword, die by the sword." You endlessly talk about how you're not going to quiet down or act in a decorous manner, but then you complain when one of your indecorous comments is used.

To extend the cliches a bit, No, you cannot eat this cake.

By https://www.go… (not verified) on 05 Mar 2010 #permalink

Scott Hatfield said in reply to multiple posts:

If you read ["Finding Darwin's God"], you probably wouldn't have a problem with 98 percent. The 2 percent, with the quantum dodge, isn't science but a careful reading of the text shows Miller isn't presenting it as such.

Scott, the problem is that Miller is using fallacious reasoning to support a predetermined conclusion not supported by the data. The fact that he isn't making a "scientific" argument isn't the problem, it's that he isn't even using using sound reasoning. But his status as a scientist magnifies this error. As a scientist who regularly confronts creationists who prop up their own suppositions with selective data-mining, Miller more than anyone should know how profoundly intellectually dishonest such apologetics are. And, yes, when he he engages in the same behavior in order to defend his own religious views he is being hypocritical. The god-of-the-gaps argument is a logical fallacy, as he must know. So why invoke it? And why should anyone pretend that he isn't abandoning the principles of honest, rational inquiry when he does so?

You'd have to be pretty stupid to claim that simply because so-and-so is a scientist, any claim they make is a scientific claim.

Yes, that would be stupid. But it isn't too much to hope that a man invested in and enlightened by the pursuit of science would not abandon all its principles when making a claim about the nature of reality. The principles of the scientific method should inform theology if theology is to be seen as a legitimate pursuit of knowledge. A person who would not accept excuses and handwaving in a laboratory cannot justify accepting excuses and handwaving whenever they're standing inside a church. Bad reasoning is bad reasoning.

Miller doesn't really put quantum indeterminacy forward as a scientific claim, as it is obvious from a careful reading of his text. It does the enterprise of science no harm if individuals defend their personal beliefs with arguments that can not be adjudicated by science, as long as they don't pretend the claims they are making are scientific claims.

This entirely ignores the fact that Miller's arguments are poor ones. Miller did put forward the claim that god operates through quantum indeterminacy. You seem to be suggesting that this claim isn't subject to critical examination on the grounds that the claim isn't strictly scientific in nature. Nonsense. It should be evaluated in the same manner as all claims, including scientific ones. If facts are hard to come by to support some position, then the rational response is to avoid taking such a position, not to whine about that position being challenged even though it was non-scientific speculation. Saying "this isn't a scientific claim" is not a magical mantra to protect a statement from critical examination, an examination which usually begins with looking for supporting evidence. If there is none, then the belief in question should be discarded as a matter of intellectual honesty.

"And all that goes double when an otherwise respectable scientist goes quantum-woo on us."

There is a very real difference between saying 'Quantum IS supernatural' and for making the kind of claim Miller is making. Granted, that claim is irrelevant to physics and the rest of science as is practiced. You are not, however, going to get any traction against Miller's claim by clutching your pearls and crying 'quantum woo'.

By Scott Hatfield, OM (not verified) on 05 Mar 2010 #permalink

In Mr. Miller's defense, he's exactly the kind of person we need to use as an example when talking to audiences who might have a tendency to ignore someone if they're labeled an atheist. He has his Christian Credentials, and hopefully that makes it harder for the real fundies to just stopper their ears when someone starts talking about the evidence.
Not to mention that Miller is a very good presenter. He is clear, concise and doesn't degenerate into nonsense until the very end of his talks.

H.H.:

No one is trying to prevent Miller from being criticized here. I just don't think it's a good rhetorical strategy to go after his supposed lack of scientific ethics, simply because he indulges in a thought experiment that can't be adjudicated by science.

No one is arguing that the argument has to be taken seriously as science, including Miller. It's essentially a religious apologetic, and you don't have to like it. You can criticize it to, ahem, high heaven. That's fine. But don't make the mistake of saying things like 'This argument isn't science, therefore it's false', or 'Dr. Miller is betraying science by writing about religion' or some such nonsense. Honestly, some of you people seem to think science exists to prop up your particular metaphysical position.

By Scott Hatfield, OM (not verified) on 05 Mar 2010 #permalink

Scott Hatfield:

Ummmm....you do realize that 'Finding Darwin's God' is not a science textbook, but a personal response by Miller to the evo/creo wars? There is no requirement that it be vetted as 100 percent science. The so-called 'God argument' is not presented as a proof or demonstration of God's existence, but how the possible existence of something like the God Miller believes in could be compatible with the present state of scientific knowledge. It's an exercise in philosophy, not science. Miller does not pretend otherwise. [...]

Why impose some sort of 'seamless garment' argument to his popularizations, and not others?

I'm not sure I buy this, and I think I mostly don't.

It seems to me that you're begging a central question or two here, which need to be brought out.

As I see it, some central themes of the New Atheism are:

1) Religion is not off limits to rational scrutiny; subject matter of religion overlaps almost entirely with the domain of science, and science can scrutinize pretty much all religious claims, including claims that most people think are beyond scientific scrutiny, e.g., about the purported supernatural entities, morality, and religion itself. There is no defensible NOMA-like line anywhere. If you claim that there is one, you have to make it clear and defend it, because even people who think there is such a line do not agree on where it is.

2) Science and rationalistic philosophy are continuous with each other. There is no line, and certainly no well-defined line, which separates scientific claims "philosophical" or "metaphysical" ones. (Or if there is, you need to make it much, much clearer what you're talking about.)

3) You're allowed to use "philosophical" arguments in science, e.g., to show that a theoretical construct is wrong because it's inconsistent, or that it's very dubious because it is contrived precisely to avoid falsifiability. We do this all the time in science---we don't bother with experiments and data analysis to disprove something that's obviously nonsense, or not even wrong. Nobody has ever given a good reason to exempt religious claims from this sort of commonplace rational scrutiny, and the reasons typically given are clearly invalid because they do not work in many historical cases (e.g., Galileo's).

4) Most specifically religious claims are either evidently false in light of science (e.g., the existence of traditional dualistic souls), or contrived specifically to avoid falsifiability (e.g., liberal theologians' reduced and rarefied concepts of "God" or "souls"). Science is not neutral toward unfalsifiable theoretical constructs. They are typically provisionally discarded as probably wrong. Nobody has ever given good reasons for exempting religious claims from being regarded as wrong, not even wrong, or probably wrong, as we would similar claims not conveniently labeled as "religious."

5) All common religious claims are insupportable in light of modern science, including central claims about gods, souls, divine revelation, and spiritual experiences. We have pretty good scientific reasons to believe that religion in general is a kind of popular delusion, and excellent reasons for doubting the distinctive claims of particular religions.

One of the basic pieces of evidence against religion and religions is that they religions disagree with each other. Most religions must be wrong about at least some central claims, because they can't all be right when they disagree about such basic things as the number or gods, whether such a god is a person, whether a personal god has particular personality traits, whether a god created the universe, whether such creation was ex nihilo, the nature of morality, the relationship between gods and morality, whether the soul is immortal, the nature of any afterlife (e.g., Heaven or Hell us. Reincarnation vs. merger into the Universal Mind vs. other options), the nature of justice, etc.

That kind of argument is often characterized as "philosophical" and therefore somehow not scientific, but that's a basic and enormous category mistake. (Perhaps because it's been around since before what we'd clearly call "science" in the modern sense.)

It is simply not true that an argument can't be both a philosophical argument and a scientific one---and simple common sense, to boot. Many arguments in science are just like that.

Science would not work without this kind of basic rational analysis, and nobody has shown why religion is somehow exempt from it.

6) History shows that exempting religious claims from this kind of scientific scrutiny is typically an error, in hindsight. (E.g., attempts to reconcile Galileo's model of the solar system with scriptural geocentrism, by tweaking it "just a little" so that they're observationally indistinguishable.)

7) That pattern is so clear now that we should be extremely dubious on scientific grounds of what people like Collins and Miller do to preserve orthodoxy, and even what people like Haught and Armstrong do to preserve even minimally contentful "religious" tenets.

8) We therefore can infer scientifically that most common religion is substantially wrong, and that it's likely all wrong to the extent that it has any distinctively "religious" content that is or implies any fact claims, as essentially all religion does. (Even most rarefied modern liberal theology designed to appear consistent with science.)

Empirically speaking, religion tends toward error, and scientifically speaking, that appears to be due to systematic mistakes at the core of religiosity---e.g., belief in divine revelation, or a revelatory character of spiritual experiences.

By default, we should guess that divine revelations are not what they purport to be---most of them clearly are not, and while it's possible that some are, it's not scientifically plausible. (Or at the barest minimum, people's faith in distinctively religious propositions is typically misplaced, so people should generally be much more agnostic than they actually are, and much less self-congratulatory about being religious.)

Likewise, we should guess that personal "spiritual" insight is generally not what it is commonly interpreted as---it's an experience of an odd and extremely error-prone mode of information processing in a machine made out of meat, not anything remotely resembling a soul getting in touch with Deep Truth by any means other than unconscious guessing.

Science and atheism---broadly construed to include agnostic atheism---therefore are a seamless garment. That is largely the point.

New Atheists may differ in where they draw lines between what they choose to label "philosophy" and hat they choose to label "science," but they generally agree that it is a seamless garment of rationality that extends from the obviously sciency stuff like particle physics and biology through things like psychology and neuroscience all they way through things like cognitive anthropology at least---and that's far enough to show that religion is systematically bogus and probably wrong, in something very much like scientific terms, whether you choose to call it that or not.

At first blush, it sounds reasonable when you say that somebody like Miller is not trying to make a seamless garment of science and religion, in the way I claim the New Atheist are making a seamless garment of science and agnostic atheist rationalism.

But in context, and given their stated goals, they're doing something very similar.

The apologetics of Miller and Collins (and even Armstrong and Haught) are in fact largely intended to counter the New Atheist position that science rationally undermines religion.

That's pretty explicit---all those people are claiming that the New Atheists are wrong, and that there is a seam somewhere between science and either religion or positive irreligion. (By the latter I mean atheism in the vernacular sense, or agnosticism in the original sense of denying that anybody can know whether theres a god.)

Miller may not be presenting a seamless garment, but he is clearly trying to show that

1) if there's a seam between science and religion---his versions of each anyway---it's not a particularly awkward and uncomfortable seam; it's a well-stitched and gracefully joined garment at worst, and

2) the New Atheists are wrong to say that they do have a seamless garment.

He fails miserably at both.

No, he's saying "quantum" is not supernatural, but it allows him to smuggle in supernatural causation in evolution.

So he's saying evolution is supernatural (in part), which to some of us is even worse.

But to reject God because of the admitted self-contradictions and logical failings of organized religion would be like rejecting physics because of the inherent contradictions of quantum theory and general relativity. Science, all of science, is necessarily incomplete—this is, in fact, the reason why so many of us find science to be such an invigorating and fulfilling calling. Why, then, should we be surprised that religion is incomplete and contradictory as well?

Look, he is drawing a parallel here between the "incompleteness and contradictoriness" of physics and the "incompleteness and contradictoriness" of religion. He is using quantum theory as an example. You can call it pearl-clutching if you like, but I think it is a categorical mistake to compare theology with science in this way. And, in the particular case of quantum theory, there are already too many people looking for parallels with/explanations for various brands of woo. Miller's comments don't help.

By InfraredEyes (not verified) on 05 Mar 2010 #permalink

Scott Hatfield:

No one is trying to prevent Miller from being criticized here. I just don't think it's a good rhetorical strategy to go after his supposed lack of scientific ethics, simply because he indulges in a thought experiment that can't be adjudicated by science.

But the disparity between Miller's standards of evidence for scientific claims vs. theological claims (i.e., applied skepticism on the one hand vs. wide-eyed credulity on the other) does reveal a lack of consistency which calls his intellectual integrity and ethics into question. If a place of employment were to hire black people only after they have submitted to a drug screening, criminal background check, and provided 3 letters of recommendation while white candidates were hired on the spot unless a third party could provide ironclad evidence that they had committed a felony, it would be impossible to argue that such wildly inconsistent standards are not intrinsically unfair and unethical. Yet Miller relies on an equally disparate and unethical double standard when it comes to evaluating and accepting scientific claims vs. metaphysical ones. As Paul W. said in #120:

Science and rationalistic philosophy are continuous with each other. There is no line, and certainly no well-defined line, which separates scientific claims "philosophical" or "metaphysical" ones.

Miller is guilty of imposing such a line without sufficient justification.

Honestly, some of you people seem to think science exists to prop up your particular metaphysical position.

No, science exists because it eschews metaphysical assumptions. Atheism--the absence of such assumptions--is the only position consistent with and compatible with the principles of science.

Scott Hatfield:

No one is trying to prevent Miller from being criticized here. I just don't think it's a good rhetorical strategy to go after his supposed lack of scientific ethics, simply because he indulges in a thought experiment that can't be adjudicated by science.

The problem is that he's misrepresenting the nature of science to say that the claim can't be at least provisionally adjudicated by science.

His apologetics is analogous to the apologetic position urged on Galileo by some of the Catholic authorities trying to paper over his heresy.

They suggested to Galileo that he should present his heliocentric model as a false but useful one, becuause of course the scriptures are right that the sun goes around the Earth.

They correctly pointed out that an observationally equivalent geocentric model would only be slightly more complicated---all you have to do is add a couple of relativizing axioms, and you can say that the sun goes around the earth in just such a way that it's indistinguishable from the earth as in his original model.

It really was a very small change, smaller than many changes that were made later---e.g., elliptical orbits, not-quite-elliptical orbits with the reduction to Newtons's laws of motion, and especially geodesics through Einsteins relativistic spacetime.

But Galileo was right to resist such a small change, becuase it was a contrivance---the original model was not designed to be unfalsifiable, but the orthodoxy-saving "slightly different" version was. Galileo was right to reject that change, and to give religious considerations essentially zero weight.

Miller, on the other hand, is perfectly willing to weave a just-so story about quantum physics, etc., in order to make theistic evolution observationally indistinguishable from atheistic evolution. (In most ways. It's a bit worse than that in some ways.)

No one is arguing that the argument has to be taken seriously as science, including Miller. It's essentially a religious apologetic, and you don't have to like it.

Oh good, because I really don't like it.

More than that, I don't think saying that it's not "a scientific argument" gets him off the hook.

I think it's pseudoscientific and a kind of selective science denialism that is in fact antiscientific. (In the selective way that antiscience usually is, only more so.)

Let's avoid the term "quantum woo" and say that Miller is engaging instead in quantum fu---a tricky move designed to evade falsifiability, by coming up with a convenient interpretation of quantum mechanics that quantum physicists would think for good scientific reasons is unfalsifiable but still probably false.

Scientists rightly look askance at theories contrived to evade falsification, just as Galileo so rightly did. They make a provisional guess that such things are wrong. (Or what's maybe worse, not even wrong.)

They are not neutral toward unfalsifiable stuff, as accommodationist and compatibilist rhetoric often says or implies. The unfalsifiable is not off limits to scientific judgment. Scientists judge unfalsifiable theories every day, and almost invariably judge them skeptically and harshly.

Giving undue weight to a hypothesis that's contrived to be unfalsifiable is not just "not scientific." It's antiscientific in the same sense that many Young Earth Creationist weaselings are antiscientific---and for exactly the same reason.

Miller is doing what apologists typically do, namely bogifying the science to evade falsification, in order to preserve points of religious orthodoxy.

You can criticize it to, ahem, high heaven. That's fine. But don't make the mistake of saying things like 'This argument isn't science, therefore it's false', or 'Dr. Miller is betraying science by writing about religion' or some such nonsense.

Miller is doing antiscientific apologetics in the same basic way and for the same basic reason as creationists, and he is likewise trying to make it sound like it's not antiscientific.

That is the sense in which Miller literally is a creationist, even if he's not a Creationist in the obvious, prototypical and stereotypical sense. He's doing the same thing for the same reason, just narrowing his scope and doing it more cleverly.

It's still antiscientific.

Honestly, some of you people seem to think science exists to prop up your particular metaphysical position.

Honestly, you're full of shit, on that particular point.

We don't think that science exists to prop up atheism. We think science exists to explain things honestly and as correctly as possible.

We are atheists largely because we take that seriously, and refuse to engage in antiscientific bullshit, and the sort of selective science denialism that Miller engages in.

Science and religion do conflict, and we go with the science.

I, in particular, find modern cognitive science to be pretty compelling stuff, and think it undermines pretty much all religion that's clearly religion. Miller, like most science denialists, glosses over or conveniently bogifies the sciences most relevant to his thesis.

As a scientist, I think that sucks, whether he calls it a scientific theory or not. Calling it apologetics doesn't excuse it, because NOMA and anything remotely like it is false.

@scott hatfield.

you're dead wrong about Miller.

read this:

http://www.millerandlevine.com/evolution/Coyne-Accommodation.htm

towards the end, Miller basically says there would be no science without God:

One can indeed embrace science in every respect, and still ask a deeper question, one in which Coyne seems to have no interest. Why does science work? Why is the world around us organized in a way that makes itself accessible to our powers of logic and intellect?

this is just a simplistic tangent of the "fine tuning" argument.

sorry, but when someone is saying they represent the scientific method, and says something like THAT, he is suffering from severe cognitive dissonance.

Remind me, next time I'm asked about Ken Miller, that I shouldn't bother to say anything appreciative. It will be ignored and won't be reciprocated. And I'm not going to endorse his crusade to taint science with supernaturalism.

If the reporter failed to use all the nice things you said about Miller, what are the odds the reporter failed to use all the nice things Miller said about you?

By timpanogos.wor… (not verified) on 05 Mar 2010 #permalink

Honestly, some of you people seem to think science exists to prop up your particular metaphysical position.

and... Scott will soon exit, stage left.

If the reporter failed to use all the nice things you said about Miller, what are the odds the reporter failed to use all the nice things Miller said about you?

what are the odds that your question has already been asked and answered, and was irrelevant to begin with?

H.H.:

No, science exists because it eschews metaphysical assumptions.

Careful with that. There's a lot of ambiguity in the word "metaphysics" and most philosophers would say you can't get away with not having some metaphysics.

Science eschews metaphysics in the more popular sense, e.g., presuppositions about dualism, and tries to get by with fairly minimal metaphysics in the more technical sense.

Scot Hatfield, OM #119 wrote:

Honestly, some of you people seem to think science exists to prop up your particular metaphysical position.

My understanding is that metaphysics deals with answers to the question "what is the nature of reality?"

Do you think that we should all start out with our particular answers? If not, how do we arrive at them? Through careful methods which evolved to eliminate and correct personal bias and human errors -- or methods which are designed to enshrine them, as sacred?

Of course, per Faith, you have a RIGHT to BELIEVE whatEVER you WANT!

To anyone who would argue with my assessment of Miller: I welcome you to distinguish his position from that of Old Earth Creationism.

Quite easily, just read Only A Theory and see how Ken Miller talks about the evolution of the horse and how that is distinguished between creationism and evolution.

Paul W., warning heeded. And I'd like to say that I'm in full agreement with everything you wrote in comment #124. You'll be getting my vote for the Molly.

Oops, I missed the thread stating that you had just won a Molly. Kudos, Paul W. It's very deserved.

Keep in mind the article is not about me at all -- I do not expect that Miller said anything at all about me, personally. If there was any talk, it would have been about the "New Atheists" in general, and I would be very surprised if he had anything nice to say about that group at all.

Sastra (#112):

"But that's the problem. Miller is pretending that it should be an exercise in philosophy, instead of an argument in science."

???

Do you think the 'many-worlds' interpretation of quantum physics is philosophy, or science? When people start talking about the necessary existence of universes where Walt Disney was elected President of the United States, I turn the vacuum cleaner back on.

I just don't see how those kinds of claims can ever be considered scientific arguments, so I don't see how Miller is guilty of bad faith.

By Scott Hatfield, OM (not verified) on 05 Mar 2010 #permalink

H.H. (#116) wrote:

"Saying "this isn't a scientific claim" is not a magical mantra to protect a statement from critical examination, an examination which usually begins with looking for supporting evidence. If there is none, then the belief in question should be discarded as a matter of intellectual honesty."

Nope, sorry, I disagree. What does 'belief' have to do with scientific inquiry? For the purposes of doing science, all faith-based, evidence-free positions should be discarded. There is no need to make a special case of Miller unless it is clear that he is proposing that a faith-based, evidence-free claim should be accepted as science. I see no evidence this is so.

Let Miller's appeal to quantum indeterminancy be 'X'. Then the argument I would make is as follows:

A critical examination of 'X' shows that it not only lacks supporting evidence but it is not amenable to disproof. It is not a scientific claim and SHOULD be provisionally excluded from the business of doing science.

The possibility that something like 'X' could be tested in the future, however, has not been excluded. While it may seem parsimonious to reject items that have been provisionally excluded, there is no rule that says we have to do that.

By Scott Hatfield, OM (not verified) on 05 Mar 2010 #permalink

how is that not just a restatement of a god of the gaps argument, Scott?

how is Miller saying that science wouldn't even exist as a discipline without god not a fine tuning argument?

I see no evidence this is so.

you're willfully blind to it, you mean.

Scott Hatfield, OM #136 wrote:

Do you think the 'many-worlds' interpretation of quantum physics is philosophy, or science?

I'm not at all up in quantum physics, but my understanding is that 'many-worlds' is a hypothesis which is at least attempting to meet the criteria of science, as an inference to the best explanation of certain data. Its advocates would love to think of a way to test, and possibly falsify it (or not.) I suppose you could make an argument that it's proto-science, or pseudo-science, or even bad science, but it's clear that those who propose it consider it a hypothesis.

I just don't see how those kinds of claims can ever be considered scientific arguments, so I don't see how Miller is guilty of bad faith.

I think the existence of God is a hypothesis; it's more testable than 'many worlds,' because there's a lot of evidence given in support of it.

Ichthyic: (#138)

"how is that not just a restatement of a god of the gaps argument, Scott?"

Some of you guys and gals just aren't applying your usual toolkit to the question at hand. 'God of the gaps' is properly applied to explanations that attempt to plug mysteries with the divine. Miller doesn't appear to be doing that. As PZ correctly noted above, Miller is not claiming that the quantum is supernatural, he's suggesting that the supernatural could be using quantum indeterminacy to game the system in a way not presently detectable to natural science.

I don't actually think much of that argument, but I think it is sufficient to exclude it on methodological grounds. One doesn't need to accuse Miller of bad faith, intellectual dishonesty, etc. to reject the argument.

Some people here have also suggested that 'god of the gaps' is a fallacy of sorts. This is only true, however, if one takes the step of presenting the conclusion of the argument as logically warranted:

A: We don't have any explanation for this mystery 'X'

B: Therefore, the explanation MUST be Mystery God #17

Yes, that's a version of the Argument from Ignorance. But is that really what Miller is arguing? Isn't it true that he's merely showing how he might attempt to be squaring his beliefs with the present state of scientific knowledge? Does he really present his conclusion as the necessary consequence of the facts available to us?

If so, that is a logical fallacy and he should probably think more deeply about his position.

If, on the other hand, he didn't make such an argument, then it seems to be that his position is neither illogical or necessarily fallacious...the hue and cry from outraged partisans here not withstanding.

"how is Miller saying that science wouldn't even exist as a discipline without god not a fine tuning argument?"

I don't follow this. I'm not aware that science requires God to exist. I think the actual argument is that cultures that presumed lawfulness (a fruitful axiom for scientific pursuits) tended to also presume a Lawgiver, and hence tended to foster certain kinds of scientific inquiry. But I don't see necessarily any connection between that assumption and cosmological fine tuning arguments.

"you're willfully blind to it, you mean."

Where is your evidence in support of my previous brief, that "a faith-based, evidence-free claim should be accepted as science?" I don't see it. If,indeed, I am blind, you should be able to point to where Miller makes this assertion chapter-and-verse and I will gladly withdraw my objection.......

By Scott Hatfield, OM (not verified) on 05 Mar 2010 #permalink

'God of the gaps' is properly applied to explanations that attempt to plug mysteries with the divine. Miller doesn't appear to be doing that. As PZ correctly noted above, Miller is not claiming that the quantum is supernatural, he's suggesting that the supernatural could be using quantum indeterminacy to game the system in a way not presently detectable to natural science.

Please explain how these meaningfully differ with respect to a "gaps" argument.

Some people here have also suggested that 'god of the gaps' is a fallacy of sorts.

suggested?

God of the Gaps is an informal logical fallacy where a participant uses a lack (real or presupposed) of mundane explanation for something as evidence of supernatural intervention.

This is a fallacy because just because one is unaware of a mundane explanation, or just because a mundane explanation has not yet been found, does not mean that one does not exist. As such, it is an example of argument from ignorance.

http://skepticwiki.org/index.php/God_of_the_Gaps_Fallacy

scott, seriously, exit yourself stage left.

But I don't see necessarily any connection between that assumption and cosmological fine tuning arguments.

*headdesk*

Scott Hatfield says, "One doesn't need to accuse Miller of bad faith, intellectual dishonesty, etc. to reject the argument."

No, one could instead accuse him of not understanding the first fucking thing about quantum mechanics and using the fact that quantum mechanics seems mysterious to him as a way to justify the unjustifiable.

This isn't a "god of the gaps" argument so much as it is a "god of the--oh, sorry, I thought there was a gap here" argument! In effect, he is saying, "Oh, I don't understand it. It must be God!"

For some reason, Catholics seem to love this sort of argument. Look, has Ken ever bothered to look at how frigging tiny Planck's constant is? Has he ever figured out how many little rigged quantum dice a deity would have to throw to get H2O to turn into sugar, H2O and C2H50H?

This is worse than a fallacy. This is a self-delusion wearing a mask of science.

By a_ray_in_dilbe… (not verified) on 05 Mar 2010 #permalink

Scot Hatfield OM #140 wrote:

Yes, that's a version of the Argument from Ignorance. But is that really what Miller is arguing? Isn't it true that he's merely showing how he might attempt to be squaring his beliefs with the present state of scientific knowledge? Does he really present his conclusion as the necessary consequence of the facts available to us?

Not all 'God of the Gaps' arguments are Arguments from Ignorance. "We don't understand X, therefore God."

Sometimes they're attempts to use a gap -- a hole in our understanding -- to support something, by giving it a hole to live in. "We don't understand X, maybe God is in that thing we don't understand."

Because there is a gap, science can't rule it out. The claim is that science even supports it, because it provides a suggestive hole which suggests God.

If the hole closes, then God moves into the next suggestive gap in our understanding. It does not disappear, because the hypothesis must be saved.

Theistic evolutionists remind me of Doug Adam's story about the man who didn't understand how a television set worked: he thought it was filled with millions of tiny little men moving images around at high speed. After a technologically-savvy friend patiently explained the step-by-step process of how a television set actually worked, the man pronounced himself satisfied. Of course! Now he understands.

But couldn't there still be just one teeny tiny little man inside the television set, somewhere?

Miller's not arguing that there are millions of little men making tv sets work: he fully accepts modern electronics. But maybe you can find a small place for just a very tiny one, where it can hide small enough to be consistent with fully accepting modern electronics, and yet somehow control the whole damn tv set.

Scott wrote:

'God of the gaps' is properly applied to explanations that attempt to plug mysteries with the divine. Miller doesn't appear to be doing that.

No, Miller's doing the reverse, plugging the divine into a mystery. That's still fallacious reasoning, as Sastra correctly elucidates in the previous post.

Sastra wrote:

"We don't understand X, maybe God is in that thing we don't understand." Because there is a gap, science can't rule it out. [...] If the hole closes, then God moves into the next suggestive gap in our understanding. It does not disappear, because the hypothesis must be saved.

And that's the bit that's intellectually dishonest. It's also why Scott just doesn't get it when he writes things like this:

A critical examination of 'X' shows that it not only lacks supporting evidence but it is not amenable to disproof. It is not a scientific claim and SHOULD be provisionally excluded from the business of doing science. The possibility that something like 'X' could be tested in the future, however, has not been excluded.

Scott, we both know the whole point of Miller's apologetics is to find a sufficiently unknowable corner of reality for his god to hide in. He has no interest testing 'X.' He wants 'X' to remain a mystery, because if it is ever solved then he has to go hunting for a new mystery to stick his interventionist god who nevertheless needs to remain unseen. Pretending that god is simply a reasonable proviso in this particular instance is absurd. Theists have been shunting this god of theirs from gap to gap for millennia. They have no intention of ever admitting their hypothesis failed a long time ago. And for you to pretend this apologetics shell game isn't blatantly and obviously intellectually dishonest is very disheartening.

Icthyic: (#142)

My brief 'suggested' refers to the claim that Miller himself is proposing a 'god of the gaps' argument. I did not mean to suggest that something labeled as such is a logical fallacy. As evidence of that fact, I draw your attention again to my post #140, wherein I wrote:

"If so, that is a logical fallacy and he should probably think more deeply about his position."

I regret if my prose style was confusing. But you have to admit that people here are using the term 'god of the gaps' very loosely, and not every sense in which it has been used here refers to the sense which is a logical fallacy sensu stricto...

Sorry about the *headdesk*....I hope it didn't leave a mark. But, again, the first claim I was referencing (which concerns the historical relationship between science and theism) is distinguishable from the second claim of cosmic fine tuning. I mean, this is so obviously the case I fail to see why your head slipped. I invite you to explain how they are indistinguishable or necessarily linked.

By Scott Hatfield, OM (not verified) on 05 Mar 2010 #permalink

I'd recommend St. Werner, but you can't know for sure how those would turn out.

How about St. Hugh? He grants all your prayers, though you might not know it.

Scott Hatfield:

As nice a guy as you may be, you still have yet to explain why anyone ought to believe in your Christian deity. You're nice and all, but dude you've got nothing.

You're one of the nicest, most polite commenters here. But your theology is just as stupid and vacant as Ken Miller's, or anyone else's.

By Josh, Official… (not verified) on 05 Mar 2010 #permalink

Oh, and you know what else, Scott Hatfield? You're completely full of shit. You show up here as the "reasonable theist," all polite and Mooney-Approved. But you have no defensible or intellectually sound rationale for your adherence to Christianity.

So, how about you pony up some logically defensible reason why anyone should take your Christian apologetics seriously?

By Josh, Official… (not verified) on 05 Mar 2010 #permalink

H.H. (#147) writes:

"Scott, we both know the whole point of Miller's apologetics is to find a sufficiently unknowable corner of reality for his god to hide in. He has no interest testing 'X."

Actually, I'm not sure that we know that, only that our mutual experiences with religious apologists may lead both of us to suspect that they habitually reason from their preferred conclusion, rather than toward it.

But let's say that this is true. I dunno, could be. I'm not Ken Miller, I don't know what goes on in his heart of hearts. But let's grant it.

What difference does it make what motivates Ken Miller?

Does that have any bearing one way or the other on whether his notion is scientific? (It isn't)

Does it have any bearing on the question of whether the argument is made in the imperative form that would render it a logical fallacy? (As far as I can see, this claim has not been demonstrated)

Does it have any bearing or other on whether the conclusion of his argument is logically permissible? (It is permissible, by the way)

Thinking clearly, Ken Miller's motives do not appear to have any bearing on any of those questions. Why bring it up? Would you reject an argument made against the misconduct of a Catholic prelate simply because the person making the argument was known to reject Catholic teachings? I doubt it. I suspect that you would like to believe that you could dispassionately evaluate the claim itself.

As would I, where Miller is concerned. I am not much impressed by his argument, as I mentioned before. But I would hope that I would not make the mistake of giving either a 'thumbs up' or 'thumbs down' based on his presumed motives.

By Scott Hatfield, OM (not verified) on 05 Mar 2010 #permalink

@ Scott Hatfield:

But I would hope that I would not make the mistake of giving either a 'thumbs up' or 'thumbs down' based on his presumed motives.

See, I will give you a "thumbs down" based on your motives. Your motive is clearly to apologize for a Christian world view. And you have absolutely nothing to back it up,aside from your emotional allegiances.

Come on, Scott. What's your best argument? Cut the "I'm a nice, non-threatening Pharyngulite commenter" bullshit. I don't give one whit that you're the favorite-Christian-commenter-who-gets the-benefit-of-the-doubt. What's your argument?

By Josh, Official… (not verified) on 05 Mar 2010 #permalink

So, what is it Scott? What's your evidence for your god? Don't talk to me about Ken Miller, I don't care about that.

I want to hear your evidence for your god.

Come on. It's not hard.

By Josh, Official… (not verified) on 05 Mar 2010 #permalink

Josh, Official SpokesGay: (#151)

My comments here were not intended to persuade anyone of any particular theological position. I am not engaging in Christian apologetics in this thread. If that is your impression, you are mistaken.

I happen to hold the opinion that it is bad manners to have those kinds of conversations at this blog, where I consider myself a guest. Anyone who wishes to give me personal grief and have a go at my particular beliefs can blast me on my own blog. If you're genuinely interested, you can harangue me there, at a post summarizing my take on PZ and Miller:

http://monkeytrials.blogspot.com/2010/03/myers-miffed-miller-mocked.html

If, on the other hand, you just want to try to make a few remarks at my expense in front of the hordes of loyal Pharynguloids, count me amused by not terribly interested. I don't come to Pharyngula to push my beliefs, but neither am I here to be your metaphysical punching bag.

(sigh) Please don't compare me to Chris Mooney. I don't agree with the tack that he and Nisbet have taken where science communication is concerned. In fact, Nisbet kind of quote-mined me once, which really hacked me off. I much prefer the rough-and-tumble environment here to their attempt to paper over real points of disagreement in the name of 'framing.' I am not a 'Mooney-Approved' voice, I am my own voice. I think you should carefully consider why you so clearly want to lump me into a particular category for dismissal.

By Scott Hatfield, OM (not verified) on 05 Mar 2010 #permalink

Well, Josh, you asked Scott, not me, but I'm going to answer anyway.

I want to hear your evidence for your god.

None. Zilch.

Let me go further: there is no evidence (that I'm aware of) for any god at all.

"Evidence" must, by definition, be demonstrable and verifiable by other people. The only method I'm aware of to construct a mutually-agreed-on definition of reality that transcends culture is the scientific method.

What I have, and what I presume Scott and Ken Miller have also, is personal experiences. What I have done is to take my personal experiences and fit them to a cultural framework. In so doing, I have very consciously adopted those parts of the framework that seem to me ethical, or at least neutrally benign, and reject those that are evil.

The God that I've constructed for myself is, therefore, what I've called in the past "a pink and fluffy God". S/he is represented by Jesus Christ, also a personal construct, who may (or may not) have lived on earth as a Jewish rabbi who came to relieve suffering, advocate for justice, and overcome death through selfless love for everyone. The Christ represents my aspiration to overcome my own tendency to self-absorption and turn my attention, time, and material wealth towards other people. It is a way to reinforce my desire to be a giver instead of a taker.

I don't participate too much any more in organized religious activities, however, because I see quite clearly that basing decisions on what we religious people erroneously assume to be a shared and identical experience of God is very, very dangerous. Personal experiences are not evidence.

Any group of people who come together on such a basis are all too apt to turn into an echo chamber that reinforces culturally-engendered prejudices, a dangerous submission to authority, and an us-versus-them mentality that inflames our natural impulse towards tribal violence.

By this of course I mean real violence, not the hot language that we sometimes use here. Pharyngula is many things, but it is not an echo chamber.

By Leigh Williams… (not verified) on 05 Mar 2010 #permalink

Scott Hatfield, #140:

Some of you guys and gals just aren't applying your usual toolkit to the question at hand. 'God of the gaps' is properly applied to explanations that attempt to plug mysteries with the divine. Miller doesn't appear to be doing that. As PZ correctly noted above, Miller is not claiming that the quantum is supernatural, he's suggesting that the supernatural could be using quantum indeterminacy to game the system in a way not presently detectable to natural science. [my emphasis]

Is there any difference? You know what the word "indeterminacy" means, right? I don't understand QM, nor does anyone else, but let me try to explain it. We can't precisely measure events at those scales, and thus are partly ignorant of them. Hence, there is a gap (stuff we can't know or detect), and he's suggesting that gap is where Yahweh's hiding, fiddling with knobs while dutifully hating amputees. The stupid goes all the way down.

Now, I'd also like you to explain your crazy-ass religious beliefs. I don't actually want to hear apologetics. No, I've already had plenty of that. However, I've been waiting so long for some religious person to support their beliefs with rational argument. It hasn't happened yet. Would you like to do that? Somehow I doubt it.

Leigh Williams, Queen of Cognitive Dissonance:

What I have, and what I presume Scott and Ken Miller have also, is personal experiences. What I have done is to take my personal experiences and fit them to a cultural framework. In so doing, I have very consciously adopted those parts of the framework that seem to me ethical, or at least neutrally benign, and reject those that are evil.

I also have personal experiences, fit into cultural frameworks. I think they're mostly ethical and benign too. Would you please be more specific?

The God that I've constructed for myself is, therefore, what I've called in the past "a pink and fluffy God". S/he is represented by Jesus Christ, also a personal construct, who may (or may not) have lived on earth as a Jewish rabbi who came to relieve suffering, advocate for justice, and overcome death through selfless love for everyone. The Christ represents my aspiration to overcome my own tendency to self-absorption and turn my attention, time, and material wealth towards other people. It is a way to reinforce my desire to be a giver instead of a taker.

So you're an atheist then, albeit one with an unhealthy penchant for anthropomorphic metaphors. It is at least humorous. Welcome to the party.

Oh no, I'm not an atheist. God is very real . . . to me.

I have no desire -- not to mention, no ability -- to convince you that s/he is real, and in fact would consider that to be very disrespectful. I'm content to let you have your personal life ordered as you like it.

As far as our corporate, political, and societal life goes, I think it's much for the best that we all agree that decisions should be evidence-based.

By Leigh Williams… (not verified) on 05 Mar 2010 #permalink

Sorry, I overlooked your request for more specificity.

Let's take, for example, the issue of gay marriage. The priests who wrote Leviticus called homosexual sex an abomination (exactly the same word they used for wearing mixed-fiber clothing and eating shellfish). St. Paul also seems to have found it rather squicky, perhaps a culturally-induced bias due to his Jewish upbringing, though some argue that he was condemning temple prostitution. I don't care which it was.

To me it is a clear civil rights issue in which a minority is denied a benefit by the majority. There is no evidence, none at all, that societal harm will come from it. Ethically the choice is therefore clear, and I am an advocate for gay marriage. I see no reason that the prejudices rampant in several-thousands-year-old Jewish culture should be legally enforceable in U.S. law. Happily, most modern-day Jews agree, with the lamentable exception of the Orthodox, who share a dim view of sexual equality of all kinds with their fundamentalist Muslim and Christian brethren.

By Leigh Williams… (not verified) on 05 Mar 2010 #permalink

Oh no, I'm not an atheist. God is very real . . . to me.

Are you for real? Either God is real, or God is not real. Not real to you, or real to me, or real to Santa Claus, just real. (Do you think it makes your case better by having to qualify it as "very real"??) Don't take this personally, but it's hard to take you seriously when you say things like the following, then insist that you don't actually mean any of it as it is plainly written on the page:

The God that I've constructed for myself is, therefore, what I've called in the past "a pink and fluffy God". S/he is represented by Jesus Christ, also a personal construct, who may (or may not) have lived on earth.... The Christ represents my aspiration.... It is a way to reinforce my desire....

Just be a good person because you want to be a good person.

You don't need to believe Santa's real to give people Christmas presents, do you? On the contrary, if you did think Santa was real, your desire to give presents might not be reinforced but diminished, since you know he's jetting around the world giving people stuff. It's basically same deal with your invisible pink fluffy thing. It's based on nothing, answers nothing, and can lead to making a lot of bad choices.

Leigh: I can't speak for Ken Miller, but you and I are certainly sympatico in the sense that we recognize that our personal experiences do not count as evidence.

Mr. T: With respect to my earlier brief, at #157 you ask: "Is there any difference?"

If you will read my earlier comments, this has to do with the question of whether Miller is committing a fallacy when he suggests where (in your words) Yahweh might be hiding out, hating amputees. I argued that what is recognized as a logical fallacy only comes into play when it is framed in the IMPERATIVE form. That is to say, an argument of the form "There is mystery, therefore, it MUST be Hidden-Hater-of-Amputees." Miller, on the other hand is indulging a possibility: the Hidden-Hater-of-Amputees MIGHT be acting, in a way not detectable, such as in the quantum world.

I am not advocating for Miller's viewpoint when I point out that this is not a logical fallacy. I am implicitly arguing that he is not guilty of bad faith with respect to the scientific enterprise in entertaining this prospect as a personal matter.

By the way, I take the same stance with respect to 'The God Delusion'. Dawkins doesn't damage the scientific enterprise by expressing his personal views on God's non-existence, nor does he attempt to claim that his arguments impel non-belief. He goes out of his way to acknowledge that he is not making such a claim, and that there is a remote possibility that he is wrong. Many people who no doubt view themselves as his acolytes would profit from a closer reading of their idol.

By Scott Hatfield, OM (not verified) on 05 Mar 2010 #permalink

I am not advocating for Miller's viewpoint when I point out that this is not a logical fallacy.

nope, in this case you're just being flat out wrong.

it's been directly pointed out to you that Miller's argument is in fact, an exact god of the gaps argument, i even quoted the damn definition for you so you could see how well it fits, and you simply, willfully, refuse to acknowledge it.

see why atheists think theists have a problem with reality, no matter how reasonable they think they are?

there is a fundamental dissonance there, period.

neither you, nor Miller, nor the fucking Pope hisself has ever shown that there is value in maintaining that dissonance.

By the way, I take the same stance with respect to 'The God Delusion'. Dawkins doesn't damage the scientific enterprise by expressing his personal views on God's non-existence, nor does he attempt to claim that his arguments impel non-belief.

then you would be WRONG, again, since Dawkins is not trying to hide god in an indeterminacy, like Miller is trying to do, he is attacking poor theology, while Miller is not only CREATING poor theology, but muddling science with it as well.

so, no, I do not grant you equivalence.

Dawkins is talking about religion on its own merits, while Miller is trying to talk about science using religion's merits.

NOT THE SAME THING.

I really hate religious apologists. no matter how nice and friendly they are, they always muddle things up in the end.

I am implicitly arguing that he is not guilty of bad faith with respect to the scientific enterprise in entertaining this prospect as a personal matter.

boy you are really getting my goat tonight, Scott.

what part of "published his inane screed in novel form" is part of "personal matter" to you?

or how bout:

published as public rebuttal to those who criticized his accomodationist stance?

Indeed, if Miller would simply stop publicly and vehemently trying to create inane fallacious arguments to stuff his god into, nobody would have even bothered.

hell, there are likely millions of individuals with bizarre rationalizations for how they manage to compartmentalize their religious nonsense from reality. They just don't claim to have science as an authority behind them for publishing them in fucking book form.

Both Collins and Miller have the same problem, btw.

though I have to say Collins is way off the deep end any more.

Miller, on the other hand is indulging a possibility.

What exactly about quantum indeterminacy allows for an interventionist deity? What scientific evidence is there that leads one to "indulge" in such an idea?

If it's not simply that he's arguing from ignorance, then what supports it? Is he not making an argument, but only "indulging" himself by asserting without evidence that there's some kind of possibility?

Lots of things can lead to making bad choices. Religion is high on the list; but then again, so are patriotism, materialism, tribal loyalties, ignorance, and plain stupidity. On the personal level, we've got selfishness, narcissism, addictions of multiple kinds, and again, plain stupidity. Obviously these are by no means comprehensive lists.

We appear to agree that our public and societal lives should be evidence-driven, and I presume that we also agree that we can, as a society, arrive at a system of ethics that is evidence-driven.

On what basis, therefore, do you demand that I give up a religious belief that is important to me? Because you fear that it might, just might, cause me (me personally) to make bad choices in my life? What business is that of yours?

If you want to argue that religion has historically been used to provide protective coloration for greed, sexual corruption, and colonialism (among many other bad things), I agree with you based on the evidence. If you want to argue that it should be kept out of governance and particularly out of education, I agree again; I think it should be a purely private matter.

But if your point is that I shouldn't believe in God because you don't think he's real, I'm going to tell you to mind your own business and leave mine to me.

By Leigh Williams… (not verified) on 05 Mar 2010 #permalink

Leigh:

On what basis, therefore, do you demand that I give up a religious belief that is important to me? Because you fear that it might, just might, cause me (me personally) to make bad choices in my life? What business is that of yours?

I try to say what I actually mean without confusing myself and others with a string of metaphors and other ways of knowing. By my count, I only wrote two demands. The rest were questions and statements.

Don't take this personally...

Just be a good person because you want to be a good person.

I think these are reasonable. If you have a problem with either, then please feel free not to comply with them.

I'd like to know on what basis you think you are capable of anything remotely like this: "The God that I've constructed for myself...".

Hubris and self-centeredness come in many forms.

If you're really busying yourself constructing gods, then as a fellow participant in this universe, I personally would like to know. However, if you're just bullshitting and playing pretend, then I couldn't care less so long as no one is harmed.

Leigh Williams, QoCD

if you agree that religion is to be a purely private matter, have no business in our public and societal lives, ethics, governance, education, ... what's left for religion as an organisation, as a body of knowledge, or even as a subject of discussion ?

By negentropyeater (not verified) on 05 Mar 2010 #permalink

Just be a good person because you want to be a good person.

many people apparently need role models to motivate them in doing just that.

By negentropyeater (not verified) on 05 Mar 2010 #permalink

PZ #2

But at least now if any reporter wants to hear my views on Miller, I'll be sure to give them lots of juicy biting quotes.

Don't let 'em break you. Hold your nerve.

gillt #13

Shame on you PZ....Censor yourself!

Well, that might not be a bad idea, pragmatically speaking!

MAJeff #55

Find a nice way to fit your point into the story, and keep going back to it.

Sounds good to me.

PZ #90

That sounds like intellectual cowardice.

I disagree. You know what the media is like. So you need to accomodate that. Not diluting your arguement, but by making it positive, not negative. If you don't say anything negative, they can't make you out to be a meanie.

Celtic_Evolution #92

I'd rather be taken out of context and thought of as a nasty old atheist asshole and know that I was honest and forthright

Once again, I disagree. There's good pride and there's bad pride. Methinks you're promoting the bad kind of pride. ;-) I am not necessarily promoting the Dale Carnegie-fu here, but he recommended with good reason to never criticize. This is of course ridiculous if taken literally. But the principle of it is worth reflecting on.

Sastra #111

Instead, sit down and figure out a series of short, punchy, reasonable, polite, but devastating criticisms of the Moderate Middle, and trot them blandly out when you think you're being wheeled into position as the Bad Guy.

Makes sense. Especially talking about the love!

negentropyeater #169

if you agree that religion is to be a purely private matter... what's left for religion?

A personal framework, maybe? Like art, but not quite. For example, I'm not religious, but I care a lot about art. I'd say that the true enemies of civilization and anything good in the universe have one thing in common: they try to censor art. To me, art is sacred.

So religion can be like an art for living, as well as inspiring subject matter. I don't know, I'm just trying to see if the question can be answered.

By Pikemann Urge (not verified) on 06 Mar 2010 #permalink

Scott:
"a few remarks at my expense in front of the hordes of loyal Pharynguloids"
...
"Many people who no doubt view themselves as [Dawkins'] acolytes would profit from a closer reading of their idol."

I was just going to defend you against the outrageous charge of being a Mooninite, but what's all this stereotyping then?

"Dawkins doesn't [...] attempt to claim that his arguments impel non-belief. He goes out of his way to acknowledge that he is not making such a claim, and that there is a remote possibility that he is wrong."

What book did you read? What would have been the the point of going on about the "God Hypothesis" and scientific evidence, if Dawkins was just talking about a "personal matter"? There's always the possibility that we might be wrong about any claim, that's not the issue here.

It seems that you are trying to say that we can't have a double standard where Dawkins is allowed to write popular books endorsing atheism but Miller is attacked for writing popular books endorsing Catholicism? I agree, but I don't think anyone is arguing for that exactly, but against the specifics of Miller's claims.

A personal framework, maybe?

A personal framework about what ?

By negentropyeater (not verified) on 06 Mar 2010 #permalink

It's more that Miller is attacked for writing books which claim, falsely, that the findings of science support Catholicism, whereas Dawkins is praised for writing books which claim, correctly, that the evidence supports atheism (no positive evidence for any deity). Trying to render it all as personal endorsements is to miss the point.

By Stephen Wells (not verified) on 06 Mar 2010 #permalink

Mr. T:

If you're really busying yourself constructing gods, then as a fellow participant in this universe, I personally would like to know.

Perhaps my choice of words was infelicitous; I thought it obvious I was using "construct" in its verb meaning, "To create (an argument or a sentence, for example) by systematically arranging ideas or terms" and in its noun meaning, "A concept, model, or schematic idea."

What, did you imagine me molding a golden calf with laser eyes, or some such?

negentropyeater:

if you agree that religion is to be a purely private matter, have no business in our public and societal lives, ethics, governance, education, ... what's left for religion as an organisation, as a body of knowledge, or even as a subject of discussion?

As an organization, service to the community. That's what we're explicitly called to do: feed the hungry, comfort the sick (medical attention works well for this!), visit the prisoners, advocate for social justice. My reading of the New Testament doesn't indicate we should engage in politics or fight in holy wars (even culture wars); quite the contary, in fact.

As a body of knowledge, to provide a benevolent and ethical framework into which those of us subject to apprehensions of diety (neurological storms of some kind? neurotic need? weakmindedness? perhaps), which is apparently a sizable proportion of the population, can understand those experiences and use them to do good instead of harm to society.

As a subject of discussion where? In the body politic? No; though our opinions about issues such as universal health care might be informed by our faith, I think we need to present evidence that providing it is both ethical and sensible. Amongst our co-religionists? Yes, if only to persuade them to follow Christ instead of James Dobson and point out to them the dangers of theocracy. Amongst fellow theists of other faiths? Yes, to find common ground and lessen sectarian strife. With atheists? Yes, to work together toward our mutual goals of educating the young in secular public schools that teach real science rather than mythology, and preserving a secular democracy from encroaching theocracy.

It seems to me that Ken Miller has devoted his life's work to these last two goals. He wrote Finding Darwin's God specifically to counter creationism and persuade our co-religionists away from the science-as-demon hogwash they're being fed from some pulpits.

Do you really think that two chapters in this influential book negate the value of his voice for science, however much wishful thinking about quantum mechanics he may have speculatively considered?

By Leigh Williams… (not verified) on 06 Mar 2010 #permalink

Personal experiences are not evidence.

Eyewitness (and other-organ-witness) testimony is evidence, it's just usually not very good.

Catholicism, like most religions, presumes a directly interventionalist god(s). People go to Lourdes in hopes that they'll throw away their crutches and dance in the streets because god performs magic on them. (Unless they're a dirty amputee, in which case god can't be bothered.)

Miller is a Catholic. His god hangs around Heaven, chatting up the saints, and arbitrarily answering prayers in a distinctly interventionalist, magical manner. No matter how hard Miller tries to squeeze his god into Planck's Constant sized gaps, it doesn't work. The differences between Miller's god and Pat Robertson's god are so minor as to be indistinguishable to the non-believer.

By 'Tis Himself, OM (not verified) on 06 Mar 2010 #permalink

As an organization, service to the community. That's what we're explicitly called to do: feed the hungry, comfort the sick (medical attention works well for this!), visit the prisoners, advocate for social justice.

All these services can and must be performed by the community without the need to be called for by a religion.
If an organization does the same thing whether it's religious or not, why call it religious ?

My reading of the New Testament doesn't indicate we should engage in politics or fight in holy wars (even culture wars); quite the contary, in fact.

Why the need to read the New Testament to know that we must feed the hungry ?

As a body of knowledge, to provide a benevolent and ethical framework into which those of us subject to apprehensions of diety (neurological storms of some kind? neurotic need? weakmindedness? perhaps), which is apparently a sizable proportion of the population, can understand those experiences and use them to do good instead of harm to society.

But where do people get apprehensions of deity if not from religions ?

Same thing with a subject of discussion, aren't you saying that the role of religions is to deal with the problems caused by religions ?

Look, I've already shared that I've had similar strong experiences as you, Scott and Ken Miller. But why the need to fit them into a cultural framework that's evidently imposed by other people ?
I just don't see what benefit you get from this, and I can see a lot of disadvantages of doing so. Maybe you can explain more about this.

So even if people have strong personal experiences that they might interpret as communication with God, why not just stop there, and as you rightfully say, leave it to a purely private matter ? It seems to me that as soon as they start trying to fit this into preexistant cultural frameworks, whichever it is, they put themselves at the mercy of all the pitfalls of religions.

By negentropyeater (not verified) on 06 Mar 2010 #permalink

The differences between Miller's god and Pat Robertson's god are so minor as to be indistinguishable to the non-believer.

Well they are both figments of their imagination, but it seems to me Pat Robertson's god is more of an asshole.

By negentropyeater (not verified) on 06 Mar 2010 #permalink

Leigh Williams - re: believing in "god" scientist or not: "...if they just leave it at this (excuse a way of saying) 'this stuff floats my boat' -- I say great whatever helps give you peace and happiness as long as you don't dictate it on me. ... [But] Atheist attitudes in science are MORE than OK - they are REQUIRED!! You may not be a Atheist - but when you put on a lab coat or take up a pen as as a scientist you better act like one!"

People at the stature of Miller and Collins are using their credentials as scientists to fit god into science and even worse undermine science by providing stealth-like cover to those that would actively and viciously undermine the enterprise. Again - anyone - even an esteemed scientist who just says [and leaves it at this] "I just believe - it gives me peace and I FEEL better for it - it is just a personal thing and my felt belief and any god have NO place in science" I for one would NEVER call any foul committed nor BTW think that person silly; for me it would be ridiculous but hey if it floats your boat and you do no harm go for it.

To the general defenders of Miller again I say: "because Miller and the like actually do place god into science in their musings. To say another way Dawkins in my readings is presenting a counter to "goddidit" not [a counterpoint] to science. Where Miller is in essence presenting a counter to a fundamental rule of science [That seems dishonest for a SCIENTIST]. [H]e is a scientist arguing against [the fundamental rule of] science. Dawkins by example in my view is presenting a counter to those that would inject god into what should be an atheist enterprise. [That seems legit for a SCIENTIST]"

I am not saying a scientist must be atheist - please read above.

By ConcernedJoe (not verified) on 06 Mar 2010 #permalink

Einstein: God does not play dice with the universe.
Bohr: Einstein, stop telling God what to do.
Miller: God only plays dice with the universe.
Myers: For fuck's sake.

Seriously? God's interaction is restricted to setting parameter values on quantum distributions at the beginning of time? Does He look like this?

So what kind of spin/charge should we expect for the theoretical but soon to be discovered Jes-on (forgiveness particle)?

By Antiochus Epiphanes (not verified) on 06 Mar 2010 #permalink

Frankly, I find Miller's God of the Bandgaps even more pernicious than the God of the Gaps. The latter is bad enough. It leads to people opposing scientific research into the white spaces of scientific knowledge. However, the God of the Bandgaps seems to require a deity who hides in the shadows away from the most precise instruments of scientific measurement--a deceitful deity. If I were an Xtian, I think that I would be especially wary of positing a deity that was subject to Bell's Inequality.

Frankly, I find Leigh's deity to be more palatable. Her deity seems to be more like the deities in Terry Pratchett's diskworld--an anthropic personification of the orderliness of the Universe. While it may be unsatisfying to be unable to resolve this question empirically, at least such a deity doesn't find it necessary to continually jam its divine thumbs into the celestial clockwork.

By a_ray_in_dilbe… (not verified) on 06 Mar 2010 #permalink

By the way, I take the same stance with respect to 'The God Delusion'. Dawkins doesn't damage the scientific enterprise by expressing his personal views on God's non-existence, nor does he attempt to claim that his arguments impel non-belief. He goes out of his way to acknowledge that he is not making such a claim, and that there is a remote possibility that he is wrong. Many people who no doubt view themselves as his acolytes would profit from a closer reading of their idol.

You're right that he doesn't claim to have disproven the God Hypothesis.

But... did you seriously miss the fact that Dawkins immediately goes on to make an argument from parsimony, and a very forceful one at that?

Miller, likewise correctly recognizing that arguments from parsimony aren't proofs, goes on to make an argument against parsimony. (Or at least he tries.) That puts him at odds with science, which means he needs a defense that he apparently hasn't put forth yet; Dawkins, however, doesn't make a single unscientific inference as far as I can see.

"I just believe - it gives me peace and I FEEL better for it -

I don't understand that at all. I mean, if I believed everything that made me feel better, I'd be either dead or institutionalized. (Or maybe both.)

Einstein: God does not play dice with the universe.
Bohr: Einstein, stop telling God what to do.
Miller: God only plays dice with the universe.
Myers: For fuck's sake.

LOL! Perfect! :-D

forgiveness particle

:-)

If I were an Xtian, I think that I would be especially wary of positing a deity that was subject to Bell's Inequality.

Is Miller's god a local hidden variable...?

By David Marjanović (not verified) on 06 Mar 2010 #permalink

Ichthyic (#163 and onward):

"it's been directly pointed out to you that Miller's argument is in fact, an exact god of the gaps argument, i even quoted the damn definition for you so you could see how well it fits, and you simply, willfully, refuse to acknowledge it."

No. I understand the definition. I've claimed that, in order for an argument to be fallacious, it must have a logical structure whose outcome is imperative. I've made that point more than once in this thread. I will make it again.

Ex. 1

Brand 'X' is favored by 9 out of 10 doctors.

Therefore, Brand 'X' is the best brand.

This is a 'bandwagon' argument, and a logical fallacy. Brand 'X' may be the best brand, but the claim that 9 out of 10 doctors favor it is not conclusive. 9 out of 10 doctors could be wrong.

But suppose we say this....

Ex. 2

Brand 'X' is favored by 9 out of 10 doctors.

This is evidence that the medical profession is well-disposed toward the merits of Brand 'X'.

This makes pretty much the same appeal to a herd mentality, but it is NOT a logical fallacy.

My claim is that in his PUBLIC pronouncements about his PRIVATE beliefs, Miller makes statements that are more like Ex. 2 (conditional, provisional, non-imperative) than Ex. 1 (imperative, and clearly fallacious).

That doesn't mean, by the way, that Miller's argument is convincing. It doesn't convince me, for example, precisely because it is not amenable to test.

By Scott Hatfield, OM (not verified) on 06 Mar 2010 #permalink

I think Scott would agree with me that the following argument contains a logical fallacy:

1. We have an incomplete understanding of X.
2. Therefore, there is an invisible dragon in my garage.

I would further assert that it is useful to think of the following argument as also containing a logical fallacy:

1. We have an incomplete understanding of X.
2. This provides any justification whatsoever for my belief that there is an invisible dragon in my garage.

Maybe Miller is not committing a logical fallacy - maybe he doesn't think that our incomplete understanding of X is in any way connected to his belief in god. As Scott points out, I'm not Ken Miller.

Oh, I missed nice stuff last night!

Leigh Williams, Queen of Cognitive Dissonance #156 wrote:

What I have, and what I presume Scott and Ken Miller have also, is personal experiences. What I have done is to take my personal experiences and fit them to a cultural framework.

And what you apparently did not do, was try to analyze your personal experiences against the framework of modern science, to test your interpretation of said experience. Personal experience is indeed evidence, and can be fit into a larger, intersubjective model of reality.

Consider this analogy: a person who is lost in the desert sees an oasis which disappears when he approaches it. He concludes that he has come across a magical desert-based Brigadoon. After he is rescued, he reads extensive literature on the desert, and discovers that it is very common to hallucinate under certain conditions. He studies the neurological and psychological explanations for hallucinating an oasis in the desert.

But his conclusion re Brigadoon does not change, because he had a personal experience. It's not open to investigation, because he was there, and he saw what he saw, and nobody else can say he didn't. Not for sure.

The God that I've constructed for myself is, therefore, what I've called in the past "a pink and fluffy God."

Serious question here: does it matter to you if God exists as anything other than a useful mental construct? Would it change anything if it doesn't? Would it change anything, even for you?

Oh no, I'm not an atheist. God is very real . . . to me.

"Real" as in factually true, or "real" as in personally valid? Does the distinction matter?

But if your point is that I shouldn't believe in God because you don't think he's real, I'm going to tell you to mind your own business and leave mine to me.

I think our point is that you shouldn't believe in God because it's not real -- and your point seems to be that this doesn't -- and wouldn't -- matter to you.

SaintStephen,

Maybe it's neither here nor there, but what does a smart scientist like Ken Miller say about the problem of infinite regression? Who, or what, created this "thing" that acts through quantum indeterminacy in our physical world? Has he ever answered such a question?

I've asked why infinite regression isn't the "deal-breaker" for religion before, in my earliest days as an atheist, but lately it seems like nobody even uses this argument anymore. Michael Ruse dismissed it out of hand recently in a video on RD.net, without even offering a reason. He just scoffed at it, and that was it. No explanation whatsoever.

There are a couple of reasons why regression really isn't a deal breaker, in itself---even infinite regression. It's only a deal-breaker if the person ignoring the regress says something contradictory to accepting the regress, which they usually do, at least implicitly.

Regresses do happen in real science, and we have to acknowledge that.

Occam's razor isn't always right, and sometimes data come up that tell you the simpler hypothesis is wrong, and a more complex hypothesis you'd provisionally discarded may be right.

Here's a hypothetical example.

Suppose somebody actually came up with serious evidence for intelligent design---e.g., DNA sequences which were decoded and turned out to be statements of interesting verifiable facts, e.g., stuff about quantum physics and relativity and an underlying Theory of Everything, and maybe plans for a workable warp drive.

As far as we can tell, there's no way such a thing could have evolved naturally. Something that was already intelligent enough to understand quantum physics and such would have to have constructed it, and left it in our genes as an easter egg.

All the sudden, intelligent design would be a live scientific hypothesis, and in fact far and away the leading hypothesis. Not the BS Intelligent Design of the ID movement, but a real, scientific intelligent design hypothesis with evidence behind it.

In that case, we'd have to accept that we had found something like a 747, and that we have a regress on our hands---who designed it, and where the heck did they come from?

The basic lesson there is a regress doesn't necessarily make a hypothesis wrong. It makes your theorizing and testing obnoxiously more complicated, but if the facts strongly suggest that there really is a regress, you have to accept that and deal with it.

So in our hypothetical case, we'd suddenly take seriously ideas like ancient astronauts, or hyperdimensional beings who set up our universe but left clues behind, etc.

(And it would be scientific, not "just philosophy" or "metaphysical speculation.")

Even an infinite regress isn't necessarily a deal-killer. Suppose, for example, that we found that our universe was the inside of a black hole that formed in another universe, which was likewise the inside of another black hole...

We don't know for sure that such a regress is impossible, or even that it's theoretically a bad thing. It's just not clear that we should expect there to be a starting point, rather than an infinite sequence of past stuff (or more generally, metaphysically prior stuff) that goes back forever.

Our intuitions about causality have to break down somewhere. Normal causation can't explain why the cosmos would just pop into existence ex nihilo (from nothign), or why it would always have been here.

Asking for a cause of everything seems superficially like asking for the cause of a thing, but it's really not. Everything else we explain in terms of causation (or some more general metaphysical priority) is always explaining something in terms of something else that caused it. (Or constitutes it.)

When it comes to explaining everything, you just can't get an answer like that---at that level, there's no other thing to cause or constitute the thing you're trying to explain.

It seems like a good question---it always seems right to ask "Why X?"---but it can't possibly be answered in the usual way we intuitively expect. There must be something wrong with the question.

Asking for the cause of something, which presupposes something else to cause it, can't be right if the thing in question is everything.

It's a bad question, with a false presupposition, like asking "which way is up?" in deep space. There simply is no up out there, and can't be. Our assumption that the question "which way is up?" is always a valid question is simply wrong.

Likewise asking for the cause of everything is not a valid question. Once you're that far out---outside of everything---there's no causes there. Apparently, it just is, and is neither caused by something else, nor self-causing.

(Some theologians will tell you it's self-causing, and that's God, who's the First Cause. Bunk. That doesn't help, because it does just as much violence to our intuitions about causation, and whatever "self-causing" thing there is, we have to just accept that it nonetheless just is that way, in terms of anything we understand as causation. And of course, there's no reason to identify such a bizarre thing with any "God" concept that's useful for theology, and very good reasons not to.)

One of the things that annoys me about religion is that it's systematically phenomenally bad at this sort of thing. It's always coming up with bogus answers to badly-formed questions, which just hide the real deep questions and wrap them up in something that's anthropomorphic, or at least hokily anthropocentric in some basic way.

Sastra, I really like the oasis analogy.

Leigh, of the cognitive dissonance:

Perhaps my choice of words was infelicitous; I thought it obvious I was using "construct" in its verb meaning, "To create (an argument or a sentence, for example) by systematically arranging ideas or terms" and in its noun meaning, "A concept, model, or schematic idea."

What, did you imagine me molding a golden calf with laser eyes, or some such?

No, I didn't imagine it as such. I don't consider a "golden calf" to be equivalent to a "god". I'm familiar with the phrase "mental construct", which I also do not consider equivalent to a "god". I don't think your mistake was an infelicitous remark, but a result of unclear thinking.

Pardon me for stating the obvious, but mental constructs do not create the universe, nor are they in any other way supernatural or divine. You're conflating or equivocating two very different things: (1) the existence of a "God", and (2) the existence of your concept of a "God".

You wrote that you create this concept for yourself. You wrote about how Jesus "represents [your] aspiration" and is "a way to reinforce [your] desire". You've made a religion of your own mind, and do this by confusing (1) with (2). Which of those are you referring to when you say "God is very real . . . to me"?

Although I didn't feel like responding to this before, I'll do so now:

Because you fear that it might, just might, cause me (me personally) to make bad choices in my life? What business is that of yours?

I don't care in the slightest about your Personal Beliefs™, which you claim are so benign and ethical. You made them public here, so in this limited domain it is "my business" and I am free to comment on them. If some belief makes you feel better, or helps you to be a more ethical person, I don't think there's any reason to object to it -- knock yourself out. Atheists can be "good without God", so unless you're implying otherwise, it doesn't matter to me which stories you use to motivate yourself. It also has nothing to do with the truth of the belief itself....

When you make a choice (good or bad), it affects the world. However much it may comfort you to believe it, you are not in some kind of causal vacuum where your Personal Choices™ are untouchable and unassailable. So please don't whine about how I "demand" that you give up your religion. I would like you to seriously question whether they're in any way true or meaningful. If they're so important to you, then their truth should also be important.

I found both Bryan and Paul's posts helpful. They both make points I agree with.

Bryan: "I would further assert that it is useful to think of the following argument as also containing a logical fallacy:

1. We have an incomplete understanding of X.
2. This provides any justification whatsoever for my belief that there is an invisible dragon in my garage."

Yes. This is still a logical fallacy. If Ken Miller believes that the fact of quantum indeterminacy alone provides justification for his God, he is committing a logical fallacy. We need evidence of the sort that can be independently tested. I think it is an open question what Ken believes on this point.

Perhaps someone should ask Ken Miller?

Paul: "Our intuitions about causality have to break down somewhere. Normal causation can't explain why the cosmos would just pop into existence ex nihilo (from nothing), or why it would always have been here."

Yes. Because this is true, we need to have some epistemic humility when attempting to apply logic to every possible situation, including claims based on personal experience. "Reason's last step is the recognition that there are an infinite number of things which are beyond it."

Mr. T (#188) makes a good point that theists who bring up their beliefs have certainly put them on the table for discussion. It strikes me as completely reasonable for any member of the 'A(theist)-Team' to put the hammer down here. He makes good points there.

Having said that, consider Mr. T's conclusion in post #188: "I would like you to seriously question whether they're in any way true or meaningful. If they're so important to you, then their truth should also be important."

See, to me this implies that since Leigh Williams is not approaching her belief system from Mr. T's preferred point of attack, that she has not seriously questioned whether they're in any way true or meaningful. I'm not sure that there's any way Mr. T could actually know that. A different set of assumptions up front could lead to a very different process of self-examination, and, perhaps, a different outcome.

In particular, some of us here clearly believe that there are no limits to the application of reason. Some of us take a more modest view, that there may be limits, yet still see the desirability of employing reason as broadly and deeply as possible.

By Scott Hatfield, OM (not verified) on 06 Mar 2010 #permalink

Scott, #189:

See, to me this implies that since Leigh Williams is not approaching her belief system from Mr. T's preferred point of attack, that she has not seriously questioned whether they're in any way true or meaningful. I'm not sure that there's any way Mr. T could actually know that. A different set of assumptions up front could lead to a very different process of self-examination, and, perhaps, a different outcome.

I'm only basing that on what I've read here. I don't know her definitely can't read her mind.

Leigh wrote at #167:

But if your point is that I shouldn't believe in God because you don't think he's real, I'm going to tell you to mind your own business and leave mine to me.

If she took it seriously and had good reasons for believing as she does, she could've given a response other than "mind your own business".

Sastra responded at #186:

I think our point is that you shouldn't believe in God because it's not real -- and your point seems to be that this doesn't -- and wouldn't -- matter to you.

This is how I interpret it as well: intellectual dishonesty. Given her pseudonym, she appears to wear her cognitive dissonance as a badge of honor.

Leigh again, back at #159:

I have no desire -- not to mention, no ability -- to convince you that s/he is real, and in fact would consider that to be very disrespectful. I'm content to let you have your personal life ordered as you like it.

If she does not want to and is incapable of convincing me, then she is not taking it seriously. She may forfeit the argument; but then you can't step in as referee and claim that she may still have won on some unstated and unsubstantiated technicality.

If "God" to her means some kind of personal life choice, rather than an actual being that actually exists with actual properties, then I am at a loss for words. I do not think it means what she thinks it means. There is little indication in her servings of word salad that she takes seriously the possibility that her beliefs may be false.

Otherwise, if she is talking about an actual being who actually exists, it is a claim about the natural or supernatural world, and can be evaluated as such. No amount of handwaving or backpedaling about the limits of reason, epistemic humility or other ways of knowing will change that.

Scott Hatfield:

Our intuitions about causality have to break down somewhere. Normal causation can't explain why the cosmos would just pop into existence ex nihilo (from nothing), or why it would always have been here.

Yes. Because this is true, we need to have some epistemic humility when attempting to apply logic to every possible situation, including claims based on personal experience. "Reason's last step is the recognition that there are an infinite number of things which are beyond it.

You have a funny idea of epistemic humility---funny strange, and even funny ha ha.

The fact that certain ultimate and anomalous questions are not answerable in the ways we intuitively expect---and may not be answerable at all---does not mean that logic doesn't apply.

It certainly doesn't mean that illogic is going to give us a good answer. Religion is never going to give us an answer to the question of which way is up in deep space. There isn't an answer to that question, because it's a bad question with false presuppositions.

Likewise, if religion claims to explain why there's something rather than nothing at all, you can bet the ranch it's going to get it wrong.

The concept of causation breaks down at ultimate causes, but that doesn't mean logic does; it's precisely logic that shows us that our naive intuitions about causation are wrong, when applied erronously to everything, as though it were just a thing, to be caused by or reduced to another distinct thing.

So far as we know, logic doesn't break down anywhere in describing reality. It just doesn't give us all the answers we might hope for. And when it doesn't, that doesn't license pulling any old story out of your ass or an old book and saying it's a reasonable thing to believe. We have plenty of reason to think that's an excellent way of getting it wrong.

Miller's especially bad about this with regard to quantum physics. The "contradictions" in quantum physics are not contradictions that reveal the limitations of logic, and license illogical religious inferences. They are just contradictions between our naive prescientific intuitions about particles vs. waves, determinism, and things like that. In science, we don't accept such contradictions---not accepting them is what drives science to look for deeper, non-contradictory explanations of superficially "contradictory" phenomena.

The fact that rationality can't answer absolutely all questions doesn't mean religion can answer those questions, or answer any questions correctly. And it sure doesn't mean there are no constraints on what counts as a reasonable belief vs. a popular delusion.

If we are allowed in our "epistemic humility" to believe anything we want, without rational warrant for it, that's not epistemic humility. That is precisely massive hubris. (If not a kind of paranoid lunacy.)

Miller does the same kind of thing that some Catholic honchos said that Galileo should do--he adds a few relativizing axioms to the scientific view, to make his "religious" views observationally indistinguishable from an atheistic scientific view.

If that's a good move, then Galileo could have done it too. He could have said that he thinks the Sun goes around the Earth, in just such a way that it looks just like it would if the Earth went around the Sun.

You seem to be dodging the significance of that analogy.

Would Galileo have been reasonable to do that, and maybe to say that whether the Earth goes around the Sun is just a "metaphysical" question of "philosophy," and then defer to scripture about what's true, as he was pressured to do?

Or was he right to resist that? Is the question a scientific one, about a truth that science has in fact revealed? (Perhaps provisionally, but also pretty darned convincingly? Doesn't it convince you?)

Hasn't science revealed that the Earth does in fact orbit the Sun, as surely as we know just about anything?

Please answer that question. Seriously. Do not dodge it.

Isn't it a bad, unscientific and even antiscientific move to explain away scientific facts with contrived unfalsifiable weirdness like that, in order to shield points of theology from falsification?

Please answer that question, too.

Isn't Miller's apologetics doing the same basic thing---contorting both science and theology to protect theological claims from scientific refutation?

If that's valid---even as "apologetics"---what wouldn't be valid?

Wouldn't it be equally valid to say that the Sun goes around the Earth, and it only looks like the opposite is going on, and that scientists ought to have the "epistemic humility" to avoid strong statements like the earth goes around the sun?

And even if somebody says that as a point of theology, in the course of doing something they label "apologetics" or "metaphysics," rather than "science" isn't it clearly antiscientific?

If that doesn't count as antiscientific, what on Earth would, and why?

Leigh Williams #159 wrote:

I have no desire -- not to mention, no ability -- to convince you that s/he is real, and in fact would consider that to be very disrespectful. I'm content to let you have your personal life ordered as you like it.

Why does this question break down into personal lives? Imagine we are talking about whether vitalism exists, or horizontal gene transfer, or whether sheep live in Scotland. I won't argue against your views on those matters when I think you're wrong, because I'm content to let you have your personal life ordered as you like it.

I think that if we wish to "respect others" and divest ourselves of ego and self-seeking, then we need to take ourselves out of the question. Both sides. Dropping the issue -- or refusing to take it up -- out of consideration for the other person doesn't seem like respect to me: it's the kind of forbearance you would give to the infirm, or weak. It's an expectation that they can't take themselves out of the issue, and just look at it for truth, because it's really about them.

Paul W. #191 wrote:

If that's a good move, then Galileo could have done it too. He could have said that he thinks the Sun goes around the Earth, in just such a way that it looks just like it would if the Earth went around the Sun.

This got me thinking. During the Victorian era, there was an attempt to reconcile science and religion with the claim that the earth was really only 6,000 years old, but God had made it look exactly as if it were quite a bit older. This way, you got all your science, and all your religion. All the instruments and measurements are quite right and correct -- but, under the circumstances, inadequate. No conflict.

I'm not sure that's exactly what Miller is doing here -- the guy who came up with the 'theory' (did it start with an "o"?) didn't get into trying to figure out some scientifically respectable place where God couldn't be ruled out from working this miracle -- but there's some resemblance, in that a world made by God, will look exactly like one that wasn't made by God.

Paul: Thanks for your reply. I think you are reading a bit too much into my 'funny' comments. I'm not endorsing Miller's "argument" regarding quantum indeterminacy.

As presented in 'Finding Darwin's God', it's offered as a thought experiment about how an immanent God could interact with creation without leaving any detectable evidence. Like many readers of Miller's book, I found that bit anticlimactic and terribly unsatisfying.

My point has been to protest that the charges of 'logical fallacy' and 'intellectual dishonesty' are overstated. As I read the original argument as presented in 'Finding Darwin's God', I do not find that it rises to the level of a fallacy nor do I detect signs of intellectual dishonesty. The Boston Phoenix piece, however, understandably exercises our suspicions in this area due to its author's decision to deliberately exclude a balanced treatment of Miller's views by Prof. Myers and Coyne.

That's disappointing, of course. Perhaps, as one commenter at Dr. Coyne's site suggested, that author (David Scharfenberg) is angling for a Templeton Fellowship. You might be interested to learn that Scharfenberg has written a few public pieces about the need to (get this) to subsidize a dying breed, journalists such as himself. Who knows? Perhaps he's taking his own advice to the next level!

Anyway, with respect to your bolded questions...I mean, obviously, we can treat the heliocentric view of the solar system as something so well-established that, in the words of S.J. Gould it would be perverse to treat otherwise. You seem to feel that this is such a great rhetorical strategem that you are concerned I might dodge it. Well, I gave the answer that any sensible person would. Are you surprised? How does this impel any conclusion regarding Miller's alleged scientific misconduct?

As to the next question..."Isn't it a bad, unscientific and even antiscientific move to explain away scientific facts with contrived unfalsifiable weirdness like that, in order to shield points of theology from falsification?"

Well, sure. But, what are the scientific facts that Miller is allegedly explaining away with quantum indeterminacy? As I read it, he's not explaining away the quantum itself, which he treats as a real natural phenomena. He's using it as an example of the limits of our knowledge, and how his version of God might be operating behind the scenes. I mean, it may seem contrived to you, and it's certainly unfalsifiable. No argument there. But the theology in question is already shielded. It doesn't need Miller's thought experiment to be non-falsifiable.

Along the same vein, you ask: "Isn't Miller's apologetics doing the same basic thing---contorting both science and theology to protect theological claims from scientific refutation?"

This strikes me as muddled. I'm not sure that a few isolated comments by a well-known scientist that strike a religion-friendly tone rise to the level of apologetics, which as I understand it is a branch of Christian theology. If I perceived his work largely in such a light, it wouldn't hold much use for me and I would join you in condemning its' misapplication. But, as I've written previously, I'm not persuaded that Miller's argument as presented in 'Finding Darwin's God' is evidence of any bad faith on his part.

Again, what science is being contorted here? Is Miller's description of quantum indeterminacy inaccurate? And what are the supposed theological claims that he is 'shielding' from scientific refutation? Paul, if they are theological claims, chances are they can't be refuted by scientific investigation and nothing you or I or Ken Miller has to say on that point is going to change that fact, one way or the other.

Since that is manifestly the case, what do partisans here hope to gain by questioning Drl Miller's motives? Doesn't it make more sense to point out the logical reasons why we might find his argument with respect to QM unsatisfactory, and leave it at that?

By Scott Hatfield, OM (not verified) on 06 Mar 2010 #permalink

Scott @194,

Doesn't it make more sense to point out the logical reasons why we might find his argument with respect to QM unsatisfactory, and leave it at that?

Only if you feel that the basis on which it's unsatisfactory (that is, it's an argument intended to reconcile the Christian deity and its miracles with science by excluding such from testability) is of no significance.

"A Scientist's Search for Common Ground Between God and Evolution" is a significantly different subtitle to 'A Person's Search for Common Ground Between God and Evolution', with (presumably) intended connotations.

By John Morales (not verified) on 06 Mar 2010 #permalink

Mr.T writes:

"If she does not want to and is incapable of convincing me, then she is not taking it seriously. She may forfeit the argument; but then you can't step in as referee and claim that she may still have won on some unstated and unsubstantiated technicality."

Fair enough, though I didn't see my comment as attempting to establish some loophole for Leigh. And, presumably, she's a big girl and can speak for herself on this matter.

I just think that you folk are kind of talking past each other. For example, when you write:

"If she does not want to and is incapable of convincing me, then she is not taking it seriously..."

I have to ask: how do you know that? If LW is unable to provide you with the kind of answer you seek, does that really mean that she hasn't thought seriously about her own beliefs?

That strikes me as as a presumptuous and unnecessary stance. After all, when dealing with LW or any other disputant, you are already within your rights to say this:

"Since you are incapable of addressing my questions in a way that's meaningful to me, I feel no obligation to take you seriously."

I think the difference matters. One can reject arguments on their own merits, including the lack of evidence given in support of this or that claim. There is no need, as many people have done on this thread, to without evidence presume bad faith or lack of seriousness on the part of people like LW or Ken Miller. After all, we're not talking about people who have made a habit of bashing science, pushing religion or personally trashing their ideological foes. Quite the opposite, in fact.

Life is short, and I have other commitments. Those who want to continue to joust with me over this can leave comments here.

By Scott Hatfield, OM (not verified) on 07 Mar 2010 #permalink

Scott Hatfield,
First, I do not impugn Miller's bona fides. I do however think the he is taking a fundamentally unscientific if not anti-scientific attitude in trying to explain an unknown (how divine will might manifest itself) in terms of an unknowable (indeterminacy at the quantum level).

What is more, I think that his idea of how quantum indeterminacy works is fundamentally flawed. Isn't he in effect introducing God as a hidden variable in quantum mechanics? Now granted, God could be a "global" hidden variable rather than a local hidden variable, but I still think it would be extremely difficult to reproduce the results of quantum mechanics if such tampering exists at the quantum mechanical level (ala Bell's inequalities). It seems to me that a deity would have to fudge the results only when nobody was looking--and I would think such a deceitful god would be unacceptable for theological reasons.

Frankly, I find Leigh's pink fluffy God a little easier to stomach as it is a subjective conception and so not jamming its thumbs into the celestial clockwork all the time. Ultimately, I think Kierkegaard's belief as an act of will is the only way to justify a belief in God. Does it make you a better person. This is purely subjective. However, based on my survey of Christendom, I would have to pronounce in the negative for the vast majority of Xtians.

By a_ray_in_dilbe… (not verified) on 07 Mar 2010 #permalink

From what I can tell, and my understanding may be completely off, Miller is suggesting a butterfly effect god. That is, one who creates hurricanes by having a butterfly flap its wings. No butterfly can flap its wings hard enough to regrow amputees' limbs, so amputees' prayers go unanswered.

But this puts a limit on a supposedly omnipotent being. This isn't a "microwave a burrito so hot he couldn't eat it" paradox. Rather it's a specific limit that says on the microscopic (nanoscopic? picoscopic?) level god can fit into the QM indeterminacy gaps but can't intervene in the natural world at a macroscopic level. Hence no new limbs for amputees.

By 'Tis Himself, OM (not verified) on 07 Mar 2010 #permalink

a_ray_in_dilbert_space #198 wrote:

Ultimately, I think Kierkegaard's belief as an act of will is the only way to justify a belief in God. Does it make you a better person. This is purely subjective.

Ah, but that's just the problem: from within the framework of fideism, this epistemically unjustified "belief as an act of will" is an objective virtue, just in itself. This virtue is not necessarily connected to the behavior it later inspires, treating each other well. You could do that, anyway.

The merit implied in Having Faith (and being the kind of person who Has Faith) is presumably how and why God created a world where amputees do not heal when you pray for them, and God hides itself in quantum indeterminacy. If God's existence were as unambiguous and obvious as the existence of trees, then believing that God exists would show no merit. Instead, this is how God can tell those who are sensitive and accepting of a loving, good God apart from those who are not. The will to believe -- the choice to believe -- is THE litmus test, for the most important test there can be.

Can you remain loyal to God, by believing it exists? Or will you betray your highest calling, chicken out, abandon your best nature, and think only of yourself and Lower Things?

Not that there is anything wrong with that. Really.

Right -- that so works. I do think this is bound to spill into a folk rule-of-thumb on how people can tell those who are trustworthy and meritous, from those who aren't -- even with liberal sophisticates. And that's what we see. What's wrong with atheists? From the standpoint of the Millerian Moderate Middle, it's not their reasoning, or their conclusion. It comes down to {{{{LOVE}}}}.

Who wouldn't rather just be wrong?

I suspect that the only way to avoid this condemnation, is to do as the Good Leigh is apparently doing: belief in God doesn't matter, because God doesn't care if you believe in Him, because God doesn't matter. I think it's going to be hard to walk the tightrope between that viewpoint, and God being the mostest importantest and impressivest thing ever in the whole wide world and beyond, amen.

I've never read Kirkegaard, but I'm afraid I just can't take seriously someone whose works include "Fear and Trembling" and "The Sickness Unto Death". sounds like he fitted Bertie Wooster's description of Schopenhauer: "A grouch of the most pronounced description".

Then of course there was Monty Python's wonderful "Piranha Brothers" sketch:

"Well one day I was at home threatening the kids when I looks out through the hole in the wall and sees this tank pull up and out gets one of Dinsdale's boys, so he comes in nice and friendly and says Dinsdale wants to have a word with me, so he chains me to the back of the tank and takes me for a scrape round to Dinsdale's place and Dinsdale's there in the conversation pit with Doug and Charles Paisley, the baby crusher, and two film producers and a man they called 'Kierkegaard', who just sat there biting the heads off whippets and Dinsdale says 'I hear you've been a naughty boy Clement' and he splits me nostrils open and saws me leg off and pulls me liver out and I tell him my name's not Clement and then... he loses his temper and nails me head to the floor."

By Knockgoats (not verified) on 07 Mar 2010 #permalink

Ok... Glad I kept this thread open in my browser. There is no greater laugh I have had this week than Leigh Williams positing that his personal version of God is, while directed at the real world, basically indistinguishable from my being a follower of Brell Serilis, and thus, in that context, angry at how badly Kobolds and other Serilian races are treated in Everquest 2, while playing that game... My only question has got to be.. If you are going to construct one, why construct one that a) lends credence to some of the most egregiously stupid people on the planet at the moment, and b) which already has a lot of silly mythology behind it, which automatically muddies the waters, when talking to anyone about it? A desire to be "with the in crowd"? A fear of making one up that will be laughed at, because everyone will know its made up? Complete disinterest that, without getting you to admit the construct, everyone will assume its actually the same god as the local church's? I don't get it. As silly as it is, in as much as it was created to mock other religions, why would something like the FSM not make more sense? Because you can't pin it on some, highly probably, mythological figure, who was also "invented" to promote a new religion? You couldn't find some other ancient god to pick, who might not have been carrying around as much baggage?

This is just... very odd. Not in the least since, personally, I can't imagine indulging in such fantasy "in the real world", the same why I do in at least one semi-Roleplay setting, one "real" Roleplay setting, and not at all in a third (where my corporation actually takes the active stance that, more or less, "the only real immortality comes from a jump clone vat, the only soul the data in the networks, so its the data, and the technology we must protect from the religious idiots, who might destroy it"). I don't know.. Maybe I have always just been too down to earth, so have never made up "imaginary friends", which I didn't a) know where imaginary, b) know where not real, and c) knew where myself, talking to myself, so shouldn't be attributed values, such as, "Yesterday I talked to god, or Mark Twain, or the guy down the street that died last week, because I didn't have anyone better to imagine talking to." Just... to me, quite bizarre.

J., a friend of my wife, accompanied her elderly aunt to Church a week ago (she never goes otherwise, AFAIK), and the preacher said at one point something like:
"It's not easy believing in God. You have to work at it."
J., who is somewhat wooish, was impressed; my wife, who isn't, not so much. I'm sure the preacher was right, for anyone intelligent in a society where disbelief is an option - but it would surely take just as much work to believe in leprechauns; and to believe in both God and leprechauns, even more. so is that more virtuous?

By Knockgoats (not verified) on 07 Mar 2010 #permalink

Knockgoats and Sastra,

Kierkegaard is an interesting read--particularly "Fear and Trembling," which is a meditation on the story of Abraham and Isaac. In it, ol' Soren views the faith of Abraham--faith that would move him to kill his own son--as a terrible thing (terrible in the sense of invoking terror). He doesn't seem to pass judgment on whether such a faith is a good thing, per se, merely that it is powerful. And for the time, Kierkegaard's unapologetic rejection of apologetics is refreshing

Whether we admit it or not, I think that there are some things we have to take on faith. As a personal example, consider the fate of the human race as it faces threats of climate change, overpopulation, resource depletion, etc. Those of us who are trying to get humans to see reason have to at times ask ourselves if it's worth it. In the climate wars, I have faced personal harassment and even threats.

At times I have to wonder if the extinction of the human species is really a bad thing. And since I have no children, I have no personal stake in the continued existence of humanity or human civilization once I am dead. And yet, I am human. Both my training and my nature as a social animal would make it unethical to try and avert what will undoubtedly be great suffering if we proceed on our current course.

Humanity has certainly given me no reason to be optimistic about its survival in the long term, nor to be enthusiastic that such survival would even be a good thing. I don't see a choice other than to take it as an article of faith that perhaps humanity is worth working to save, whether this is possible or not.

I'd be perfectly happy if you could provide some other rationale, as faith is contrary to my nature. I have not, however, been able to get beyond it in this case.

By a_ray_in_dilbe… (not verified) on 07 Mar 2010 #permalink

Knockgoats #203 wrote:

"It's not easy believing in God. You have to work at it."

A few years back I ran across a video (can't find it any more) which appeared to be from a British Candid-Camera style television show, involving two young men setting up a booth outside the Scientology headquarters in a city somewhere. They were giving 'tests' to random passer-bys, and asking a progressive series of Scientology-related questions on what they might find it possible to believe. "Do you think you would be able to believe that your body is inhabited by dead aliens?" and so forth.

As I recall, what impressed me most about the clip wasn't the ridiculousness of the Scientology theology, but the attitude of the apparently unsuspecting ordinary folks who were answering. They seemed to approach the "could you believe?" question the same way they might have approached progressive questions on how much weight they could lift, or how long they could hold their breath under water, if they tried.

Well, yes, I could believe that. Oh, that would be harder, but I could try. Hmmm... no, that one might be too much for me. Sorry.

They didn't seem to be assessing likelihood of facts. They looked like they were judging their own skills and strengths, with, apparently, the highest ability tracking with the more absurd delusions.

At least, that's how I remember it. I thought it rather illuminating. I wonder how much of that was simply provoked by the situation of the survey and its leading questions -- and how much was the result of their understanding that this was about religion. Or, perhaps, is that just the way people think about all their beliefs: they're not objective conclusions, they're about belonging to the right group, or showing the right character?

a_ray,
I don't think I get your point. As you say, if we proceed on our present course, there will undoubtedly (well, very likely, maybe Jesus or the Benevolent Aliens or Its Imminence The Singularity will come and safe us) be great suffering. Why do you need any justification for trying to avoid that? As for the more distant future, none of us can know what will happen, so there's no point worrying about it; we can see far enough to have a good idea what needs to be done.

By Knockgoats (not verified) on 07 Mar 2010 #permalink

Seriously? God's interaction is restricted to setting parameter values on quantum distributions at the beginning of time? Does He look like this? - Antiochus Epiphanes

More like this, I think.

By Knockgoats (not verified) on 07 Mar 2010 #permalink

Knockgoats,
Well, first, I won't be here when the caca really hits the fan. I have no children who will suffer.

And one could argue that humanity in aggregate has brought a future of great suffering and ultimately extinction upon itself. So, what distinguishes between the actions of someone fighting to try and get humanity to come to its senses and someone who merely decides to develop, say, a strain of vaccine resistant smallpox to bring about the end sooner and limit suffering to a few weeks rather than a couple of generations?

One could argue that the latter action would minimize damage to the ecosystem, but most of us would dismiss it as monstrous. How could this be unless we take it on faith--against mounting evidence--that the continuance of human civilization is a good thing?

By a_ray_in_dilbe… (not verified) on 07 Mar 2010 #permalink

Trying to catch up on this thread after jumping in with facetious remarks. Just read Paul W.'s #120 - typically incisive, and shows why the Molly was rightly awarded. Many congratulations Paul!

By Knockgoats (not verified) on 07 Mar 2010 #permalink

For the purposes of doing science, all faith-based, evidence-free positions should be discarded. - Scott Hatfield, OM

That sentence really is much better without the first clause.

By Knockgoats (not verified) on 07 Mar 2010 #permalink

How could this be unless we take it on faith--against mounting evidence--that the continuance of human civilization is a good thing? - a_ray_in_dilbert_space

Respect for others' preferences.

By Knockgoats (not verified) on 07 Mar 2010 #permalink

We need SC,OM's standard link to the argumnet that it is a moral duty to adjust your beliefs to the evidence - that it is actually unethical to have faith. I need to go to bed - anyone remember the link?

By Knockgoats (not verified) on 07 Mar 2010 #permalink

Damn, SC, that's one big ass thread.

By 'Tis Himself, OM (not verified) on 07 Mar 2010 #permalink

One could argue that the latter action would minimize damage to the ecosystem

Here you seem to take it on faith that the continuing existence of the ecosystem is a good thing? But all those other species will one day go extinct anyway, and the ecosystem will eventually be destroyed too. So why not just nuke the whole planet now to end the suffering?

(In case any Intersectionites are reading this: I'm not being serious. Let's not nuke the planet mmkay)

ARIDS: "How could this be unless we take it on faith--against mounting evidence--that the continuance of human civilization is a good thing?"

Knockgoats: "Respect for others' preferences."

Well, it's the unrealistic preferences of others that is jeopardizing the continued viability of human civilization, is it not? So, I still have the problem of why I should devote my blood, sweat and tears toward preservation of human civilization, when 1)humans are bringing this fate on themselves; 2)humans are doing untold harm to global ecology; 3)the likelihood of success is remote; and 4)I derive no personal benefit and experience significant sacrifice in doing so. This is not something that I think can be justified rationally.

By a_ray_in_dilbe… (not verified) on 07 Mar 2010 #permalink

ARIDS,

[...] 4)I derive no personal benefit and experience significant sacrifice in doing so. This is not something that I think can be justified rationally.

No personal benefit? Significant sacrifice?

Really.

Why can't you, say, advocate for good policy whilst maintaining a hedonistic lifestyle?

By John Morales (not verified) on 07 Mar 2010 #permalink

A similar argument was brought up with respect to wages in the last negotiation with the company I work at (several others where in similar negotiation). The result of such brain dead thinking was a) the company whose union folded because of it now has 100% of their employees, under management level, working minimum wage, b) our union was able to keep existing wages, but the company 1) extended the time needed to gain increases, and 2) declared certain employees as "intended to be temp positions, therefor they will never receive a wage increase, unless the fed/state increases the minimum wage.

Basically, the, "I have mine, and won't live long enough to see the consequence.", mentality screwed **everyone** at one company out of having enough to live off of, and rendered the other an impossible place to seek work (we have probably 1% turnover at the store, per year, not including seasonal jobs, so... there is ***no*** such thing as "moving on to another department or pay scale"). Logic would dictate that those of us that *do* have a stake in the matter do away with the people that believe they don't, if you get right down to "justifying things rationally". Just saying...

Oh, and its also the same illogic applied to right wingers/libertarians for cutting their own taxes, cutting funding of social programs, then insisting that, since they don't use X, Y or Z service, nor plan to, nor expect to, they shouldn't have to pay into them. It is *not* a logical position at all, especially since the same logic, had it been applied *before* you where born, would, potentially, have you living in places that are toxic, eating toxic foods, taking toxic "buy at your own risk" medicines, and many other things, which individuals, groups, corporations, and even other governments, looking for a place to dump their waste, have done in the past. Such short sighted thinking leads people to make short sighted choices, and all you have to do is ask someone that lived near one of the rivers in Europe that ***caught fracking fire***, what they think of, "Why should I worry about what won't effect me?"

Because, you cannot **rationally** say that you are 100% certain that something that won't get done, because you and others don't feel they have a stake in it at all, won't directly effect you. In fact, you have to be blind to not see "some" things, like water standards, where it would be, if someone else hadn't figured that how much arsenic and other things where in it, **mattered** to more than the guy able to afford to live by a well, and have his own tests run.

Rational? Not by any standard most people apply, or have, since they started building cities, instead of guarding their private spring, while living in mud huts, using primitive spears.

humans are doing untold harm to global ecology

There's that assumption again. What's your rational justification for the preservation of the global ecology?

"Suppose that it was common knowledge that if you ... prayed deeply, all of a sudden, your limb would grow back, ... What would that do to moral independence?"

Yup, I can just imagine - if that really were the case - deeply religious people all over the world thinking it was just awful.

One wonders, then: why they do routinely engage in petitionary prayer at all, if it would be so terrible should God answer it?

By paulmurray (not verified) on 07 Mar 2010 #permalink

One wonders, then: why they do routinely engage in petitionary prayer at all, if it would be so terrible should God answer it?

No, no. It wouldn't be terrible if God answered their particular prayer. It would be terrible if God answered all prayers by all righteous people because—— Um.

Hm.

Well, it would mean that being righteous -- believing in and loving God and loving your fellow humans -- would have clear empirical benefits. So more people would have an incentive to be righteous. There would be no more religious wars, or even religious disagreements -- prayer would provide all the answers. Those who do bad things would know that they are in danger of eternal punishment, and would have an incentive to repent.

There might be those who think they can get away with whatever they want while young, and then repent on their deathbeds. But presumably a real God would see the cynical and selfish hypocrisy in this attitude, and reject their insincerity. So people would know that starting off with a sincere desire to be better people is the only way to actually be accepted by God.

And all in all, it would therefore mean that people would have less reason to be mean to each other, and more reason to be nicer, and all in all, more people would end up in heaven instead of being in eternal agony in hell.

But this would do ... something to "moral independence".

OK, Ken -- why would everyone being nicer to each other, and fewer people in hell, and more people in heaven, be a Bad Thing?

By Owlmirror (not verified) on 07 Mar 2010 #permalink

PZ said:

Nowhere anywhere in the article is any reasonable support for the notion of a god, nor especially of any peculiarly Catholic deity. Of course there isn't, because he doesn't have any.

It's funny that you think he cannot have any reasonable support for the notion of God when - in reality - your position (that there is no God) is the unreasonable one.

You (I assume) would like to think that everything is reducible to matter. But how does that explain the existence of frogs, rocks and trees? What are the specific properties of matter that determine whether this bit of matter becomes a frog, that bit of matter becomes a rock, and another bit becomes a tree?

That these things cannot be explained by appeal to matter alone is sufficient to necessitate that there exists something outside of matter that determines what matter does.

That "something" we call God.

By Daniel Smith (not verified) on 08 Mar 2010 #permalink

It's funny that you think he cannot have any reasonable support for the notion of God when - in reality - your position (that there is no God) is the unreasonable one.

Only for those that reason backwards.

You (I assume) would like to think that everything is reducible to matter. But how does that explain the existence of frogs, rocks and trees?

What does that even mean? Are you trying to say that frogs, rocks and trees are not made of matter?

What are the specific properties of matter that determine whether this bit of matter becomes a frog, that bit of matter becomes a rock, and another bit becomes a tree?

Chemistry and physics.

That these things cannot be explained by appeal to matter alone is sufficient to necessitate that there exists something outside of matter that determines what matter does.

The physical properties of matter determine what matter does. They are only "outside" of matter if you insist that matter in and of itself has no properties, which is a self-contradiction, and is unreasonable.

That "something" we call God.

God is the physical properties of matter? You worship electrical charges and the geometry of molecular bonds?

Are you even capable of trying to make sense?

By Owlmirror (not verified) on 08 Mar 2010 #permalink

That "something" we call God

You're a deist?

By WowbaggerOM (not verified) on 08 Mar 2010 #permalink

Daniel Smith, are you going to respond?

By John Morales (not verified) on 08 Mar 2010 #permalink

The physical properties of matter determine what matter does.

So the physical properties of the matter in me are different from the physical properties of the matter that makes up the rest of the Earth?

How so?

By Daniel Smith (not verified) on 09 Mar 2010 #permalink

Daniel Smith:

The physical properties of matter determine what matter does.

So the physical properties of the matter in me are different from the physical properties of the matter that makes up the rest of the Earth?

How so?

Pretty much the same way that the physical properties of silicon in the CPU of the computer you're using are different from the physical properties of silicon in sand on the beach, or the glass in your front window.

That is, they're not different---silicon is silicon---except that they're organized very differently.

You're made of the same stuff as lots of other things that aren't alive. How things are put together is what makes them really interesting.

Your comment #222 comes across as staggeringly ignorant on a science blog, especially a biology blog.

Vitalism is long dead.

Daniel says, "That these things cannot be explained by appeal to matter alone is sufficient to necessitate that there exists something outside of matter that determines what matter does."

Um, actually, we can. In fact, a lot of the properties of the variety of matter we call "life" can be explained just by looking at the properties of the carbon atom. Now add hydrogen to carbon, and you get a whole new type of chemical bonding that explains more. Oxygen and Nitrogen--still more.

More and more, Daniel, we know how genes turn on and off, how enzymes turn our food into energy, how our thoughts form and propagate into movement and on and on.

And YOU, unfortunately are missing it all because you insist on explaining everything with "GODDIDIT". Sad.

By a_ray_in_dilbe… (not verified) on 09 Mar 2010 #permalink

Daniel Smith #222 wrote:

What are the specific properties of matter that determine whether this bit of matter becomes a frog, that bit of matter becomes a rock, and another bit becomes a tree?

Matter in motion, forming different levels of patterns due to interactions.

I don't know if this line of attack is going to go towards vitalism and teleology, just reduce to the Fallacy of Composition/Division, or some combination.

I'm not familiar with the Argument from Frogs.

I'm not familiar with the Argument from Frogs.

I love you, Sastra.

I love you too, RickR. ;)

I'm not familiar with the Argument from Frogs.

Don't worry, it's about the same as the argument from Walruses :

1. a walrus is different from a king or a cabbage or a shoe.
2. therefore God exists

By negentropyeater (not verified) on 09 Mar 2010 #permalink

That is, they're not different---silicon is silicon---except that they're organized very differently.

Exactly! There is nothing in the properties of matter itself that explains that organization. Silicon is silicon, carbon is carbon, hydrogen is hydrogen, etc. What causes some carbon to become people and other carbon to become dirt? It can't be the properties of carbon - because all carbon is the same.

So what is it?

By Daniel Smith (not verified) on 09 Mar 2010 #permalink

Daniel Smith #233 wrote:

So what is it?

Chance and necessity.

I'm not sure how far down you want to go. Physics, chemistry, interactions ... You seem to be asking a science question, but it's very vague.

Is this going to end up with Platonic forms? Or "X is the way it is, because someone wanted it to be that way, and not some other way." Willpower is not a fundamental cosmic force.

OK, Daniel, since we can take some silicon and turn it into a water glass and some silicon and turn it into a microprocessor, does that mean humans are god?

By a_ray_in_dilbe… (not verified) on 09 Mar 2010 #permalink

Exactly! There is nothing in the properties of matter itself that explains that organization.

You're missing the point. Patterns of organization, flows of energy through matter, have properties also. You seem to be appealing to the fallacy of composition.

Exactly! There is nothing in the properties of matter itself that explains that organization. Silicon is silicon, carbon is carbon, hydrogen is hydrogen, etc. What causes some carbon to become people and other carbon to become dirt? It can't be the properties of carbon - because all carbon is the same.

good grief.

How about environmental interaction? Location? Proximity to other elements? Chemistry? etc..

By Rev. BigDumbChimp (not verified) on 09 Mar 2010 #permalink

What causes some carbon to become people and other carbon to become dirt?

How are humans able to consume the grains and fruits of plants that grow in dirt?

God is in the digestive system.

By Janine, Mistre… (not verified) on 09 Mar 2010 #permalink

Janine: "God is in the digestive system."

God is full of shit?

By a_ray_in_dilbe… (not verified) on 09 Mar 2010 #permalink

My gut tells me so!

By Janine, Mistre… (not verified) on 09 Mar 2010 #permalink

What causes some carbon to become people and other carbon to become dirt? It can't be the properties of carbon - because all carbon is the same.

So what is it?

Because Carbon disposes of 4 electrons and is capable of forming multiple stable covalent bonds with other small atoms including itself : when it gets in contact with other atoms it can form, under specific thermodynamic conditions thanks to the attraction of the electromagnetic force between electron and nuclei, more than ten million known different chemical compounds. And those same compounds eventually end up making humans and frogs through the known biological evolutionary processes, or dirt, rocks, methane and millions of other things.
It's not magic, but science.

By negentropyeater (not verified) on 09 Mar 2010 #permalink

Exactly! There is nothing in the properties of matter itself that explains that organization.

There's nothing intrinsic to matter per se that directly explains any particular organization per se---only the capacity to be organized in a variety of ways, if the conditions are right.

In that sense, looking at the properties of matter doesn't tell you why, for example, you might have a river, or an eroded hillside, much less life and such.

For that, you have to look at the history of how it came to be that way, e.g., with gravity causing water to flow downhill, with many consequences.

No magic there. There's no "extra" thing that explains organization, and none is needed.

Complex things emerge from the interactions of simpler things, by boring mechanical regularities.

Pretty simple stuff will result in life and intelligence, in a very tiny fraction of places in the universe, given simple things interacting in simple ways for a very, very long time. Most places, the conditions aren't right for that, and you don't get such complex patterns emerging.

It's not magic, or imposed from outside, or difficult to explain in a general way.

I'm curious what you're doing here. Are you trying to convince us that there's some magical shaping force that turns matter into life, or something like that? If so, we can recommend a few of books that will disabuse you in a hurry.

Or are you really just asking sincere questions, because you know no science? In that case, too, those books would be informative.

What causes some carbon to become people and other carbon to become dirt?

Actually, carbon doesn't just become dirt. Before it was dirt it was a person, or a plant, or a deer. Or perhaps it was in the air and got stuck in the dirt. It is all the same carbon. Well, it's really not, actually. Mostly it's 12C, but there are others. But that's further than it's worth it do discuss. Suffice to say, the carbon atom has some properties that make it very chemically useful in nature, and in labs.

Chance and necessity.
How about environmental interaction? Location? Proximity to other elements? Chemistry? etc..

None of this explains why nature is anything other than randomness and chaos.

Instead big chunks of nature are observed to reliably tend towards certain ends as if guided to do so.

Is this going to end up with Platonic forms?

Plato was right (about that anyway.)

Willpower is not a fundamental cosmic force.

No? The bulk of nature is observed to reliably tend towards certain ends. Science has documented this repeatedly (indeed science would be impossible if it weren't so.) How can intentionality, purpose, goal-directedness, or "willpower" be ruled out? Especially when the alternative explains nothing!

since we can take some silicon and turn it into a water glass and some silicon and turn it into a microprocessor, does that mean humans are god?

It shows how intentionality, purpose, goal-directedness, and willpower affects matter.

By Daniel Smith (not verified) on 09 Mar 2010 #permalink

Oh for fuck's sake. Seriously? You can't WILL change.

By https://me.yah… (not verified) on 09 Mar 2010 #permalink

Methinks that Daniel, my brother, has had a serious break with reality.

So is it true that like attracts like?

By Janine, Mistre… (not verified) on 09 Mar 2010 #permalink

Instead big chunks of nature are observed to reliably tend towards certain ends as if guided to do so.

Is 1+1 "guided" to be 2?

When a rock falls, is it "guided" towards the ground?

When a water flows, is it "guided" towards the lowest point it can reach?

When a paper clip is pulled to a magnet, is it being "guided"?

The bulk of nature is observed to reliably tend towards certain ends. Science has documented this repeatedly (indeed science would be impossible if it weren't so.)

The universe behaves in a consistent fashion. Why should more be inferred from this than the consistent behavior of the universe being a reasonable scientific axiom?

How can intentionality, purpose, goal-directedness, or "willpower" be ruled out?

If everything in the universe is the result of "intentionality, purpose, goal-directedness, or "willpower"", how would you know that anything wasn't the result of this intention?

If rocks regularly falling to the earth is "intentional", then so is a baby falling from a 10th-floor deck and smashing their little head in, right?

More to the point, the very words "randomness" and "chaos" would be meaningless. You wouldn't be able to logically or coherently distinguish between "randomness and chaos" and "being guided".

It shows how intentionality, purpose, goal-directedness, and willpower affects matter.

Sigh. Are you familiar with fulgerite?

By Owlmirror (not verified) on 09 Mar 2010 #permalink

Would it be fair to summaries this as the 'stuff is; therefore, God' argument?

By WowbaggerOM (not verified) on 09 Mar 2010 #permalink

Are you familiar with fulgerite?

Or obsidian, for that matter?

====

Would it be fair to summaries this as the 'stuff is; therefore, God' argument?

It certainly looks like a modified form of generally absolutist presuppositional apologetics.

1: Can you account for the universe behaving in a consistent way?
2: Ha, no you cannot.
3: Therefore, God exists.

By Owlmirror (not verified) on 09 Mar 2010 #permalink

It certainly looks like a modified form of generally absolutist presuppositional apologetics.

Ah, I thought I could smell something.

By WowbaggerOM (not verified) on 09 Mar 2010 #permalink

Tell you what, Daniel. Let's do some experiments. You use your GODDIDIT theory and I'll use physics and chemistry and let's see which theory has the most predictive power. I'll even let you ask God how each one is going to work out. How 'bout it?

By a_ray_in_dilbe… (not verified) on 09 Mar 2010 #permalink

You can't WILL change
Sure you can. You just "willed" that your fingers change positions in order to type that reply.

Your will affected matter.

Wonderful i'nit!

By Daniel Smith (not verified) on 09 Mar 2010 #permalink

None of this explains why nature is anything other than randomness and chaos.

Why should we expect nature to be random and chaotic? We have no grounds at all to expect this to be so. Order is naturally emergent, stability much more stable than instability. We shouldn't expect to see a random chaotic universe for two reasons: firstly that we should see structure because structure is more stable than non-structure, and secondly that if there were no order then we wouldn't be here.

This entire line of argument boils down to "we exist, therefore Jesus died on the cross for our sins". It's nonsense apologetics, and one that is philosophically and scientifically vapid.

Get the fuck out of here. I never expected to met an actual vitalist.

By Antiochus Epiphanes (not verified) on 09 Mar 2010 #permalink

You just "willed" that your fingers change positions in order to type that reply.

Now you're just being disingenuous. Or silly.

Either way, you're not advancing your argument.

By Owlmirror (not verified) on 09 Mar 2010 #permalink

If everything in the universe is the result of "intentionality, purpose, goal-directedness, or "willpower"", how would you know that anything wasn't the result of this intention?

I never said "everything in the universe is the result of intentionality, purpose, goal-directedness, or willpower", nor do I believe that.

The second part of your question is interesting however as it brings up the issue of "the good". If we can discern intentionality in nature, then we can infer that a standard exists by which to objectively judge the level of intentionality in a given object. That standard is "the standard of goodness". We use this objective standard everyday. A doctor might tell you that your back is "bad", or that your heart is "good". This presuppose a standard of excellence for hearts and backs against which we judge the individual instances. Note that this is not a subjective opinion but rather an objective comparison based on observed traits. We can rationally determine what a "good heart" or a "good back" is based on our observations of the design and function of hearts and backs.

So it is not that "everything is the product of intentionality", but rather that nature reveals an intentionality and standard of form for many things against which we can objectively weigh their adherence and determine their level of "goodness".

By Daniel Smith (not verified) on 09 Mar 2010 #permalink

More to the point, the very words "randomness" and "chaos" would be meaningless. You wouldn't be able to logically or coherently distinguish between "randomness and chaos" and "being guided".

The fact that we can logically distinguish between these things tells us something too - namely that both exist, and nature definitely leans heavily towards the "being guided" side.

By Daniel Smith (not verified) on 09 Mar 2010 #permalink

Why should we expect nature to be random and chaotic?

Why shouldn't we?

We have no grounds at all to expect this to be so.

We don't?

Order is naturally emergent

It is? Why?

stability much more stable than instability.

That goes without saying.

We shouldn't expect to see a random chaotic universe for two reasons: firstly that we should see structure because structure is more stable than non-structure,

Why does matter care about stability and structure?

and secondly that if there were no order then we wouldn't be here.

There you go! Case closed I guess.

By Daniel Smith (not verified) on 09 Mar 2010 #permalink

That standard is "the standard of goodness". We use this objective standard everyday. A doctor might tell you that your back is "bad", or that your heart is "good". This presuppose a standard of excellence for hearts and backs against which we judge the individual instances.

Oh, Jeezusfuckingchrist - are you so imbecilic you don't understand colloquialisms and metaphors? You are the embodiment of the category error argument.

I never said "everything in the universe is the result of intentionality, purpose, goal-directedness, or willpower", nor do I believe that.

Ah, so I misunderstood your essentialism as being universal.

Very well, then.

So it is not that "everything is the product of intentionality", but rather that nature reveals an intentionality and standard of form for many things against which we can objectively weigh their adherence and determine their level of "goodness".

So salmonella, cholera, the Plasmodium and Trichinella parasites, and many other disease microbes and parasites, which excel at making people sick, often even to the point of death, also reveal such intentionality and standard of form?

If not, why not?

=========

The fact that we can logically distinguish between these things tells us something too - namely that both exist, and nature definitely leans heavily towards the "being guided" side.

This is a false inference, or non-sequitur.

=========

Why should we expect nature to be random and chaotic?Why shouldn't we?

Because the universe certainly appears to operate by generally consistent rules. Since those rules are not themselves the result of "intentionality" -- as you agree above -- the resulting behavior would not be random and chaotic, but would follow those generally consistent rules.

Order is naturally emergentIt is? Why?

Because of those generally consistent rules that everything follows.

and secondly that if there were no order then we wouldn't be here.There you go! Case closed I guess.

Did you not agree that not everything is the result of intentionality, purpose, goal-directedness, or willpower? It therefore follows that not all order must be the result of intentionality, purpose, goal-directedness, or willpower.

QED.

By Owlmirror (not verified) on 09 Mar 2010 #permalink

Your will affected matter.

Funny, if that was the case I would be out of a job. You could just think your new chemical compound and it would appear by magic. Care to cite this happening?

By Nerd of Redhead, OM (not verified) on 09 Mar 2010 #permalink

Daniel,
Nature's intentionality towards us: It's trying to kill us--viruses, bacteria, toxins, accidents, fire...all trying to turn us into worm food. The End

By a_ray_in_dilbe… (not verified) on 09 Mar 2010 #permalink

There you go! Case closed I guess./blockquote>Yep, you're a religious fuckwit. All blather and sophistry, no hard or scientific evidence.

By Nerd of Redhead, OM (not verified) on 09 Mar 2010 #permalink

Why shouldn't we?

This was explained already.

We don't?

Nope, we have no grounds presuppositionally to expect there to be order without guidance, and from observation we can see that order and patterns are naturally emergent from process.

It is?

Yes.

Why?

Why should there be a why? It is how it is, evidentially it is.

That goes without saying.

So if we live in a universe where stability is possible (we do), then of course we should expect to see structure because we are looking at stable structures as opposed to unstable ones.

Why does matter care about stability and structure?

It doesn't. You're mistaking process with intentionality.

There you go! Case closed I guess.

Not case closed. Just that your line of argument doesn't get us anywhere.

I wonder if Daniel is familiar with teleonomy (as discussed with the Ken Wilber fan earlier¹, back in the Thread).

--

¹ Who introduced it to me.

By John Morales (not verified) on 09 Mar 2010 #permalink

Why should there be a why?

Just to add a little to this. To ask why is to anthropomorphise reality. To even ask "why?" requires the firing of millions of neurons, ordered through millions of years of evolution. And the need to ask such questions is vital when considering the motivations and actions of oneself and others. Yet it makes no sense to ask "why does it rain?" for the very same reason. When people ask "why does it rain?", in actuality they are asking "how does rain come about?" And one can explain rain purely in the form of the process that comes about.

Yet there are those who engage in promiscuous teleology - they would argue that there is rain for watering plants. Yet rain doesn't exist to water plants, rain just happens to be a natural emergent property of atoms and plants have evolved to take advantage of such a thing. At no stage is a teleological explanation needed, yet our minds are wired to expect one. In other words, as much as we would like to think there is rain to water plants - all we are doing is putting our cognitive bias on the universe.

Man, I'm getting a contact high by just reading Daniel Smith's post.

By Feynmaniac (not verified) on 09 Mar 2010 #permalink

Ah, now we've moved on to 'I can ask an endless succession of why questions; therefore, God exists' approach.

Can we call this the interrogative ad infinitum fallacy?

By WowbaggerOM (not verified) on 09 Mar 2010 #permalink

It seems to me Daniel is using the fine tuning argument.

By 'Tis Himself, OM (not verified) on 09 Mar 2010 #permalink

You (I assume) would like to think that everything is reducible to matter. But how does that explain the existence of frogs, rocks and trees?

Because you're forgetting about process. How matter is organised by process determines how it will go. Rocks form through particular processes involving pressure or cooling. Trees and frogs come from reproduction where the order is an expression of genetic sequences - stemming from some 3.5 billion years of evolution.

What are the specific properties of matter that determine whether this bit of matter becomes a frog, that bit of matter becomes a rock, and another bit becomes a tree?

You're asking the wrong question. Frogs, rocks and trees are not inherent in matter where the matter goes one way or another. The matter stays exactly the same - a carbon atom is a carbon atom no matter what it's in. Rather it's the process by which the atoms are arranged. Your body is continually taking on new atoms (well new to the structure of you) and discarding old ones, cells dying and being recycled. You are made of atoms, but it's not the atoms that make you. For you, it's the fact that your mum and your dad had sex and the genes that got passed to you expressed itself in your form.

That's a pretty important point to take in. You are here because your parents had sex. They didn't build your structure, they merely provided the genetic instructions. They may have known that having sex would lead to offspring, they may have intended that. But your form was not determined by their intentions, but the fact that you exist was. Think about that for a moment, your existence is not dependent on the process by which you exist to know anything about how to make you. Now dogs may or may not know they are having sex, yet they still have dog offspring. Frogs probably don't know that they're even having sex, yet they still have frog offspring. Trees do not know anything about their reproductive process, yet still produce tree offspring.

It's important to understand evolution in order to understand life. Life doesn't make any sense without evolution; the process of replication means that form is inevitable. Once you understand that, you understand why looking for intentionality makes no sense - and that form is emergent from process. If you can grasp that, then perhaps you can move away from promiscuous teleology and towards an understanding of nature. You're seeing the world as a child sees it, it's time to put away childish notions and become a man.

Because you're forgetting about process. How matter is organised by process determines how it will go.

But process is not reducible to matter. Matter does not determine process, otherwise all matter of a certain type would initiate the same process. No, it's the other way around - process determines the organization of matter.

So you're left with something immaterial organizing matter and no material explanation for the various organizations we see.

By Daniel Smith (not verified) on 10 Mar 2010 #permalink

Daniel Smith:

But process is not reducible to matter. Matter does not determine process, otherwise all matter of a certain type would initiate the same process. No, it's the other way around - process determines the organization of matter.

Daniel, you evidently know essentially zero science---no physics, no biology, etc.---or are being disingenuous, or something.

You should at least understand two very basic things about the scientific worldview.

1. Higher-level entities and events (such as animals and life) are made out of lower-level entities and events (such as molecules and chemical reactions). This doesn't require magic or guidance.

2. Minor random small-scale differences in initial states of things can, under some circumstances, cascade into quite major differences in higher-level and larger-scale things. (E.g., a butterfly flapping its wings, or not, might rarely end up determining whether a hurricane forms and destroys a city, and a single utterly random quantum event might, very rarely, determine whether the butterfly flaps its wings and all that happens.) There is no magic or guidance required for that to happen.

3. All the interesting high-level phenomena that we observe, e.g., galaxies, stars, planets, life, ecosystems, minds, beliefs, emotions, social structures, religion, etc. appear to be the result of that sort of thing, without any magic or guidance required.

If you don't understand these things, you do not understand the scientific worldview that you seem to be criticizing.

You also keep making the same dumbass assertions about the requirements for guidance, something extra being needed besides "matter," etc., which are well known in light of science to be false.

Given that we're mostly scientists and scientifically-inclined people, you are just being an annoying troll demonstrating your utter ignorance of the most basic and important facts, or perhaps your sheer trollery, framing things with evidently false presuppositions so that you can make false high-level claims that may seem to the utterly ignorant to make some kind of sense.

Stop it. Learn some science. Read a couple of reasonable, popular books like Stuart Kauffman's At Home in the Universe and Dawkins's The Blind Watchmaker so that you get some clue of what the fuck you're talking about, and don't just keep spouting patent falsehoods. Then get back to us.

Or if you have already read similar books, and understood them, then act like it. Stop making high-level assumptions that most scientists think are patently false, and begging the questions relevant to your preferred conclusions.

If you've actually read such books, and not understood them, then start asking real, sincere questions rather than "gotcha" questions with presuppositions we know to be false.

If you've read such books, and been unable to understand them, then come way, way down off your high horse, and try to convey what it is you didn't get, such that if somebody cares enough, they might be willing to explain that aspect of the scientific worldview to you.

If you can't do those things, please go the fuck away.

You seem to have a case of Dunning-Kruger syndrome, or a penchant for dishonesty, or some of each. (If you don't know what Dunning-Kruger is, google it, and seriously consider that we think it applies to you.)

So please, before expecting anybody to answer your silly bogus "questions," tell us a few things:

1. What's your real point? Is it that God must exist? That seems obvious, but I thought I'd check.

2. Have you ever read a book on evolution by natural selection written by a scientist or philosopher who believes it's true?

3. If not, why should we think you're actually interested in the subject, or intellectually honest enough to be interested in what our views actually are, before you come telling us we're wrong?

I suggest that if Daniel won't answer these questions, everybody should stop feeding the troll, right now, and if he persists in trolling, he should be banned.

By Paul W., OM (not verified) on 10 Mar 2010 #permalink

1. Higher-level entities and events (such as animals and life) are made out of lower-level entities and events (such as molecules and chemical reactions). This doesn't require magic or guidance.

I'm not disputing what things are made of. A car is made of simpler parts as well. It DOES require guidance to put a car together though. So, it has nothing to do with the makeup of the disputed object. It has EVERYTHING to do with the organization. You still cannot explain, by appeals to material things, how matter comes to be consistently organized for the functions it is known to perform.

By Daniel Smith (not verified) on 10 Mar 2010 #permalink

You still cannot explain, by appeals to material things, how matter comes to be consistently organized for the functions it is known to perform.

Actually, we can: the process of genetic change and inheritance, reproduction, and differential survival, is exactly that explanation. You have not shown how that explanation is incorrect; you have merely asserted that it is.

I note that you still haven't responded to my question @#260.

Are the organisms consistently organized for the functions that are known to kill millions of humans the result of some "immaterial" thing?

By Owlmirror (not verified) on 10 Mar 2010 #permalink

Daniel,

Are you following the Taliban science curriculum where they say Hydrogen + Oxygen doesn't become Water unless you say "by the grace of god"?

Here I thought electronegativities did the trick just fine.

By a_ray_in_dilbe… (not verified) on 10 Mar 2010 #permalink

Matter does not determine process, otherwise all matter of a certain type would initiate the same process.

It does. What do you think a physical or chemical reaction is, if not that "same process" being initated by the material components involved?

No, it's the other way around - process determines the organization of matter.

No, you're still doing it wrong. "Matter" and "process" are inextricably intertwined; each is capable of causing a feedback loop into the other. That feedback loop may well terminate for simpler "processes" (chemical reactions), but so far, the "process" of the biological evolution of life has been continuing for a long time.

So you're left with something immaterial organizing matter and no material explanation for the various organizations we see.

Chemical reactions are only "immaterial" if you insist that the physical properties of atoms that undergo the reactions are not material, which is incoherent.

By Owlmirror (not verified) on 10 Mar 2010 #permalink

The watchmaker argument is so boring.

By Rev. BigDumbChimp (not verified) on 10 Mar 2010 #permalink

It DOES require guidance to put a car together though.

Wrong. It requires a set of instructions to be followed by the machinery. In the case of life, the set of instructions and machinery evolved without the need for an imaginary "designer", and is present in the DNA throughout the biosphere. Your analogies fail, and you fail in the necessity to show physical evidence for your "designer" outside of the "design".

By Nerd of Redhead, OM (not verified) on 10 Mar 2010 #permalink

Kel @270

As Sagan put it: "the beauty of a living thing is not the atoms that go into it, but those atoms are put together."

Daniel,

So you're left with something immaterial organizing matter and no material explanation for the various organizations we see.

and again

You still cannot explain, by appeals to material things, how matter comes to be consistently organized for the functions it is known to perform.

Like, say, a snowflake? :)

By John Morales (not verified) on 10 Mar 2010 #permalink

But how does matter know it's supposed to make a snowflake, instead of something else? And why that particular one, instead of another one?

If you draw targets around each snowflake, you will realize that every little flake is a direct center hit. Someone, or something, aimed.

Besides, can you hold a process in your hand? No?

Materialism, refuted.

William Paley gave the most famous presentation of the watchmaker argument in 1802. Unfortunately for Paley, David Hume refuted the argument in a book published in 1748. That's right, Daniel, Paley's argument was answered 52 years before it was made.

The watchmaker argument only works if everything ordered is ordered by design. However, we know there are systems in nature that are ordered but not designed. For example, salt in water will form crystals as the water evaporates. Further, if we assume that order can only come by design, and design only by the order of intellect, then if we posit that God designed it, then he is ordered, and thus requires a designer as well.

By 'Tis Himself, OM (not verified) on 10 Mar 2010 #permalink

Daniel:

I'm not disputing what things are made of. A car is made of simpler parts as well. It DOES require guidance to put a car together though. So, it has nothing to do with the makeup of the disputed object. It has EVERYTHING to do with the organization. You still cannot explain, by appeals to material things, how matter comes to be consistently organized for the functions it is known to perform.

Ah. I see. You're dishonest and rude.

To justify that inference, I remind you of 2 of my 3 questions, which I suggested that you answer before proceeding as you have been---and which you have conspicuously not answered:

2. Have you ever read a book on evolution by natural selection written by a scientist or philosopher who believes it's true?

A assume that the answer to this is no. Do kindly correct me if I'm wrong, and tell me which book.

3. If not, why should we think you're actually interested in the subject, or intellectually honest enough to be interested in what our views actually are, before you come telling us we're wrong?

I assume that you don't have a good answer to this, and you're just here to spout woefully ill-informed or dishonest apologetics, and don't care that it's rude to waste the time of people who are actual scientists, or have put considerable study into the subject.

Do look up Dunning-Kruger.

By Paul W., OM (not verified) on 10 Mar 2010 #permalink

You still cannot explain, by appeals to material things, how matter comes to be consistently organized for the functions it is known to perform.

Maybe if you reformulated it this way:

explain, by appeals to material thingsphysical laws, how matter comes to be consistently organized for the functions it is known to perform.

1. start learning about the time dependent Schrödinger equation for a simple atomic system
2. move up to condensed matter physics
3. move up to statistical mechanics
4. move up to inorganic and organic chemistry
5. move up to biology

then you'll understand how physical laws ewplain how matter comes to be consistently organised for the functions it is known to perform.

Everything can be derived from those God damned solutions to the Schrödinger equation.

By negentropyeater (not verified) on 10 Mar 2010 #permalink

Why do you people think I'm talking about, or disputing, evolution?
(I'm not)

Why do you think I'm using Paley's argument?
(I'm not)

I'm attempting to use (rather feebly I must admit) Aquinas' teleological argument. His five proofs of God are the basis for my position.

By Daniel Smith (not verified) on 10 Mar 2010 #permalink

Daniel, philosophy isn't going where you want it to. I believe Aquinas' arguments are long dead. Try scientific proof. Starting with conclusive physical evidence for your imaginary deity, say an eternally burning bush. Something that can only exist by a supranatural explanation. If it isn't there, you have nothing of interest for us.

By Nerd of Redhead, OM (not verified) on 10 Mar 2010 #permalink

Daniel:

Why do you people think I'm talking about, or disputing, evolution?

Because you appear not to understand it, e.g., that it is not teleological, and that its significance is largely that it explains the appearance of teleology while showing that many inferences of teleology are wrong.

Why do you think I'm using Paley's argument?

Because it sounds like you are, and you haven't made it clear what you're arguing for, if not that. You ask seemingly rhetorical questions that seem to imply that, and you refuse to come out and say what the fuck you're talking about.

It makes you sound like a dishonest asshole. If you're not one, start spelling out what you're saying, clearly, rather than asking "questions" that seem to rhetorically imply your preferred answers while embodying false presuppositions.

I'm attempting to use (rather feebly I must admit) Aquinas' teleological argument. His five proofs of God are the basis for my position.

Holy crap. Kant and Hume disposed of the Five Ways about two centuries ago. Have you got something new to add?

If not, at least pick an argument and spell it out.

The progress of science over the last several hundred years has largely been a matter of debunking teleological reasoning, with evolution by natural selection and emergent complexity (a la chaos theory and complexity theory) completely obviating any need for teleological explanation of natural phenomena, including life, minds, and religion.

If you don't understand that, you don't understand the scientific world view.

If you don't understand the scientific worldview, then stop acting like you think you do.

Absolutely do not treat the Five Ways as something you can insinuate the truth of, since they're all generally agreed to be bogus by the preponderance of philosophers, scientists, and especially philosophers of science.

And seriously, I hate to be rude, but...

ANSWER THE FUCKING QUESTIONS, ASSHOLE.

Have you ever read even one book on evolution by a scientist or philosopher who thinks its true?

If you can actually understand evolution and Aquinas---which I doubt, from the way you talk---then you should easily be able to understand The Blind Watchmaker, and I'd recommend that you read Dennett's Darwin's Dangerous Idea.

If not, why should we think you're actually interested in the subject, or intellectually honest enough to be interested in what our views actually are, before you come telling us we're wrong?

By Paul W., OM (not verified) on 10 Mar 2010 #permalink

Aquinas' fifth proof and Paley's watchmaker analogy all boil down to the same : because life and other things are complex, they must have been designed.

Science has done a bit of progress since Aquinas and Paley, the teleological argument is long dead.

By negentropyeater (not verified) on 10 Mar 2010 #permalink

Daniel Smith #285 wrote:

I'm attempting to use (rather feebly I must admit) Aquinas' teleological argument. His five proofs of God are the basis for my position.

Aquinas' 5 Ways are all basically versions of the Cosmological Argument: First Mover, a first efficient cause, a necessary being, a cause of perfection, and a director of natural things to their ends. There's not just a single thing wrong with them -- there are many things wrong with them.

If nothing else, they're scientifically naive. They rest on an armchair Aristotlean folk understanding of physics and motion that was already in trouble back with the Greeks.

And that last one -- which is what you seem to be pushing in particular -- begs the question, by assuming that everything has a natural "end" to fulfill.

Sure, a Thomist can accept evolution. A Thomist can accept large swathes of modern cosmology too. But he or she then has to abandon their science and shift the way they think about reality, and return to pre-scientific intuitions. It won't work.

But process is not reducible to matter. Matter does not determine process, otherwise all matter of a certain type would initiate the same process.

Do you think that electromegnetic force is some external force to the matter? No, it's part of matter itself. The forces that act externally on any given matter are forces that are inherent in the matter interacting with the forces inherent in other matter.

No, it's the other way around - process determines the organization of matter.

You're reflecting the very point I made, then acting as if it is against what I said.

To go back to your rock, think of how a rock is formed. It's material acting on material, which in turn gives a material outcome. Rock formation is not an immaterial process, it's purely the interaction of the material. You're incredibly naive about how the forces of physics works. Surely you should be looking at studying some elementary physics before continuing this line of argument.

So you're left with something immaterial organizing matter and no material explanation for the various organizations we see.

Nonsense, the forces that organise matter are material. Just go read up on how electromagnetic force works.

Why do you think I'm using Paley's argument?

Because it sounds just like the same argument.

By Rev. BigDumbChimp (not verified) on 10 Mar 2010 #permalink

The teleological argument falls apart because life is not comparable to human design. Consider the difference between how to get a watch and how to get a baby.

If we want a watch, we go to a watchmaker (or seller of a watchmaker's product). Watches are manufactured devices, their components artificially and purposefully put in place. If you want a baby, you get two people to have sex. The parents while they may choose to make a baby, don't have a say on what features the baby has.

The fundamental problem with the watchmaker argument is that life is not purposefully ordered by those manufacturing it. As pointed out in a previous post, you don't need a parent to know how to make a heart or brain - you just need them to have sex. They may or may not be aware that sex will lead to a baby, or that they may or may not be aware that they are having sex - but that's all that's needed.

Life is a reproductive process, life begets life, and structure comes from the evolutionary process. This is the main problem of the teleological argument - the similarities are artificial, yet the differences are fundamental. Cars, watches, houses - these are artificially designed and built. Life is not, life reproduces itself - and thus form is emergent from the process.

Why do you people think I'm talking about, or disputing, evolution?
(I'm not)

It's true that you've been terribly vague, and not particularly coherent, but your vagueness and incoherency certainly has not excluded disputing evolution.

And when you gave examples, you offered biological structures like the heart and the back, which means that you are necessarily discussing evolved organs, and trying to say something about them -- it's not at all clear exactly what, though, if you claim to not be disputing evolution.

I'm attempting to use (rather feebly I must admit) Aquinas' teleological argument.

Why do you think you can use an antiquated argument that embodies philosophical confusion about everything, and not end up disputing everything involving empirical science, which of course necessarily includes evolution?

1: Some thing are better than other things for some given purpose.
2: We can know or discover what those things are and classify them in accordance to their functionality
3: Therefore, God exists.

By Owlmirror (not verified) on 10 Mar 2010 #permalink

His five proofs of God are the basis for my position.

Oh wow, seriously?

Perhaps you can stop pretending that you're making a priori arguments and embrace the a posteriori nature of the arguments you are making and learn some science. Otherwise you're putting yourself into a completely untenable position and thinking that you're somehow vindicated in doing so.

Evolution is not the issue. Please stop trying to make this about evolution, complexity, or any of the other ID buzzwords. I'm not in that camp (at least not fully). My arguments are more basic and apply to all of matter - not just life and complexity.

The issue for me is whether or not the organization of matter which we observe can be reducible to material causes. We know that things that are material (matter), material-dependent (energy) and immaterial (time, space, mathematics, etc.) exist. You all seem to think these things are sufficient to explain the organization of matter. I don't. You have yet to elucidate a reason why the properties of matter favor the organization we observe. Your reasoning seems to be that since we can point to forces at work in nature, those forces must be what organizes nature. It's akin to pointing at the forces at work in a car and concluding that those forces organized the car (and no, my position has nothing to do with complexity. It could just as easily be a table.)

As for the other four ways, there is no denying that the science of the day was not as advanced as the science of today, that does not however negate the metaphysical arguments made.

First way: The argument from motion (or change). Things are observe to move (or change). That movement or change is caused by another. There cannot be an infinite regress of movers (else there cannot be a first mover to start the whole thing). Therefore there must be an unmoved mover. The only weakness I can identify with this argument is the "movement or change is caused by another" part. It could be said that animals move themselves, so this negates that. But are animals really moving themselves? Or is one part moving another? (Which puts the argument back on track.)

Second way: The argument from (efficient) causality. Nothing can be the efficient cause of itself. ("Efficient cause" being an immediate cause as opposed to an historical cause, IOW that which is causing something to exist right now.) There cannot be an infinite regress of efficient causes (for the same reasons cited above). Therefore there must be a first efficient cause. I find no weakness in this argument.

Third way: The argument from contingency. There are things which are observed to be contingent. Nothing contingent could exist forever (by definition). If everything were contingent, then it is possible that at some time in the past nothing existed - and that would mean that nothing could exist now - which is an absurdity. So there must be something that exists that is not contingent but necessary. For the same reasons as seen in the first two ways, there cannot be an infinite regress of necessary objects so there must be something that has its own necessity. This one loses me a bit when he talks about necessary objects, but the reasoning is still sound as far as I can see.

Fourth way: The argument from degree. That there are observed to be degrees of qualities in beings. Some beings are more or less intelligent, good, true, etc. That there are gradations in these qualities points to there being a maximum intelligence, goodness, trueness, etc. That this argument is about beings at first threw me, especially because he cites the archaic belief of his day that fire was the cause of all hotness. But it is evident from the context that he is not talking about things but about beings.

Many of you claim that these arguments have been refuted. Please tell me (if you can) how a particular argument has been refuted.

By Daniel Smith (not verified) on 10 Mar 2010 #permalink

and immaterial (time, space, mathematics, etc.) exist.

If you think time and space are immaterial, then you are wrong. As for mathematics, well that's debatable. It's more an abstract that can be applied on the universe.

Many of you claim that these arguments have been refuted. Please tell me (if you can) how a particular argument has been refuted.

Okay

First way: The argument from motion (or change).

This argument is founded on an a posteriori observation, and thus suffers from the problem of induction. Even if all observed particles have prior causes, you can't say that all particles have prior causes. Besides, get into the world of quantum mechanics and that clockwork view of the universe falls apart.

Second way: The argument from (efficient) causality.

This is pretty much the same as the first argument. So just to expand, ask yourself "from whence came God?" If God can just exist, then why can't the universe? If God is operating in the universe, then why isn't God subject to the same causality? In other words, you have no grounds for making any assumptions about the nature of the universe or the nature of God.

I'll let otherse deal with the rest. Though I've got to say, even if those arguments are valid (they aren't), it doesn't mean what theists take it to mean. There was a prime mover, therefore that prime mover is omnipotent, omniscient, omnipresent, omnibenevolent, interacts in the affairs of the human race, died for our sins, judges people at the pearly gates, etc. It's completely unjustified to take this diffuse nebulous notion and call it God and pretend that you've logically proven Christianity true. Doesn't work that way.

Daniel Smith,

I'm not overly familiar with the standard responses to your points - though I have no doubt they exist and will be cited before too long - but I have read what you've posted and this one stands out:

That there are gradations in these qualities points to there being a maximum intelligence, goodness, trueness, etc.

Why? You're applying a teleology to things without anything to back it up beyond the bare assertion. The idea of perfection itself is very much a subjective human concept rather than any kind of undeniable fact.

By WowbaggerOM (not verified) on 10 Mar 2010 #permalink

The issue for me is whether or not the organization of matter which we observe can be reducible to material causes

That's what science has done. End of story.

that does not however negate the metaphysical arguments made.

Yes it does. Science has progressed. The sophistry behind the argument hasn't.

Many of you claim that these arguments have been refuted.

Science. Read some. End of story.

Philosophy without evidence is sophistry. Try finding the evidence (read science) for your claims. Then get back to us.

By Nerd of Redhead, OM (not verified) on 10 Mar 2010 #permalink

Daniel:

Evolution is not the issue. Please stop trying to make this about evolution, complexity, or any of the other ID buzzwords. I'm not in that camp (at least not fully). My arguments are more basic and apply to all of matter - not just life and complexity.

Your reply makes it pretty clear that you do not know what I am talking about. When I mentioned "complexity theory" (and "chaos theory") I was not talking about pseudoscientific ID stuff. I was talking about actual science that you don't appear to know the name of, much less the significance of.

I ask you again

HAVE YOU READ A BOOK ON EVOLUTION BY A SCIENTIST OR PHILOSOPHER WHO BELIEVES IN IT? And if so, which one?

That's a valid question, because you're coming in from left field and wasting our time trying to figure out what you're talking about, and what your background is.

So, for example, if you'd read The Blind Watchmaker, I would have some idea about certain ideas you've been exposed to, like cellular automata and Conway's Game of Life, and could refer to those in answering your other questions, or addressing your apparent misconceptions.

It's a legitimate question. Answer it, please.

I infer that you have not read any books that explain basic complexity theory, like Kauffman's or Prigogene's work. I won't even bother to ask whether that's correct, because I'm pretty sure it is, and I want you to answer the first question

HAVE YOU EVER READ A BOOK ON EVOLUTION BY A SCIENTIST OR PHILOSOPHER WHO BELIEVES IN IT?

By Paul W., OM (not verified) on 10 Mar 2010 #permalink

Asshole, you ARE making the watchmaker argument. It's not our fault that you're too stupid to realize this.

I refuted your "second way" back in post #282. I'll do it again and this time I'll type slowly so you can understand why your idiotic argument is a fucking idiotic argument.

You write:

The argument from (efficient) causality. Nothing can be the efficient cause of itself. ("Efficient cause" being an immediate cause as opposed to an historical cause, IOW that which is causing something to exist right now.) There cannot be an infinite regress of efficient causes (for the same reasons cited above). Therefore there must be a first efficient cause. I find no weakness in this argument.

First, let's get rid of the silly "there has to be a first efficient cause" stupidity. This is known in the rhetoric biz as special pleading. You want your watchmaker to be uncaused but you want everything else to be caused. Instead of claiming only one thing was uncaused, why can't we say ten things were uncaused, or a million things were uncaused or even everything but one thing were uncaused? You don't get a special pass for your watchmaker. You have to justify why one and only one thing is uncaused. You have completely and utterly failed to do so. Try again, only this time don't use a logical fallacy. (Hint: Folks have been trying to do this for centuries but failed to do so.)

Even the Big Bang cannot be shown to be uncaused. Physicists describe what happened at 10-43 seconds after the Big Bang but cannot describe what happened before then. If you try to squeeze your watchmaker into this tiny slice of time you're just using a god of the gaps argument. Sorry but we're not buying that shit either.

By 'Tis Himself, OM (not verified) on 10 Mar 2010 #permalink

Daniel:

We know that things that are material (matter), material-dependent (energy) and immaterial (time, space, mathematics, etc.) exist.

Your ontology appears muddled; matter and energy are different aspects of mass-energy and have physical existence, categorically different to abstract concepts which you classify as immaterial (e.g. mathematics) and have no physical existence.

You all seem to think these things are sufficient to explain the organization of matter. I don't.

Yes, you argue from incredulity. It's not that you don't concede there's a perfectly natural explanation for things, it's that you don't understand it. So, you invoke an extra (epistemically unwarranted) assumption, to justify your inchoate feeling that agency is responsible for reality.

You have yet to elucidate a reason why the properties of matter favor the organization we observe.

Because if they favoured otherwise, we'd observe otherwise; our conceptualisation of those properties is derived from empiricism.

Many of you claim that these arguments have been refuted. Please tell me (if you can) how a particular argument has been refuted.

Bah. I'll do one:

Third way: The argument from contingency. There are things which are observed to be contingent. [1] Nothing contingent could exist forever (by definition). [2] If everything were contingent, then it is possible that at some time in the past nothing existed - and that would mean that nothing could exist now - which is an absurdity. [3] So there must be something that exists that is not contingent but necessary. [4] For the same reasons as seen in the first two ways, there cannot be an infinite regress of necessary objects so there must be something that has its own necessity.

1. No. Contingent merely means possible but not necessary. It says nothing about duration of existence.
2. You're just asserting at least some things are not contingent¹; you only pretend to infer this from your erroneous [1], but it doesn't follow.
3. Again, you're pretending to infer when you're just restating your assertion at [2].
4. Why can't there be an infinite regress?

--

¹ And poorly at that.
(even if everything were contingent, then it could still be possible that at there was never a time in the past when nothing existed)

By John Morales (not verified) on 10 Mar 2010 #permalink

Daniel,
All of Acquinas's arguments have been shown faulty. So has Descartes. So has Augustine's. God doesn't have to exist for the world to be the way it is. Nor would his existence explain why the world is as we observe it.

Now none of this has any bearing on whether God ACTUALLY EXISTS. We don't know and probably can't know. So if you want to believe in God, great. Then you can go learn some actual science and say what a great architect he is. Or you can decide God doesn't exist and learn some science to figure out how the world actually works.

Either way, Daniel, learn some science. It will really enhance your appreciation of life, regardless of your theological orientation.

By a_ray_in_dilbe… (not verified) on 10 Mar 2010 #permalink

Please stop trying to make this about evolution, complexity, or any of the other ID buzzwords. I'm not in that camp (at least not fully). My arguments are more basic and apply to all of matter - not just life and complexity.

But it is the exact same argument. The fact you can't either see this or admit it is telling.

By Rev. BigDumbChimp (not verified) on 10 Mar 2010 #permalink

Wowbagger, it's a silly argument indeed.

That there are gradations in these qualities points to there being a maximum intelligence, goodness, trueness, etc.

"That there are gradations in these qualities points to there being a maximum doggyness, fidelity, tail-waggliness, etc."... therefore, there's an ultimate Dog, since we can conceive of one.

By John Morales (not verified) on 10 Mar 2010 #permalink

Asshole, you ARE making the watchmaker argument. It's not our fault that you're too stupid to realize this.

I disagree, he's making a teleological argument but not the specific argument that Paley made. And of course he's trying to make an a priori argument using (fallacious) a posteriori axioms, but that's the standard mistake of the presuppositionist.

He really should just read David Hume's Dialogues Concerning Natural Religion so when we say the arguments were shown to be fallacious some 250 years ago, he could see why.
there is an evident absurdity in pretending to demonstrate a matter of fact, or to prove it by any arguments a priori. Nothing is demonstrable, unless the contrary implies a contradiction. Nothing, that is distinctly conceivable, implies a contradiction. Whatever we conceive as existent, we can also conceive as non-existent. There is no being, therefore, whose non-existence implies a contradiction. Consequently there is no being, whose existence is demonstrable.

BTW, Daniel,

Do you realize that the consensus in philosophy is that none of Aquinas's "five ways" is a valid argument?

Do you realize that even most theologians in non-fundamentalist Christian sects don't think they're deductively valid? Many of them say that they're simply invalid, and offer zero evidence for the existence of God---that's why you need faith, which is supposedly a virtue. Some do think that they're suggestive and offer weak support for the existence of God, but are not deductively valid proofs.

BTW, please answer that question I've been asking.

By Paul W., OM (not verified) on 10 Mar 2010 #permalink

Daniel:

Aquinas's first three "ways" are all variants of what's called the Cosmological Argument, which is invalid for a couple of deep reasons.

The basic argument is that every normal thing must have a prior cause, and an infinite regress is impossible, so there must be a first cause that is a special kind of thing, which we call God.

One problem with this is that it's not clear that an infinite regress of causes is any weirder than a first, uncaused cause. It's not clear which we should find more believable.

There are variants of the Cosmological Argument that allow for that. They allow for an infinite regress of causes, but then say that such an infinite regress wouldn't exist without something special outside that infinite regress of normal things and normal causation, to enable it all to exist.

The real underlying question is this: Why is there something rather than nothing at all?

Once you realize that, you can start to see that there's something profoundly weird about the question, which suggests that there's a false presupposition hidden in there, and maybe it's not even a good question.

Part of the argument itself says that every thing has to have a cause distinct from, and in some sense prior to itself. (In the simple first cause argument, it's something that comes earlier in time, and causes the later thing. In other versions, it's some kind of "metaphysical priority"---I can explain that if necessary---but the structural problems are the same.)

The assumption of universal causatiion necessarily breaks down when you talk about literally everything. There is nothing outside of everything to cause it, or it wouldn't be everything.

Causal explanation is always explaining something in terms of something else. Therefore everything can't have a cause, because there's literally nothing else that could be its cause.

No matter how you slice it, you can't get a good answer to this question; a first cause simply makes no causal sense.

An infinite regress does make causal sense in that it explains each thing in terms of something else. (Which is why some philosophers and theologians prefer it.) Unfortunately, it doesn't answer the real underlying question any better than a first cause does---why is there something rather than nothing at all?

Either way, bringing God into it doesn't help a bit.

If God is a literal first cause, in time, it just raises the question of why there was God---which is just a specific version of the general question of why is there something rather than nothing at all---that is, why is there all this stuff, starting with God?

The real question is still there, untouched, and you've just pushed it back a step, not answered it. You're not any closer to understanding why there's anything at all, unless you've got a very good argument about why there's God rather than nothing at all.

(And nobody actually has that. The Ontological Argument is bogus too. The Cosmological Argument thus depends on something like the Ontological Argument---but if you have that, and can make it work, the Cosmological Argument is superfluous. Either way, the Cosmological Argument itself is useless---it's either simply wrong, or question-begging.)

If there's an infinite regress of causes, with no beginning, but God somehow accounts for why that infinite regress has always existed, you have basically the same problem. God may enable the existence of things forever into the past and maybe the future, but now you have to account for God---why has he always been around, and able to do such an amazing thing? What enables God to always have existed? (And what enables that...)

The truth is that asking why is evidently not always a good question. Causal why questions only make sense for particular things, where there can be an answer in terms of other particular things.

They can't possibly make that kind of sense when applied to everything. It's clearly a category mistake to expect an answer to the question "why everything?, because there's nothing else to serve as a possible answer.

Just asking "why everything?" actually rules out the possibility of an answer.

It's a bad question, however reasonable it seems intuitively, based on a false presupposition.

It's like asking "which way is up?" in deep space.

If you're not aware of the relevant physics, you might naively assume that "which way is up?" is always a valid question, anywhere, but it simply isn't, because up necessarily depends on local conditions, and in some places, conditions are such that there can't be an up.

When it comes to everything, it's similarly evident that there can be no why.

Anybody who claims to tell you why there's something rather than nothing at all is either lying to you, or didn't actually understand the question themselves.

Which, if I recall correctly, is what Kant pointed out about Aquinas's first three ways (and a bunch of other variants of the Cosmological argument) in the 18th century.

The Cosmological Argument is long dead, except among apologists, who refuse to really understand the deeply weird question they're pretending to give a sensible answer to.

By Paul W., OM (not verified) on 10 Mar 2010 #permalink

Oh boy, Kel.

Many of you claim that these arguments have been refuted. Please tell me (if you can) how a particular argument has been refuted.

Remember that?

Wasn't enough of a Gish Gallop before, now we must refute [checks, giggles] hundreds of proofs?

You bastard! :)

By John Morales (not verified) on 10 Mar 2010 #permalink

In Michael Shermer's How We Believe, he goes through various philosophical arguments for God's existence. It covers all the Aquinas and much more including modern arguments that have stemmed from scientific inquiry (ID, fine tuning, etc.) The book is well worth a read, and only ~$10 to buy.

Seriously though, you're making an argument from personal ignorance. Just because you haven't heard a refutation, it doesn't mean that there aren't refutations out there. The arguments have been discussed extensively and discarded by all but the most hardened of apologists. Alvin Plantinga rather focuses on the Ontological Argument and Reformed Epistemology. William Lane Craig who uses the Cosmological Argument prefers to use the Kalam Cosmological Argument to using the Aquinas argument.

75. PEACOCK ARGUMENT FROM ORIGINALITY
(1) I have written the following to demonstrate the existence of God.
(2) [insert entire text of a William Lane Craig article]
(3) Therefore, God exists.

Any chain of causality must ultimately begin with one of three things:

1. An infinite regress.
In which case there really is no beginning.

2. Nothing.
Which means everything came from nothing - a logical absurdity.

3. An eternal, uncaused first cause.
This is the most reasonable of the three IMO.

By Daniel Smith (not verified) on 11 Mar 2010 #permalink

Daniel Smith #312,

Point 2: When you say "nothing" you really mean "nothing we can know" which actually is not a logical absurdity and a possibility.

By aratina cage (not verified) on 11 Mar 2010 #permalink

How did your imaginary deity come into being? That is the logical fallacy. You are presupposing an eternal deity with your verbal salad. Already in deep doodoo.

By Nerd of Redhead, OM (not verified) on 11 Mar 2010 #permalink

This took me way too long to compose. Oh, well. Here it is anyway:

My arguments are more basic and apply to all of matter - not just life and complexity.

Then they apply to evolution as well, as your very disclaimer itself implies. But my counterarguments hold regardless.

We know that things that are material (matter), material-dependent (energy) and immaterial (time, space, mathematics, etc.) exist. You all seem to think these things are sufficient to explain the organization of matter. I don't.

You have not explained why you don't in a clear and coherent way.

You have yet to elucidate a reason why the properties of matter favor the organization we observe.

You have yet to elucidate a reason why we should need a reason.

Scientists are indeed looking at chaos theory and theories of emergence to better explain what you might intend by "organization". Is that in fact what you want?

Your reasoning seems to be that since we can point to forces at work in nature, those forces must be what organizes nature.

It's called "parsimony". Have you in fact heard of it?

It's akin to pointing at the forces at work in a car and concluding that those forces organized the car (and no, my position has nothing to do with complexity. It could just as easily be a table.)

That's a false analogy, either way.

As for the other four ways, there is no denying that the science of the day was not as advanced as the science of today, that does not however negate the metaphysical arguments made.

The problems are not just with their science, but with their logic. Do you understand that the logic is flawed, and that the conclusions are therefore unreasonable? Do you even know what the fallacies being referred to are?

There cannot be an infinite regress of movers (else there cannot be a first mover to start the whole thing). Therefore there must be an unmoved mover.

This assumes its conclusions, twice -- that there cannot be an infinite regress in order to conclude that there must be a "first mover"), and contains a subtle self-contradiction (an "unmoved mover" is an oxymoron).

Besides, from a scientific perspective, the entire argument is a category error: you cannot speak of motion without discussing energy. Everything that moves, ultimately moves because energy is input into it, and in fact, all matter is made of atoms that are moving all the time if they are at a temperature above absolute zero (which they are in most of the universe). And of course, matter can be converted to energy (and vice versa).

Or to put it another way, "motion" is an inherent property of matter.

While all matter and energy can be traced back to the first few microseconds of the Big Bang, we cannot assert with any certainty how that matter and energy arose. We certainly cannot alter the arguments regarding motion above to assert that there cannot be an infinite regress of matter and energy (or something matter-like or energy-like), nor that matter and energy cannot spontaneously occur. See below re virtual particles.

It could be said that animals move themselves, so this negates that. But are animals really moving themselves? Or is one part moving another?

Animals move as the result of differential tissue contraction; chemical energy transformed into mechanical energy. This is pretty basic science, here.

Nothing can be the efficient cause of itself.

This assumes its own conclusion. It's one of those things that looks reasonable, but could not be proven to be true, and could be false in some particular case even if generally true.

One way in particular that it's been shown to be probably false is in spontaneous virtual particle pair production, and in radioactive decay. More generally, it's been suggested that nothingness is unstable.

There cannot be an infinite regress of efficient causes (for the same reasons cited above).

This is invalid, largely for the same reasons I cited above. See also below.

Therefore there must be a first efficient cause.

Which implicitly contradicts the initial premise: A first cause is indistinguishable from something self-caused.

The problem with discussing the very concept of cause is that it implicitly involves time. But asserting that there cannot be an infinite regress of things happening in time (which is what the above assertion amounts to) means that there must have been an absolute t0. But if there's an absolute t0, positing a cause before that point implies that something happened before the absolute t0, which means the t0 is not in fact absolute but relative. The goalpost shifts back. You've contradicted your initial premise, again.

As best we can tell scientifically, time in our universe itself began with the Big Bang. We cannot say for certain what happened before that, or even if it makes sense to say there was a before. Does time extend back infinitely or loop back on itself? Is there some sort of meta-time and meta-space where universes come into existence and expand? Does the meta-time and meta-space have a beginning, or do they regress infinitely? Are they embedded in a meta-meta-time and meta-meta-space? Does any looping occur at any level? If not, why not?

This is all the sort of speculation that gives cosmologists headaches (and certainly gives me a headache), but you cannot derive conclusions as if you know which cosmological scenario is true, especially when you end up contradicting yourself and using fallacious logic in the context of your chosen scenario.

The argument from contingency.

This looks to me like a convoluted restatement of the previous argument, and which smuggles in circular reasoning again. Is it not the case that a contingent thing is one that is caused? Is it not the case that a necessary thing is one that is uncaused?

The argument from degree. That there are observed to be degrees of qualities in beings. Some beings are more or less intelligent, good, true, etc. That there are gradations in these qualities points to there being a maximum intelligence, goodness, trueness

A potential maximum does not imply that the actual maximum is infinite. Any maximum that actually exists in the real world embodied in a real being is necessarily not infinite.

This is particularly true for vague and relative concepts like "goodness". What exactly does that even mean?

This is even the case for "trueness" itself. Gödel proved that complete knowledge of truth is logically impossible: there are true things which cannot be proven true, and false ones that cannot be proven false. Given that, how is it even possible to know how to define "maximum" trueness, or that something has "maximum" trueness?

And so on.

By Owlmirror (not verified) on 11 Mar 2010 #permalink

2. Nothing.
Which means everything came from nothing - a logical absurdity.

sigh

3. An eternal, uncaused first cause.
This is the most reasonable of the three IMO.

By Rev. BigDumbChimp (not verified) on 11 Mar 2010 #permalink

Any chain of causality must ultimately begin with one of three things:

Even the terms "causality" and "begin" have implicit hidden assumptions about time in them. You're trying to argue as though time were a well-defined and well-understood concept. It's not.

1. An infinite regress.
In which case there really is no beginning.

Look more closely at your #3.

2. Nothing.
Which means everything came from nothing - a logical absurdity.

Not at all. What exactly is nothing? As noted @#316 (and by others), it's been suggested that "nothing" is unstable and necessarily produces "something". Perhaps it's not the case that nature abhors a vacuum, but rather that a vacuum, by its nature, abhors itself.

3. An eternal, uncaused first cause.
This is the most reasonable of the three IMO.

How does "eternal" differ from "infinite"? Again, there's an implicit hidden assumption about time in the very word. So too with "uncaused" and "first cause" -- there's that implicit contradiction between your assumed absolute t0 shifting eternally backwards, or a regress.

And you left out a possibility I suggested above -- not an infinite regress, but a causal temporal loop, or meta-temporal loop. No, I don't know how that would work, but it's at least as reasonable as the other options, which are equally difficult to explain coherently.

By Owlmirror (not verified) on 11 Mar 2010 #permalink

Me: There cannot be an infinite regress of movers (else there cannot be a first mover to start the whole thing). Therefore there must be an unmoved mover.

Owlmirror: This assumes its conclusions, twice -- that there cannot be an infinite regress in order to conclude that there must be a "first mover"), and contains a subtle self-contradiction (an "unmoved mover" is an oxymoron).

I've worded it badly but the idea is that an infinite regress of movers (as Aquinas defines them) is a regress of dependent relationships where the subject (the object under consideration) is moved by the former, (not in time but in priority) and the former is moved by its former, (again not in time but in priority), and etc. The logic of it is that the existing subject cannot move unless the former exists, and the former cannot move unless its former exists, ALL AT THE SAME TIME. An infinite regress has no first (in priority) mover, so it cannot have a second, a third, and so on. So dependent motion (of the kind Aquinas was talking about) cannot be caused by an infinite regress of movers.

The same logic applies to Aquinas' argument from efficient causes. He is careful not to talk about accidental causes (as he called them - I prefer 'historical causes'.) The difference being that an efficient cause is what is causing something to exist RIGHT NOW. Its cause must also exist RIGHT NOW, as must its cause, and its cause, etc. An infinite regress has no first (in priority) cause, so it cannot have a second, a third, and so on. So efficient causation also cannot be caused by an infinite regress of causes.

The accidental or historical cause is the thing that originally was responsible for bringing something into being (your parents being the accidental cause of you.) That's a whole other chain of causality and does not fit within the same logical framework. Interestingly Aquinas thought there could be an infinite regress of accidental causes.

By Daniel Smith (not verified) on 11 Mar 2010 #permalink

Where did your "first mover" come from? Assuming an uncaused "first mover" is, as I said previously, special pleading. So you have to show how your "first mover" appears on the scene without being caused by a "zero mover", which we can also call "nothing."

By 'Tis Himself, OM (not verified) on 11 Mar 2010 #permalink

I've worded it badly but the idea is that an infinite regress of movers (as Aquinas defines them) is a regress of dependent relationships where the subject (the object under consideration) is moved by the former, (not in time but in priority) and the former is moved by its former, (again not in time but in priority)

This is still confused.

What does "priority" mean if it doesn't mean temporal priority?

The logic of it is that the existing subject cannot move unless the former exists, and the former cannot move unless its former exists, ALL AT THE SAME TIME.

Because... motion cannot be transmitted? How does this make sense, even in primitive medieval concepts of physics?

An infinite regress has no first (in priority) mover, so it cannot have a second, a third, and so on. So dependent motion (of the kind Aquinas was talking about) cannot be caused by an infinite regress of movers.

Regardless of what Aquinas was talking about, he was wrong because he did not take into account how physics actually works in the real world, as opposed to how he thought physics worked.

The same logic applies to Aquinas' argument from efficient causes. He is careful not to talk about accidental causes

This is even more confused. As best I can tell, efficient causes are synonymous with what you call accidental/historical causes.

http://plato.stanford.edu/entries/aristotle-causality/

By Owlmirror (not verified) on 11 Mar 2010 #permalink

What does "priority" mean if it doesn't mean temporal priority?

The example most often used is that of a man pushing a rock with a stick. The rock is moved by the stick, the stick is moved by the man - all at the same time. The man is the 'first mover', the stick is the 'second mover' and the rock is that whose motion we are seeking to explain.

This is even more confused. As best I can tell, efficient causes are synonymous with what you call accidental/historical causes.

You're confused alright. I explained it (I think) pretty clearly. Go back and re-read what I said.

By Daniel Smith (not verified) on 12 Mar 2010 #permalink

Daniel,
First, you are applying medieval logic to the physical world of today. We've learned a wee bit since Aquinas.

For one thing, we've learned that the world behaves very differently on scales of length, time and energy different from the way it behaves in every day life.

In terms of cosmology, there is no "before" the big bang. Space and time came into existence the big bang. What is more, the theory does an astoundingly good job of explaining why the Universe looks as it does now.

Religion had several thousand years to try to explain the physical universe. It failed utterly. Science has been kicking butt for 400 years now. Me, I bet on success.

By a_ray_in_dilbe… (not verified) on 12 Mar 2010 #permalink

Daniel, how did your deity come into being? It cannot be eternal (presupposed). Try using science, not philosophy. Evidence trumps philosophy every day of the week.

By Nerd of Redhead, OM (not verified) on 12 Mar 2010 #permalink

The rock is moved by the stick, the stick is moved by the man - all at the same time. - Daniel Smith

The force applied by the man does in fact take a non-zero time to move the stick - which will compress slightly as well as move, and the end nearest the man will start moving before the other end - and the same is true of the rock. When examined at finer spatio-temporal scales, as a_ray notes, things look different: in this case, the transmission of force is due to electromagnetic interactions between atoms, which cannot travel faster than the speed of light, as is true of any information-transmitting process.

More briefly: Aquinas, through no fault of his own, was talking complete tosh: he had no understanding of how motion works at the atomic scale, as we do, so naturally he got into a hopeless muddle.

By Knockgoats (not verified) on 12 Mar 2010 #permalink

Shit, this thing is still going? How much more does it really need to take to show that Aquinas' arguments are no longer relevant than that modern Christian philosophers no longer use them? They have been refuted, they have been refuted for over 250 years at least now. The intellectual realm has moved on and left Aquinas for dead... get over it and move into the 21st century.

Oh and stop making the two egregious fallacies that a) you can prove a priori what is reasoned from a posteriori observations, and b) that if you prove there's a Prime Mover that means that the Prime Mover is the god of the bible. Your entire line of thinking is completely outdated. Like 500 years outdated...

By the way, did anyone else see Sean Carroll on Colbert the other night talking about his new book, From Eternity to Here?

A few thoughts on the "scientific worldview"...

1. Science is limited in its field of study:
Science studies physical, observable phenomena and its effects. It cannot, for instance, investigate the supernatural. Therefore scientific knowledge is a limited form of knowledge and cannot be the only type of knowledge that exists.

2. Scientific knowledge is incomplete:
Science has not discovered everything there is to discover, and there is no guarantee that the scientific knowledge of tomorrow will confirm the scientific knowledge of today - in fact history shows us that the opposite is often likely to be true. Appeals to future knowledge are therefore illegitimate for bolstering the incomplete knowledge we have today.

3. Science is based on metaphysical assumptions:
Science assumes, for example, that the world we observe is real, that we are not just brains in vats. Science also assumes that the laws of physics and chemistry apply universally. Indeed, science would not be possible if it did not make such assumptions. So science itself is based on metaphysics.

4. Scientific observations and results remain the same whether interpreted via a materialist worldview or a theological one:
There is no finding of science that contradicts sound theology.

By Daniel Smith (not verified) on 12 Mar 2010 #permalink

What does "priority" mean if it doesn't mean temporal priority?The example most often used is that of a man pushing a rock with a stick. The rock is moved by the stick, the stick is moved by the man - all at the same time. The man is the 'first mover', the stick is the 'second mover' and the rock is that whose motion we are seeking to explain.

And the first and second movers are still moving prior in time to the movement of the rock, so you've failed to achieve any distinction between the terms.

Just out of curiosity, have you ever taken a course in basic physics? The whole mechanics and laws of motion stuff?

I explained it (I think) pretty clearly.

You failed.

Go back and re-read what I said.

Go back and re-read Aristotle. Material, formal, efficient, and final causes. No accidents or historical causes distinct from efficient causes.

What you wrote:

The difference being that an efficient cause is what is causing something to exist RIGHT NOW. Its cause must also exist RIGHT NOW, as must its cause, and its cause, etc.

The accidental or historical cause is the thing that originally was responsible for bringing something into being

Your "accidental or historical" cause is indistinguishable from Aristotle's efficient cause, and your definition of efficient cause is not something I've seen before, nor something that seems very well thought out -- your notion of all efficient causes existing simultaneously is utterly incoherent. You're either using the wrong word, or you're thinking of some other analysis of causation. Either way, the confusion is entirely yours.

By Owlmirror (not verified) on 12 Mar 2010 #permalink

Scientific observations and results remain the same whether interpreted via a materialist worldview or a theological one

Sorry, but science is adeistic and/or atheistic. It does not allow the use of god or theology in it's interpretation of the world. So yes, there is different results for science versus theology. Try learning some science.

By Nerd of Redhead, OM (not verified) on 12 Mar 2010 #permalink

1. Science is limited in its field of study:

All fields of study that claim not to be so limited are indistinguishable from pure fantasy.

It cannot, for instance, investigate the supernatural.

The "supernatural" is an incoherent concept resulting from flaws in human perception and understanding.

Therefore scientific knowledge is a limited form of knowledge and cannot be the only type of knowledge that exists.

Scientific knowledge is the only type of knowledge that can be distinguished from make-believe knowledge.

Scientific knowledge is incomplete

This necessarily follows from its being limited.

Science has not discovered everything there is to discover, and there is no guarantee that the scientific knowledge of tomorrow will confirm the scientific knowledge of today - in fact history shows us that the opposite is often likely to be true.

Correcting the knowledge you have with better, more empirically correct knowledge, once that knowlege is discovered, is what learning is all about. It is the basis of empirical epistemology.

Not being willing to correct the knowledge that you have demonstrates mental delusion or other psychological problems.

Appeals to future knowledge are therefore illegitimate for bolstering the incomplete knowledge we have today.

A reasoned extrapolation from current knowledge and consistent with current knowledge is more legitimate than incoherent speculation.

Science is based on metaphysical assumptions:

Yes, that the universe is reasonable consistent in the way that it behaves.

What would a universe that was unreasonably inconsistent behave like?

Science assumes, for example, that the world we observe is real, that we are not just brains in vats.

Have you heard of parsimony?

Science also assumes that the laws of physics and chemistry apply universally. Indeed, science would not be possible if it did not make such assumptions.

Science is sufficiently self-correcting that if there were any evidence whatsoever that the laws did not apply universally, that specific assumption would change. It would indeed be possible to do science -- it's just that the inapplicability of the laws of physics would have further detectable effects that would be seen.

Scientific observations and results remain the same whether interpreted via a materialist worldview or a theological one:

So science defeats theology whenever they are in conflict.

There is no finding of science that contradicts sound theology.

Only if "sound theology" makes no empirical claims whatsoever.

By Owlmirror (not verified) on 12 Mar 2010 #permalink

When has rejecting a natural explanation in favor of a "supernatural" one ever made a positive contribution to our understanding of something?

Daniel @328,

1. Science is limited in its field of study:
Science studies physical, observable phenomena and its effects. It cannot, for instance, investigate the supernatural. Therefore scientific knowledge is a limited form of knowledge and cannot be the only type of knowledge that exists.

You have miss out a step.
In order for the above to work as an argument, you have to show that there is a 'supernatural' to investigate.
You have failed to do so.

P.s. When will you creotards get it into your thick skulls that once an argument has been refuted, you don't get to keep using it just because you didn't understand the refutation?
I mean, seriously, Aquinas?
His arguments were thoroughly destroyed hundreds of years before he published them.

A few thoughts on the "religious worldview"...

1. Religion has no field of study:
Religion studies npthing. It cannot, for instance, investigate the supernatural. As no religious belief can be justified true belief, religious knowledge is void of any content and can be safely ignored.

2. Religious knowledge is completely void:
Religion has not discovered anything. And there is a guarantee that the religious knowledge of tomorrow will be as void of content as the religious knowledge of today. Appeals to past, present or future religious knowledge are therefore illegitimate for bolstering the inexistant religious knowledge we have today.

3. Religion is based entirely on superstitions and myths

4. Religious observations cannot be verified.
There are no religious superstitions that can be verified empirically. Only theology that makes no empirical claims doesn't contradict science.

By negentropyeater (not verified) on 12 Mar 2010 #permalink

It would indeed be possible to do science -- it's just that the inapplicability of the universality of the laws of physics would have further detectable effects that would be seen.

Fixed.

And just to expand on that a bit further: Theoretical physics includes hypotheses about possible scenarios where one or more of the laws of physics has changed or is slowly changing or is otherwise different from what is assumed. But these hypotheses do still have to take into account what is observable from cosmological and physical evidence.

By Owlmirror (not verified) on 12 Mar 2010 #permalink

The difference between science and theology is science is based on evidence while theology is based on opinion and wishful thinking.

If a deity physically intervened in the real world then this intervention could be studied scientifically. There would be evidence of the deity. However deities seem to be reluctant to manifest themselves or their actions in the real world but remain supernatural, they do not leave evidence to be studied. Instead, theologians make up stories and pretend they "understand" their particular deities. For n theologians there are at least n+1 theologies, because theologians do change their opinions occasionally.

By 'Tis Himself, OM (not verified) on 12 Mar 2010 #permalink

Daniel,

I think you're messing up the concept of metaphysical priority.

A guy pushing a rock with a stick is not a good example. That's just regular temporal priority, as long as there's no instantaneous action at a distance.

A better example is the supposed priority of space and time and matter to motion.

You can't have motion without space and time and matter---they are preconditions for the possibility of motion.

People are now much more comfortable than they used top be in Thomas's day, with the concept of infinity, or the possibility of an actual infinite regress. Maybe there was no first moment in time, and no first cause or prime mover in that sense. (Our naive concept of causality breaks just as badly in the case of a first cause as in the case of an infinite regress.)

To salvage the Cosmological Argument, a standard move is to talk about preconditions for existence. The existence of motion presupposes space-time and matter, and that presupposes something else... and at the bottom you have something which doesn't presuppose anything, which is the Ultimate Ground of All Being, and we call that God.

It's bullshit, and gets no traction among modern philosophers, but if you're going to be a Thomist, at least get it right so that we have something worth arguing with.

By Paul W., OM (not verified) on 13 Mar 2010 #permalink

A bit late here, but the concept of simultaneity in the real world is not as simple as the ancients once thought, either.

By John Morales (not verified) on 13 Mar 2010 #permalink

Go back and re-read Aristotle. Material, formal, efficient, and final causes. No accidents or historical causes distinct from efficient causes.

What you wrote:

The difference being that an efficient cause is what is causing something to exist RIGHT NOW. Its cause must also exist RIGHT NOW, as must its cause, and its cause, etc.

The accidental or historical cause is the thing that originally was responsible for bringing something into being

Your "accidental or historical" cause is indistinguishable from Aristotle's efficient cause, and your definition of efficient cause is not something I've seen before, nor something that seems very well thought out -- your notion of all efficient causes existing simultaneously is utterly incoherent. You're either using the wrong word, or you're thinking of some other analysis of causation. Either way, the confusion is entirely yours.

Perhaps the confusion comes from the fact that I'm using Aquinas' argument - not Aristotle's.
Aquinas made the distinction in causality. I don't know if Aristotle did or not.

By Daniel Smith (not verified) on 13 Mar 2010 #permalink

All fields of study that claim not to be so limited are indistinguishable from pure fantasy.

I totally agree (and never claimed otherwise.)
My point was that scientific knowledge is not the only valid knowledge.

By Daniel Smith (not verified) on 13 Mar 2010 #permalink

Daniel, it doesn't matter whose argument you are using. Both don't work. You have nothing compared to the scientific evidence, which trumps all philosophy because it is based in reality. There is no way to resurrect your points.

By Nerd of Redhead, OM (not verified) on 13 Mar 2010 #permalink

In order to understand Aquinas' and Aristotle's concept of motion, one must understand Aristotle's actuality and potentiality.

Everything that exists is actual.

In order to change or move it must have the potential to change or to move - it must be able to become something different than it actually is or to move somewhere it actually isn't.

This potential is not some abstract concept (as in "anything we can imagine") but rather real potential - as in "what it can really become" or "where and how it can really move".

For Aristotle and Aquinas then, "motion" is to change from potential to actual and vice versa. So motion is not just about movement but rather about any kind of change.

Also, it must be stressed that nothing can be both actually and potentially the same thing at the same time. One precludes the other. If it is actually one thing, it is not potentially that same thing and if it is potentially one thing, it is not actually that same thing.

Also movement (or change) must be initiated on something that has the potential to move (or change) by something that is actual. Potential cannot move potential. For this reason, it cannot be the same thing that has the potential, else it would be both actually and potentially the same thing at the same time.

This is why Aristotle famously concluded "Everything that moves is moved by another".

By Daniel Smith (not verified) on 13 Mar 2010 #permalink

Daniel @343, why should I?

By John Morales (not verified) on 13 Mar 2010 #permalink

Daniel Smith @339:

Perhaps the confusion comes from the fact that I'm using Aquinas' argument - not Aristotle's.
Aquinas made the distinction in causality. I don't know if Aristotle did or not.

[proceeds to waxe pseudo-philosophically on Aristotle vx. Aquinas. Snort/Laughtrack elided.]

@342:

This is why Aristotle famously concluded "Everything that moves is moved by another".

So there's no prime mover, right?

By John Morales (not verified) on 13 Mar 2010 #permalink

So daniel, why precisely should we care about the philosophical and factually incorrect musings of some dudes from centuries/millenia ago?

they're wrong. why would it matter if their wrongness is internally consistent, complex, and appealing to you?

By Jadehawk, OM (not verified) on 13 Mar 2010 #permalink

Daniel,

My point was that scientific knowledge is not the only valid knowledge.

And you have yet to actually demonstrate that this is true.
Claiming that there is some other form of knowledge doesn't make it so.

My point was that scientific knowledge is not the only valid knowledge.it's the only kind we can rightfully call that, since no other form of "knowledge" is reliably distinguishable from fantasy and illusion. You can hardly call something "knowledge" if there's no way of telling whether it's actually real and not-wrong.

By Jadehawk, OM (not verified) on 13 Mar 2010 #permalink

argh, blockquote fail. only the first sentence was supposed to be in the quote.

By Jadehawk, OM (not verified) on 13 Mar 2010 #permalink

I seen Daniel is still busy giving philosophers a bad name by attempting bad philosophy. Philosophy without evidence is sophistry. And there is no evidence in Daniel's philosophy. Learn some science. And look at t-shirts that say it all.

By Nerd of Redhead, OM (not verified) on 14 Mar 2010 #permalink

Your "accidental or historical" cause is indistinguishable from Aristotle's efficient cause, and your definition of efficient cause is not something I've seen before, nor something that seems very well thought out -- your notion of all efficient causes existing simultaneously is utterly incoherent. You're either using the wrong word, or you're thinking of some other analysis of causation.

To clarify:
Aquinas talks about causal series that are "accidentally ordered" (ordered per accidens) and "essentially ordered" (ordered per se.) I mistakenly referred to "accidental causes" and "efficient causes" earlier. It must be noted however that Aristotle made the same distinction - calling the latter the "immediate efficient cause".

An accidentally ordered causal series does not depend upon each member remaining in existence in order for the series to continue. This is like the series of fathers and sons - once a son is born, even if the father dies, the son can still continue on and cause another son. Each cause is, in this way, independent of its predecessor.

An essentially ordered causal series is, on the other hand, a dependent series; the entire series depends upon each member existing simultaneously. Going back to the hand pushing the stick pushing the stone; if the stick doesn't exist simultaneously with the hand and the stone, the series falls apart. And, in an essentially ordered series, each member derives its motion from the first member; it is the hand that moves the stick and the stone. Another illustration is a freight train: although all the cars are connected and each is moved by the car immediately joined to it, ultimately the movement depends of the first car - the engine.

To go back to the hand, stick and stone; we know that the hand itself is not actually the first mover in the series - it being dependent upon the arm, which is dependent upon the muscles, which are dependent upon neurons, which are dependent on the state of the nervous system, which is dependent on its current molecular structure, which depends upon its atomic basis, which depends upon electromagnetism, gravitation, the weak and strong forces - and so on - each member simultaneously dependent upon the former.

Such a chain must ultimately end with a prime mover - something that is pure actuality (that is, something that is not dependent upon anything else.) All motion or change is dependent upon such a thing and that thing cannot be matter since matter can change.

By Daniel Smith (not verified) on 14 Mar 2010 #permalink

Daniel (or should that be Denial?) Smith, are you now trying to argue that everything must have a cause? Fortunately we are not in the 12th century now, and possibly to the discouragement of Acquinas’ mediæval views, we now have observed plenty of examples of acausality in nature, such as radioactive decay or the virtual pair-production of particles and anti-particles. You’re just giving us yet another repetitious version of the same sort of infinite regress represented by Zeno’s paradox, ignorant of any subsequent developments in either mathematics or physics: well in the case of Zeno, guess what happened there.

Modern cosmologists are very far from needing to assert a "first cause" before the earliest known timescale of the Universe, when a massively inflationary process began from highly dense and energetic starting conditions, but the point here is, in some of the theories postulated there is no need for anything to have happened prior - physicists such as Victor Stenger have given examples that sounds rather paradoxical, that not only the universe could have begun out of nothing through a process like quantum pair production, but that such an event would represent a lower energy state (i.e. more probable) than the contrary, that the event never occurs.

By Pope Maledict DCLXVI (not verified) on 14 Mar 2010 #permalink

All fields of study that claim not to be so limited are indistinguishable from pure fantasy.
I totally agree (and never claimed otherwise.)
My point was that scientific knowledge is not the only valid knowledge.

Hm.

What is knowledge?

What makes knowledge valid?

Your original claim was that belief in God was more reasonable than a rejection of belief in God. What do you know about God? How do you know that that knowledge is valid? What is your reasoning now, given that all of Aquinas has been demonstrated as being unreasonable?

=============

For more on the scientific worldview, see [Recovering Sight after Scientism, by Edward Feser, parts I and II]

Well, I skimmed the essays, and it certainly looks like Feser is demonstrating a deep lack of comprehension about science. Yes, there is the underlying "metaphysical" assumption or axiom of a coherent universe, but philosophy offers nothing otherwise in elucidating what a universe without this coherence would be like, or why we should reject it, or infer more about it than it being a reasonable starting point.

The greatest strength of science is that which you so denigrated above: it is based on a method of constant self-correction, that correction being based on new evidence and logic, limited by parsimony and falsifiability.

He can crow about how the metaphysics of ancients and medievals has never been surpassed, but he only demonstrates that he fails to understand the metaphysical strength of the scientific method, and the corresponding weakness of philosophy that has no empirical or logical method of self-correction. Philosophy can argue about the findings, classifications, and definitions of that which has been discovered by science, but it must submit to new empirical evidence discovered by science, and is limited by basic logic.

When philosophy has no empirical or logical grounding, it ultimately descends into arbitrary personal whims, often defended by corrupt and fallacious logic like Aquinas'.

=============

Aquinas talks about causal series that are "accidentally ordered" (ordered per accidens) and "essentially ordered" (ordered per se.) I mistakenly referred to "accidental causes" and "efficient causes" earlier. It must be noted however that Aristotle made the same distinction - calling the latter the "immediate efficient cause".

OK. Then my hypothesis that you were using the wrong word above is proven. I acknowledge the distinction as you now phrase it, and I think you're expressing it better since you've obviously been refreshing your memory on the subject.

Such a chain must ultimately end with a prime mover - something that is pure actuality (that is, something that is not dependent upon anything else.) All motion or change is dependent upon such a thing and that thing cannot be matter since matter can change.

So it looks like you are now arguing for the "prime mover" as if there is some thing that is responsible for what I call the universe behaving in a consistent fashion. The "ground of being" or something. Is that correct?

Assuming just for the sake of argument that your conclusion is correct, despite not having any empirical evidence in its favor, are you arguing, in unreasonable agreement with Aquinas, that this is "God", despite having no logic or evidence for this assertion? That is, the inherent consistency of the universe is itself a personal God; a God who is a person?

Does it not follow that your assertion in response to me @#256 is false?

And does it not also follow that everyone who suffers and dies does so because this omnipresent God of yours is utterly and mercilessly indifferent to human suffering and death?

And would it not also follow that God's "ground of beingness" acts like nothing more than the universe behaving in a consistent fashion because for it to not behave in a consistent fashion would be incoherent?

By Owlmirror (not verified) on 14 Mar 2010 #permalink

are you now trying to argue that everything must have a cause?
No. I'm not.

By Daniel Smith (not verified) on 16 Mar 2010 #permalink

What is your reasoning now, given that all of Aquinas has been demonstrated as being unreasonable?

I would not concede that point. Most of the objections to Aquinas that have been raised here were answered by Aquinas himself long ago. Many are simply strawmen - based on caricatures of his arguments. All - thus far - ring hollow.

By Daniel Smith (not verified) on 16 Mar 2010 #permalink

Many are simply strawmen - based on caricatures of his arguments. All - thus far - ring hollow.

Now if you were talking about Aquinas' arguments, you would be right. They were refuted years ago, and your ignorant attempts to resurrect them here ring hollow, just like the strawmen they are. So far, you have demonstrated nothing but your inane belief in a deity. Just being stubborn but wrong will not get you anywhere. Take your argument elsewhere. You have nothing.

The best way to convince us of a deity is with hard physical evidence, evidence that will pass muster with scientists, magicians, and professional debunkers as being of divine, and not natural, origin. Philosophy won't cut the mustard, since the deity in the babble interacted with world, and traces of his doing so should be present in the world, and can be detected with science. A totally philosophical deity isn't worth the thought going into the imagination of creating one, as it can do nothing in the real world.

By Nerd of Redhead, OM (not verified) on 16 Mar 2010 #permalink

I would not concede that point. Most of the objections to Aquinas that have been raised here were answered by Aquinas himself long ago. Many are simply strawmen - based on caricatures of his arguments. All - thus far - ring hollow.

Ok... that doesn't really answer Owlmirror's question, though (amazingly!!!).

So leaving the last line out, here's the question again, which I'd like to see you answer, frankly:

What is knowledge?

What makes knowledge valid?

Your original claim was that belief in God was more reasonable than a rejection of belief in God. What do you know about God? How do you know that that knowledge is valid?

Hint: Faith is not an answer to any of those, any more than "eleventy-six" is.

By Celtic_Evolution (not verified) on 16 Mar 2010 #permalink

the hand itself is not actually the first mover in the series - it being dependent upon the arm, which is dependent upon the muscles, which are dependent upon neurons, which are dependent on the state of the nervous system, which is dependent on its current molecular structure, which depends upon its atomic basis, which depends upon electromagnetism, gravitation, the weak and strong forces - and so on - each member simultaneously dependent upon the former.Such a chain... -Daniel Smith

But it's not really a chain because you can't isolate a single cause of anything in that system, not even at the highest levels. There are too many interdependencies at any moment you choose to examine, some highly important in the outcome, others with nearly zero importance, and everything in between.

I think the best you could do to make a "chain" would be to pick out the greatest factor that would lead to the next greatest factor at any moment in time at any scale of resolution that would ultimately lead to the action under consideration. You would have to build fuzziness (unknowability) into the causation chain to even make the chain.

By aratina cage (not verified) on 16 Mar 2010 #permalink

Daniel, you appear to be arguing that you are right until we prove you wrong. This is the reverse of the situation, but it is typical godbot logic. You are wrong until you prove yourself right. Essentially, you have to sell to us on your logic and evidence (especially the latter). So far, you haven't done that, starting with your premises. It would help if you used present day scientific knowledge and definitions. But then, you will lose Aquinas' argument, which is based on old, antiquated, and wrong physics, and obsolete definitions.

By Nerd of Redhead, OM (not verified) on 16 Mar 2010 #permalink

I wonder what Aquinas would have made of the concept of the past light cone? It pretty much destroys any causal-chain concept.

By Stephen Wells (not verified) on 16 Mar 2010 #permalink

I would not concede that point.

Because it conflicts with your psychological commitment to defend your theological presuppositions?

Because you just don't want to admit that Aquinas was easily capable of making fallacious arguments?

All of the above? Something else?

Why did you ignore every thing else that I wrote besides that one sentence?

Most of the objections to Aquinas that have been raised here were answered by Aquinas himself long ago.

We cannot trust you on this, given that you couldn't even remember Aquinas' arguments on causality without a re-read.

Bring forth these counterarguments, if they exist.

Would you concede if Aquinas' alleged "counterarguments", assuming they exist, can be shown to be fallacious as well?

Many are simply strawmen - based on caricatures of his arguments.

Um... people were responding to the arguments as you summarized them. Either your summaries were caricatures, as you say, or the responses were indeed valid refutations of Aquinas. If the refutations were invalid, you could have shown how (and you still can, I suppose).

But assuming that you are implicitly confessing to mistakenly distorting Aquinas' arguments, please feel free to demonstrate the difference between what has been soundly refuted, and what Aquinas was actually arguing.

All - thus far - ring hollow.

That's the death knell ringing for your arguments that you hear.

By Owlmirror (not verified) on 16 Mar 2010 #permalink

Ichthyic #21

I CAN direct you to the many arguments that have clearly shown this to be the case.

doubt you're really interested though, so long as this tactic appeases the faithheads, right?

If you can, go ahead.

I'm not disinterested, but I do think that what Ken does well has value. And I think that no one else is doing it as well.

Caine, Fleur du mal #24

Really? So it's okay for him to spew utter nonsense such as this?:

This sort of sly intervention, he argues, is vital to the Creator's project: if God were to re-grow limbs for amputees, for instance -- if God were to perform the sort of miracles demanded by atheists as proof of his existence -- the consequences would be disastrous. "Suppose that it was common knowledge that if you were a righteous person and of great faith and prayed deeply, all of a sudden, your limb would grow back," he says. "That would reduce God to a kind of supranatural force . . . and by pushing the button labeled 'prayer,' you could accomplish anything you wanted. What would that do to moral independence?"

Bullshit to your fanboi take. He's doing harm, just as all religious people do, by doing everything he can to keep people mentally tied to a construct which allows people to justify their pettiness, nastiness and evils towards others.

I think that you could isolate areas where anyone is doing harm. The quote seems to be a theological discussion, although I don't know the source. Which is preaching to the converted, and not likely to do a great deal of harm. Certainly its not the sort of writing that would compell a freethinker towards catholocism.

By Bored Wombat (not verified) on 16 Mar 2010 #permalink

The quote seems to be a theological discussion, although I don't know the source. Which is preaching to the converted, and not likely to do a great deal of harm.

I sometimes wonder about whether it is or isn't harmful.

On the one hand, we can roll our eyes at those who say that evolutionists just want to deny God, and point to religious evolutionary biologists. That's useful from a strategic perspective.

On the other hand, religious apologetics, like the theodicy cited above, are ridiculous and completely fallacious defenses of religious presuppositions, and when supposedly intelligent scientists use them, it looks like religious scientists are defending sloppy thinking. Which they are, of course.

And defending that kind of sloppy thinking, in the name of religion or anything else, is the opposite of rational.

By Owlmirror (not verified) on 16 Mar 2010 #permalink

That's useful from a strategic perspective.

AYUP.

it's like I've said for years now...

NOMA logically holds no water.

that sure didn't make it any less effective an argument in staving off the fundies in Ohio.

there's the battles, and then there's the war...

I used to take my students on a field trip to a nearby woods, and have them look at the bark on the trees, record density analyses of lichen and other epiphyte coverage, etc. This is the science, and is easy to focus on to the exclusion of other things...

...then I would show them the plans to log the forest the following year, and walk them to the edge of the woods where they could see the clear-cuts surrounding.

...point being, I like pharyngula because HERE we can indeed agree that:

"And defending that kind of sloppy thinking, in the name of religion or anything else, is the opposite of rational."

...but what i see more and more on the battlelines is that Miller's approach has strategic value too.

the long war is one of the rational against the irrational, but the battles often are of the form of "let the lesser evil prevail".

for the person who asked me about this:

"I CAN direct you to the many arguments that have clearly shown this to be the case.:

I would suggest starting by searching this very site for Miller's name.

then go to Jerry Coyne's site and do the same.

then Larry Moran.

...or did you need me to hold your hand?

And I think that no one else is doing it as well.

and you base that on what, exactly?

I sometimes wonder about whether it is or isn't harmful.

just to be clear, ALL logical argument about the compatibility of religion and science MUST be amenable to rigorous debate, we all agree on that much, I hope. We really can't worry about what potential "harm" is done by debating it.

In fact, it's essential the point be at least made once that accomodationist approaches like Miller's and AAAS and Collins, et al, are not logically supported.

What is really pissing me off about this accomodationist debate is that rather than just accept that the points the accomodationists make really AREN'T logically supported, instead they further rationalize, making themselves look more like they are arguing religious apologetics instead of logic and science.

...all the while using the "but my approach works!" to try and silence criticism.

*sigh*

Why can there not be tacit agreement that there is value to a battle tactic, without agreement that said tactic is necessarily in the long term, sustainable?

how about minefields?

fantastic tactical applications in certain situations, yet most countries (US still excluded?) think it abominable and have banned using them.

I hope that eventually, we will move past the necessity of using religious bait as a tactic in the long term war for rationality, but for now, I cannot disagree that there is tactical value in utilizing the NOMA minefield from time to time, or in the strangely bizarre religious apologetics of Collins and Miller.

again, on the actual battlefield it often comes down to who would you wish running your local schoolboard:

a YEC

or a theistic evolutionist?

...as a final note, I only say all this to clarify, as I've said many times before, that there needs to be a clear separation between the debate on tactics, vs the debate on rationality.

Why oh why is it so hard for accommodationists to realize that we can easily separate the two arguments, and debate the logical merits of their positions entirely apart from their current tactical merits?

and to Daniel Smith...

Aquinas and Aristotle?

REALLY?

*pssst* there were many philosophers that came after them that argued their own points better than both did.

...and even those were proven wrong.

I suggest trying to move yourself at least into the 20th century.

You channelling your inner Truth Machine, Ichthyic? With the multiple consecutive short posts, I mean.

By WowbaggerOM (not verified) on 16 Mar 2010 #permalink

...or hell, why not argue the real basis for western religious thought:

Plato.

Have you ever read the introduction to Futuyma's "Evolutionary Biology"?

It traces rather well the impact of Platonic thought on Western Philosophy, including the concept of the transcendent ideal and the Great Chain of Being.

why did this way of thinking fail?

do you even realize it did?

If not, you still have a lot of philosophy to wade through.

You channelling your inner Truth Machine, Ichthyic? With the multiple consecutive short posts, I mean.

honestly, my current liver disfunction has caused my brain to be flooded with bilirubins. that, and the concurrent lack of sleep make coherent thought lasting longer than a minute or so... difficult.

apologies for any incoherence. I'm hoping the multiple posts in rapid succession will act as a kind of "gestalt".

:P

But it's not really a chain because you can't isolate a single cause of anything in that system, not even at the highest levels. There are too many interdependencies at any moment you choose to examine, some highly important in the outcome, others with nearly zero importance, and everything in between.

Aristotle recognized this. I believe he proposed a hierarchy of causes arranged by priority (though I'm unable to find the reference to that at the moment.) It is still true that any causality chain of the type I'm speaking about - no matter how branched - still ultimately depends on all the crucial elements being in place simultaneously.

I think the best you could do to make a "chain" would be to pick out the greatest factor that would lead to the next greatest factor at any moment in time at any scale of resolution that would ultimately lead to the action under consideration. You would have to build fuzziness (unknowability) into the causation chain to even make the chain.

So long as nothing is removed from the proposed chain that would be necessary in the real world, I don't have a problem with that.

What is knowledge?

Good question. I guess it has to do with the amount of information we can spout about a certain subject.

What makes knowledge valid?

Adherence to truth. (Go ahead ask - I dare you!)

Your original claim was that belief in God was more reasonable than a rejection of belief in God. What do you know about God?

Not a lot. I'm always seeking to know more.

How do you know that that knowledge is valid?

How do we know any knowledge is valid? Scientific knowledge is overturned all the time. What it ultimately comes down to is the ability to explain our world and our experience of it. I seek to verify my knowledge by putting it into practice and seeing if it works or by putting it out there in forums like these to hear the objections to it and decide for myself if they are valid.

Bring forth these counterarguments, if they exist.

OK, lets start with Hume's objections to causality. He objected partly to Aristotle and Aquinas on the grounds that chains of causality were temporally ordered and therefore did not necessarily follow. I've already provided the counterargument to that. Hume also argued, in attempting to refute Aquinas, that events were causes. Someone throws a brick - that's one event. A brick smashes a window - that's another event. For Hume, the two events could conceivably be unrelated, therefore (again) one does not necessarily cause the other. Aquinas and Aristotle however, broke causes down into categories. There are immediate or essential causes and accidental or temporally ordered causes. Both did not (to my knowledge) spend time arguing about the latter. Second, they both held that it's not events that are causes but things. The brick is the immediate efficient cause of the window shattering. An immediate efficient cause must exist simultaneously with its effect and therefore the two are related by necessity.

Nevertheless, people still want to pretend that Hume somehow refuted Aristotle and Aquinas. Unless he had better arguments than that, (and I'll leave it to you to provide them if he did), he didn't.

Then there's the argument that since Aristotle's physics was wrong, the rest of his philosophy must also be wrong. We're not talking about Aristotles physics though, we're talking about his metaphysics. His four causes - summed by the questions: What is it? What is it made of? How did it get here? and What does it do? - are relevant whether the Sun circles the Earth or whether the Earth orbits the Sun. IOW, just because Aristotle believed the science of his day (which - shock - was wrong!) doesn't mean that all of his ideas were bogus.

Then there's the "if everything has a cause then what caused God?" argument. This is a strawman since neither Aristotle nor Aquinas ever said "everything has a cause".

Also we have the argument that "just because you can show there must be a prime mover it hasn't been shown that that must mean God". I don't have the space to answer this one. Aquinas devoted countless pages in his works to proving just that though. Read Aquinas.

Would you concede if Aquinas' alleged "counterarguments", assuming they exist, can be shown to be fallacious as well?

Of course I would. Give it your best shot.

By Daniel Smith (not verified) on 17 Mar 2010 #permalink

Aristotle recognized this. I believe he proposed a hierarchy of causes arranged by priority (though I'm unable to find the reference to that at the moment.) It is still true that any causality chain of the type I'm speaking about - no matter how branched - still ultimately depends on all the crucial elements being in place simultaneously. -Daniel Smith

So you are saying that Aristotle recognized that you were wrong in #351 about a prime mover? Or are you saying that there are infinite hidden prime movers?

So long as nothing is removed from the proposed chain that would be necessary in the real world, I don't have a problem with that.

If you don't have a problem with not knowing, then why do you put a god, an imaginary creature of the sort not detectable by instruments, at the head of all causation? Shouldn't you realize that you cannot know the cause of anything with absolute certainty given that exhaustively tracing back the cause of a single movement in a moment in time would be an unending process in itself and tracing it back through time would be so much more impossible?

If your god is the plug to fit the hole of unknowability in causation, isn't it rather like a perpetual motion machine where you disregard much of reality and focus on an elementary understanding of something (in this case, that you can probably find the cause of highest priority) to come up with the device (in this case, your all-causing god)?

By aratina cage (not verified) on 17 Mar 2010 #permalink

So you are saying that Aristotle recognized that you were wrong in #351 about a prime mover? Or are you saying that there are infinite hidden prime movers?

I don't think you understand what a prime mover is.

By Daniel Smith (not verified) on 17 Mar 2010 #permalink

God.

There's nothing to know.

By stevieinthecit… (not verified) on 17 Mar 2010 #permalink

I don't think you understand what a prime mover is. -Daniel Smith

Could you explain it to me?

By aratina cage (not verified) on 17 Mar 2010 #permalink

I don't think you understand what a prime mover is.

I don't think you understand it either, or that the concept is no longer used in physics because it has proven to be useless. So if you can't define it and show that it is a concept used today, other than by sophist philisophers, you have nothing. And we don't need to read your well refuted philosohers of nonsense, better known as theology. We are well aware of the fallacies they commit, just like you are doing, starting with presupposition. You need to learn science.

God is meaningless without conclusive physical evidence. Produce some.

By Nerd of Redhead, OM (not verified) on 17 Mar 2010 #permalink

What is knowledge?Good question. I guess it has to do with the amount of information we can spout about a certain subject.

How odd. I would have thought that you would go with "justified true belief", and then we could go into a discussion of epistemology.

What makes knowledge valid?
Adherence to truth. (Go ahead ask - I dare you!)

What -- besides empirical evidence and logical consistency -- can be known to be true?

How can it be known?

Your original claim was that belief in God was more reasonable than a rejection of belief in God. What do you know about God?
Not a lot. I'm always seeking to know more.

Do you "know" that your "knowledge" is valid?

How do we know any knowledge is valid?

Sigh. Empirical evidence and logical consistency. Parsimony and falsifiability. All knowledge besides the basics is held to provisionally.

Scientific knowledge is overturned all the time.

Scientific knowledge is corrected with better understood empirical evidence, as I keep having to point out. Why do you not acknowledge this? Are you simply that intellectually dishonest?

What it ultimately comes down to is the ability to explain our world and our experience of it.

What about experiences that are demonstrated to be illusory?

========

There are immediate or essential causes and accidental or temporally ordered causes. [...] An immediate efficient cause must exist simultaneously with its effect and therefore the two are related by necessity.

Aquinas does not qualify his reference to "efficient causes" as being "immediate" or "essential". And why did he need two arguments if both were ultimately the same, namely, for a prime mover that is exactly the same as a first [immediate] efficient cause?

Then there's the argument that since Aristotle's physics was wrong, the rest of his philosophy must also be wrong. We're not talking about Aristotles physics though, we're talking about his metaphysics. His four causes - summed by the questions: What is it? What is it made of? How did it get here? and What does it do? - are relevant

Being able to ask questions does not mean that the answers one gives are correct -- nor that the questions are necessarily phrased correctly in every case.

Also we have the argument that "just because you can show there must be a prime mover it hasn't been shown that that must mean God". I don't have the space to answer this one.

Don't have the space? These comment boxes can hold a lot of text.

Perhaps you don't know how to. Or perhaps you know that you can't, because Aquinas sure as hell didn't, and you are just being disingenuous.

Really, this is the most important part of the argument, and you can't even summarize it or reference it?

Aquinas devoted countless pages in his works to proving just that though.

Why would he devote countless pages given that he simply asserted it?

http://dhspriory.org/thomas/summa/FP/FP002.html

"Therefore it is necessary to arrive at a first mover, put in motion by no other; and this everyone understands to be God."

He doesn't refer to any "proof" of his second clause in the conclusion of any of his "five ways", and argument by fiat is a logical fallacy.

Read Aquinas.

Which part? Can you at least point to the section where this famous "proof" starts? Here's his entire corpus:

http://dhspriory.org/thomas/

Go nuts.

By Owlmirror (not verified) on 17 Mar 2010 #permalink

Owlmirror,

You understand that Aquinas' "five ways" are mere summaries of his proofs of God and that they were part of an introductory text meant for beginners - don't you?

He wrote volumes on the subject. Taken in context, most of your arguments don't even apply to what he actually said or believed. You (and most who think he's been refuted) act as if the five ways was all he had to say on the subject.

By Daniel Smith (not verified) on 18 Mar 2010 #permalink

ok, Daniel Smith... please provide for us any one of Aquinas' "proofs" for god that you think has not been refuted.

By Celtic_Evolution (not verified) on 18 Mar 2010 #permalink

He wrote volumes on the subject. Taken in context, most of your arguments don't even apply to what he actually said or believed. You (and most who think he's been refuted) act as if the five ways was all he had to say on the subject.

Daniel Smith, what this looks like is a case of you trying to get out of backing up your claims. Owlmirror graciously linked to everything Aquinas had to say, but rather than find and point to the relevant parts that you purportedly recall, you engage in hand-waving: "It's in there, somewhere!" That is not good enough, and sadly this is a pattern you are developing in your argumentation style.

By aratina cage (not verified) on 18 Mar 2010 #permalink

You understand that Aquinas' "five ways" are mere summaries of his proofs of God and that they were part of an introductory text meant for beginners - don't you?

You understand that your evasion and condescension makes you look disingenuous and dishonest - don't you?

Taken in context, most of your arguments don't even apply to what he actually said or believed.

All I've seen is that what he said contained logical fallacies and that what he believed was absurdist nonsense.

If you know better, demonstrate it.

You (and most who think he's been refuted) act as if the five ways was all he had to say on the subject.

The offer for you to back this up -- by pointing at a strong argument that proves, using non-fallacious logic, that the prime mover and the first efficient cause and the most necessary thing &c. are in fact God -- remains on the table

I note that you've kind of abandoned the last two of the five ways -- perhaps because their logical fallacies are painfully and glaringly obvious even to you?

By Owlmirror (not verified) on 18 Mar 2010 #permalink

Wow... Daniel didn't answer any of the questions.

How strange.

By Celtic_Evolution (not verified) on 18 Mar 2010 #permalink

Ah, Daniel's trying to avoid showing his ignorance here. But if not posted here, he didn't say anything. The burden of proof is upon you Daniel, not a well refuted old philosopher. Get your act together, and post your ideas/logic here, or just give up. I suggest the later. You will still have your ideas and beliefs, but you didn't convince us of anything.

By Nerd of Redhead, OM (not verified) on 18 Mar 2010 #permalink

[TA:3]This mover is itself either moved or not moved. If it is not, we have reached our conclusion—namely, that we must posit some unmoved mover. This we call God. If it is moved, it is moved by another mover. We must, consequently, either proceed to infinity, or we must arrive at some unmoved mover. Now, it is not possible to proceed to infinity. Hence, we must posit some prime unmoved mover.

But it is simply not true that we must posit one unmoved mover. There is nothing here that eliminates the possibility of infinite or at least more than one unmoved mover; more than one unmoved mover still fills the condition of not having the "causation chains" proceed infinitely through space-time. In other words: Monotheism FAIL.

By aratina cage (not verified) on 18 Mar 2010 #permalink

I don't have a lot of free time to spend here answering all of your questions. I'm online for maybe an hour a day and I have other sites I frequent during that time. I'll answer what I can, when I can.

But it is simply not true that we must posit one unmoved mover. There is nothing here that eliminates the possibility of infinite or at least more than one unmoved mover; more than one unmoved mover still fills the condition of not having the "causation chains" proceed infinitely through space-time. In other words: Monotheism FAIL.

You fail to grasp Aquinas' logic and then assert that it's untrue. He covers - at great length and from many angles - the proof that infinite prime movers is impossible. Read more, talk less.

By Daniel Smith (not verified) on 18 Mar 2010 #permalink

You fail to grasp Aquinas' logic and then assert that it's untrue.-Daniel Smith

I don't think so. I read chapter 13 of ContraGentiles which you cited, and I quoted section [3] wherein Aquinas fails to mention multiple unmoved movers. He doesn't even consider it.

Look past section [3] at 13[20]

It remains, therefore, that we must posit some first mover that is not moved by any exterior moving cause.

Again, he fails to consider multiple first movers. Or how about 13[33]

We must, therefore, posit that there exists a first efficient cause. This is God.

Once more, he fails to consider multiple first efficient causes. The whole argument falls apart there every time. This is a possibility that must be eliminated due to the impossibility of following back every single "causal chain" (that is, knowing all the causes of any event or "movement"). Aquinas does not address multiple prime movers at all in chapter 13.

Looking ahead, in 42[5] (about God, not necessarily movers), Aquinas offers up this:

Moreover, it is impossible that there be one continuous and regular motion from many movers. For, if they move together, none of them is a perfect mover, but all together rather take the place of one perfect mover. This is not befitting in the first mover, for the perfect is prior to the imperfect. If, however, they do not move together, each of them at times moves and at times does not. It follows from this that motion is neither continuous nor regular. For a motion that is continuous and one is from one mover. Furthermore, a mover that is not always moving is found to move irregularly, as is evident among lesser movers among whom a violent motion is stronger in the beginning and weaker at the end, whereas a natural motion proceeds conversely. But, as the philosophers have proved, the first motion is one and continuous. Therefore, its first mover must be one.

Does the movement have a spatial orientation? That kind of gets overlooked, doesn't it. But even so, this does not consider the possibility that unmoved movers only appear to have instantiated at different times to an observer when actually they did not and have always been, nor does he consider those unmoved movers that we will never know about (the chains that we never get to while searching for causes), nor does he consider the possibility that unmoved movers could appear to instantiate and evaporate randomly at any point in time at any place due to their effects on reality.

Then there is this from you:

He covers - at great length and from many angles - the proof that infinite prime movers is impossible.

Which is wrong. Read Chapter 43. Aquinas argues that God is infinite immediately after arguing that God is finite. And look back at chapter 13 where he says that God has to be an unmoved mover.

Read more, talk less.

Where do you get off? I so wish I were holding a starfart in my back pocket to use on you right now.

By aratina cage (not verified) on 18 Mar 2010 #permalink

Once more, he fails to consider multiple first efficient causes. The whole argument falls apart there every time. This is a possibility that must be eliminated due to the impossibility of following back every single "causal chain" (that is, knowing all the causes of any event or "movement"). Aquinas does not address multiple prime movers at all in chapter 13.

Aquinas' position, as I understand it, is that - even though we don't know all the participants in a branched causal chain (which does not mean we can't know them, only that we don't know them) - every branch in the chain of causality must necessarily itself end with a prime mover. IOW, it's the same prime mover for all chains and all branches.

Aquinas argues that God is infinite immediately after arguing that God is finite.

Aquinas never argues that God is finite. You have obviously misunderstood him.

By Daniel Smith (not verified) on 19 Mar 2010 #permalink

Once more, he fails to consider multiple first efficient causes. The whole argument falls apart there every time. This is a possibility that must be eliminated due to the impossibility of following back every single "causal chain" (that is, knowing all the causes of any event or "movement"). Aquinas does not address multiple prime movers at all in chapter 13.

Aquinas' position, as I understand it, is that - even though we don't know all the participants in a branched causal chain (which does not mean we can't know them, only that we don't know them) - every branch in the chain of causality must necessarily itself end with a prime mover. IOW, it's the same prime mover for all chains and all branches.

Aquinas argues that God is infinite immediately after arguing that God is finite.

Aquinas never argues that God is finite. You have obviously misunderstood him.

By Daniel Smith (not verified) on 19 Mar 2010 #permalink

IOW, it's the same prime mover for all chains and all branches. -Daniel Smith

Yes, but he doesn't actually know that. He made it up. It is just as much a possibility that different causal chains have different prime movers.

we don't know all the participants in a branched causal chain (which does not mean we can't know them, only that we don't know them)-Daniel Smith

How do you know we can know them all?

Aquinas never argues that God is finite. You have obviously misunderstood him.-Daniel Smith

Chapter 42, "That God is one", God is one prime mover (last sentence, 42[5]). The concept of "one" is finite.

By aratina cage (not verified) on 19 Mar 2010 #permalink

Chapter 42, "That God is one", God is one prime mover (last sentence, 42[5]). The concept of "one" is finite.

God is one infinite being (as opposed to two infinite beings, three infinite beings, four infinite beings,...)

This does not equate to God being finite. There are a finite number of Gods - specifically one - but that one God is an infinite being.

If you're going to refute Aquinas, you must refute his arguments as he meant them, not as you understand them.

It is just as much a possibility that different causal chains have different prime movers.

A prime mover is 100% actual, 0% potential. This means it is unchanging. If you are correct, there are a number of things that meet that definition. Can you name any?

By Daniel Smith (not verified) on 19 Mar 2010 #permalink

Daniel,
God is infinite? OK, is he aleph-zero or aleph-1?

By a_ray_in_dilbe… (not verified) on 19 Mar 2010 #permalink

A prime mover is 100% actual, 0% potential.

Then it is a physical object. Care to point out where we can find it, and verify it exists and does what you say? Then, and only then, might you have an argument. A philosophical concept is irrelevant if it doesn't match reality.

By Nerd of Redhead, OM (not verified) on 19 Mar 2010 #permalink

One problem with Aquinas's argument as quoted here (I haven't read the work in question) is that it is, logically as well as literally, pre-Einsteinian.

Aquinas had no concept of the limited speed of light and of the light cone. We cannot know everything about the universe we live in, because there are events outside our light cone. He assumes one mover, but that assumption is based on our ability to observe and trace all chains of causality.

The Big Bang and expanding universe also seem relevant here. If he's postulating his god as an actual entity within space-time, it's possible that this entity exists, started things going, but is no longer within our light cone. That is, we're not within us. So, there could be a prime mover that could no longer affect us, or know what we're doing.

Yes, believers often postulate/assume omnipotence and omniscience, but that's like postulating that the Flying Spaghetti Monster loves me because I have white hair. It's not evidence of anything. If someone is going to argue that god is outside the restraints of the physical universe, they can't also use what amounts to intuitive physics as evidence of that god's existence or properties.

Also, the Big Bang is a singularity in the information sense: we cannot know what happened "before" it. One mover, seventeen, none, aleph-null, we cannot ever know.

By v.rosenzweig (not verified) on 19 Mar 2010 #permalink

v.rosenzweig says: "Also, the Big Bang is a singularity in the information sense: we cannot know what happened "before" it. "

Particularly since time and space come into being with the Big Bang.

By a_ray_in_dilbe… (not verified) on 19 Mar 2010 #permalink

God is one infinite being (as opposed to two infinite beings, three infinite beings, four infinite beings,...)This does not equate to God being finite. There are a finite number of Gods - specifically one - but that one God is an infinite being. -Daniel Smith

So Aquinas does argue that God is finite—limited to one thing. Thank you for acknowledging that. And also thank you for acknowledging that more than one being of infinite size is possible. Then we can say that Chapter 42 considers that God is a countable entity and Chapter 43 considers that God has infinite size (spatial dimension, perhaps?), making God appear to encompass everything.

But how can God (a first mover) be everything if that includes things that are not God? How can a first mover be everything that it moves? And how can a first mover be incorporeal if it 1) interacts with corporeal beings and 2) is infinite in size and therefore incorporates corporeal beings into its being?

And when Aquinas writes, "Therefore, its first mover must be one." in 42[5], why does he not consider the problem of multiple first movers and thus multiple gods (or multiple infinite beings)?

A prime mover is 100% actual, 0% potential. This means it is unchanging. If you are correct, there are a number of things that meet that definition. Can you name any?-Daniel Smith

100% actual what? 0% potential what? I don't understand. Besides, it isn't up to me to prove that I am correct; I am saying that Thomas Aquinas did not address the possibility that there are multiple first movers (and neither did you), a problem that Aristotle apparently did address however briefly.

I'm really getting sick of this Aquinas nonsense. How about I propose a last stopper (or moved unmover) that stops the first mover before it moves anything thereby letting nature take its course. It only would take one moved unmover and that would pretty much kill the whole unmoved mover concept. What happens when the unmoved mover is deflected back and ends up moving itself?

By aratina cage (not verified) on 19 Mar 2010 #permalink

What happens when the unmoved mover is deflected back and ends up moving itself?

The answer, of course, is that you end up with time-paradox duplicates.

Unfortunately, they are always doomed.

By Celtic_Evolution (not verified) on 19 Mar 2010 #permalink

Me: A prime mover is 100% actual, 0% potential.

Nerd of Redhead: Then it is a physical object.

I guess you don't understand Aristotle's concept of Actuality and Potentiality. (See #342)

"100% actual, 0% potential" means that it is pure, unchanging existence.

By Daniel Smith (not verified) on 20 Mar 2010 #permalink

Daniel:

"100% actual, 0% potential" means that it is pure, unchanging existence.

Boy, your conceptual space is a mess.

Existence either is or is not actual, there are no gradations.

By John Morales (not verified) on 20 Mar 2010 #permalink

Aristotle thought women had fewer teeth than men. tell me, why should I take anything that idiot came up with seriously, when he couldn't even be bothered to check on such a basic fact? Aristotle and reality haven't precisely been well acquainted, you know. All his waffling about the nature of existence has no basis in reality. Why should I even consider it?

By Jadehawk, OM (not verified) on 20 Mar 2010 #permalink

v.rosenzweig: So, there could be a prime mover that could no longer affect us, or know what we're doing.

You are failing to grasp the distinction between chains of causality ordered per se and ordered per accidens. (See #351.)

By Daniel Smith (not verified) on 20 Mar 2010 #permalink

I'm just waiting for Daniel Smith here to start arguing against the Periodic Table based on the philosophy of the Four Elements! it would be just as rational and realistic as what he's doing now...

By Jadehawk, OM (not verified) on 20 Mar 2010 #permalink

I guess you don't understand Aristotle's concept of Actuality and Potentiality.

I'm a scientist. I don't understand bullshit concepts like philosophers pretend to do. Reality is my game. Philosophy without evidence is sophistry. And you are engaging in sophistry. Show the evidence.

"100% actual, 0% potential" means that it is pure, unchanging existence.

No, it means 100% bullshit argument, and nothing in reality to back it up. Welcome to science, which actually gets things done, and improves the knowledge of humanity.

By Nerd of Redhead, OM (not verified) on 20 Mar 2010 #permalink

Actually, it is known that there are many prime movers.

Aristotle would've been startled to see them, I suspect.

By John Morales (not verified) on 20 Mar 2010 #permalink

Aristotle and reality haven't precisely been well acquainted, you know.

Don't forget that he pretty much invented(with other greeks)naturalistic thought, came up with the first classification of animals in Historia/De Partibus Animalium, and pretty much founded biology and anatomy.
Of course he made errors, but his way of thinking in naturalistic terms and how he sought answers was radically new at the time.

The philosophical stuff he wrote is a different kettle of fish...

By Rorschach (not verified) on 20 Mar 2010 #permalink

just because he was marginally less detached from reality than other cultures of the time, doesn't mean he (and other greeks) was all that well-acquainted with it.

By Jadehawk, OM (not verified) on 20 Mar 2010 #permalink

Well, seeking, for the first time, natural instead of supernatural causes for observed phenomena was a rather big deal for mankind....:-)

By Rorschach (not verified) on 20 Mar 2010 #permalink

But how can God (a first mover) be everything if that includes things that are not God?

The problem is in your premise. Because God is "infinite" does not necessarily mean that he is "everything".

If, as you said, there could possibly be more than one infinite being (and infinite = everything) then these infinite beings would each be everything - making them indistinguishable from each other. So either there cannot be more than one infinite being OR "infinite" does not equal "everything".

So which is it?

By Daniel Smith (not verified) on 20 Mar 2010 #permalink

Owlmirror: I note that you've kind of abandoned the last two of the five ways -- perhaps because their logical fallacies are painfully and glaringly obvious even to you?

Not even close. I'd love to continue talking about them as well.

By Daniel Smith (not verified) on 20 Mar 2010 #permalink

Because God is "infinite" does not necessarily mean that he is "everything".

actually, yes it does, for any coherent definition of "infinite". because if god weren't everything, then he'd be finite: he'd "end" where other things begin.

By Jadehawk, OM (not verified) on 20 Mar 2010 #permalink

Because God is "infinite" does not necessarily mean that he is "everything".

Bullshit. Infinite implies everything. You have no rationality or logic, just presupposition. Which is why you're failing to convince us of anything.

Your deity doesn't exist, because there is no need for one, and science does quite well without one, and there is no evidence for one. Your batting average is abysmal.

By Nerd of Redhead, OM (not verified) on 20 Mar 2010 #permalink

Don't forget that he pretty much invented(with other greeks)naturalistic thought, came up with the first classification of animals in Historia/De Partibus Animalium, and pretty much founded biology and anatomy. Of course he made errors, but his way of thinking in naturalistic terms and how he sought answers was radically new at the time. The philosophical stuff he wrote is a different kettle of fish...

That's a switch... Aristotle being praised for his science - not his metaphysics!

By Daniel Smith (not verified) on 20 Mar 2010 #permalink

This is still going? While I admire your tenacity, there comes a point where blindly chasing rabbits down rabbit-holes becomes an exercise in futility. Just what are you trying to achieve here? You're obviously not convincing anyone that God exists or that Aquinas proves it, it's evidentially not happening. So what is it? Are you trying to demonstrate to yourself that your position is valid?

Even if Aquinas' arguments held true (they don't), then what would that demonstrate? Would it mean that there is a theistic entity operating within the bounds of this localised part of spacetime? Would it mean that there is an external intelligence unburdened by the normal restriction of having a brain? Would it mean that this abstract intelligence cared about the affairs of humanity? Would it mean that this abstract entity impregnated a young woman to give birth to himself? Would it mean that there's such thing as an afterlife? Heaven and Hell? Those arguments give you none of that.

If you can demonstrate the logical consistency or even necessity of unicorns, it doesn't mean Julius Caesar rode into battle on one. It doesn't even demonstrate that they exist in any meaningful way. And you still haven't demonstrated your initial assertion that without God there would be chaos - even after the principle of stability was explained to you.

Why are you here? What exactly are you trying to argue for? Are you scared that God doesn't exist?

actually, yes it does, for any coherent definition of "infinite". because if god weren't everything, then he'd be finite: he'd "end" where other things begin.

Bullshit. Infinite implies everything.

Then answer the question:

If there could possibly be more than one infinite being (and infinite = everything) then these infinite beings would each be everything - making them indistinguishable from each other. So either there cannot be more than one infinite being OR "infinite" does not equal "everything".

So which is it?

By Daniel Smith (not verified) on 20 Mar 2010 #permalink

Because God is "infinite" does not necessarily mean that he is "everything".

And what does God being "infinite" even mean?

By Feynmaniac (not verified) on 20 Mar 2010 #permalink

Why are you here?

Isn't that the great unanswered question? Actually I am just learning this stuff and wanted to "throw it to the wolves" and see if it gets ripped to shreds.

What exactly are you trying to argue for?

That should be pretty clear by now.

Are you scared that God doesn't exist?

Nope.

By Daniel Smith (not verified) on 20 Mar 2010 #permalink

I never suggested that there can be more than one infinite being. but then, I find the concept of an infinite being absurd to begin with for any other concept other than pantheism in which god is indeed everything, i.e. the whole material universe; at which point the god part can simply be excised as an exercise in parsimony, since the null hypothesis hasn't been disproven.

By Jadehawk, OM (not verified) on 20 Mar 2010 #permalink

Daniel,

That's a switch... Aristotle being praised for his science - not his metaphysics!

Nah, not a switch, it's quite consistent — though you might note it's his proto-science that receives acknowledgement.

If there could possibly be more than one infinite being ...

There can't be even one, as per #414 and #415.

By John Morales (not verified) on 20 Mar 2010 #permalink

So either there cannot be more than one infinite being OR "infinite" does not equal "everything".

It is not even clear that there can be one thing that is "everything". The idea certainly seems intuitive, but it leads to paradoxes (see Universal set).

By Feynmaniac (not verified) on 20 Mar 2010 #permalink

Can infinites even exist in anything more than the abstract mathematical sense?

Can infinites even exist in anything more than the abstract mathematical sense?

"God made the integers, all the rest is the work of man." - Leopold Kronecker

It's an interesting topic if whether infinities and the real numbers are nothing more than just a mathematical artifact. Both lead to counter-intuitive results.

Whatever the case may be, I'm not even sure if Daniel Smith's definition of infinite bears any resemblance to the mathematical sense because I have no idea of how he is using the word.

By Feynmaniac (not verified) on 20 Mar 2010 #permalink

Kel,

Can infinites even exist in anything more than the abstract mathematical sense?

That would depend on your definition of 'infinite' as an attribute; it can be used in different senses (e.g. something without bounds, something that encompasses the totality of a set, something which is absolute).

Physically speaking, the Universe is defined as everything that physically exists, so the only possible candidate for infinitude would be it.
No subset of it would qualify as infinite, and nothing else physically exists (both definitionally).

By John Morales (not verified) on 20 Mar 2010 #permalink

I don't think the/our universe is a great example for something infinite, what with multiverses seeming more and more likely to exist and all...

To me the term really is just a mathematical construct.Nice big gap to shove a god into, though.

By Rorschach (not verified) on 20 Mar 2010 #permalink

Aquinas' first three arguments can be summarized thusly:

  1. Contingent, non-self-existent, changing beings exist.
  2. Their existence implies the existence of other beings upon whom they depend for their existence and their change.
  3. The sum of contingent, non-self-existent, changing beings upon which any given being depends has a first member.
  4. This first member, a necessary, self-existent, unchanging being, must exist. Ex nihilo nihil fit (Nothing comes out of nothing).

Let's consider Ex nihilo nihil fit. It's an assumption. What happens if we consider it to be wrong:

  1. Contingent beings exist.
  2. Their existence implies the existence of other beings upon which they depend for their existence.
  3. Because no necessary being exists at least one otherwise contingent being does not depend for its existence on another being.
  4. Because this being is not necessary, its existence must be explained somehow.
  5. The only explanation is that it simply comes into existence.

Therefore, ex nihilo nihil fit is false.

Actually if we look at both sets of arguments we see a whole lot of question begging. An atheistic universe with a first member requires that at least this first member simply come into existence. However, to assume ex nihilo nihil fit guarantees the theistic conclusion in advance. By definition, the first member cannot be caused by another being. If it were, it would not be the first member of the chain. However, to assume that the first member must be caused (which is what ex nihilo nihil fit amounts to) is to assume that a necessary being must exist. This is the very issue in question.

By 'Tis Himself, OM (not verified) on 20 Mar 2010 #permalink

To me the term really is just a mathematical construct.

Somehow I feel this is like Zeno trying to prove that motion is impossible.

It is not even clear that there can be one thing that is "everything".

Does this mean Godel's incompleteness theorem disproves God? ;)

Daniel, forget your sophistry, what is the evidence. That is where your argument fails. Your argument and reality do not agree. Therefore, your argument, which is not based on reality, but rather sophistry and presupposition, fails. Show me the physical evidence for your imaginary deity. Then, and only then, will you have an argument based on reality, that might convince us.

By Nerd of Redhead, OM (not verified) on 20 Mar 2010 #permalink

I have seen folks claim that God is infinite, and other such tripe, before. They often argue for the existence of an god for various logical reasons, yet completely overlook the fact that a logical, infinite, and divinely compassionate god is very much NOT the snarly little JHW of the Bible.

The other odd argument believers advance about an infinite god is that the disproof of his existence would require a search of every little-bitty bit of the infinite universe, which would require that the searcher be infinite and therefor would be God. Or some such nonsense. Logic, at least the way I see it, says that one can disprove an infinite god by examining ANY part of the universe and finding no god in that part. My argument then is that if there is no God in the heart and mind of a Christian, there is not God anywhere.

So you folks that come here and make inane arguments for infinite gods, be aware that if your argument fails, your god was obviously not helping you, and it evaporates in a puff of logic.

Lesser versions of that argument apply to all religious folks, but are particularly sweet in the case of Christians. Their holy book say that if they drive someone away from God, they go to Hell. So hassling us is really risky.

Of course, Christians are playing for infinite stakes already, which just makes it more strange that most of them don't bother to put more effort into their religious research.

But then, if I had a mad god threatening me with eternal fire for the least screw-up, I'd probably be hiding under the bed with my fingers in my ears, going, "La-la-la-la" and praying that I had lucked into the right religion, denomination and flavour, just by being born in the right place.

By Menyambal (not verified) on 20 Mar 2010 #permalink

That is where your argument fails.

Actually, his argument fails within his own sophistry, the fact that he's making bare assertions about the nature of reality is the least of his troubles ;)

What is he going to do, bleed on me?

They often argue for the existence of an god for various logical reasons, yet completely overlook the fact that a logical, infinite, and divinely compassionate god is very much NOT the snarly little JHW of the Bible.

sssh, don't say it too loudly. Let them keep arguing that a prime mover means Jesus died on the cross for your sins - it's incredibly funny to see the triumph one feels from pulling out the cosmological argument while thinking it means that some Jew 2000 years ago performed some miracles and put them on the guest-list for the afterlife-party.

If, as you said, there could possibly be more than one infinite being (and infinite = everything) then these infinite beings would each be everything - making them indistinguishable from each other. So either there cannot be more than one infinite being OR "infinite" does not equal "everything".

Okay, so if the set that contains everything in the universe exists it is unique. Why does it deserve the title of "being" anymore than the set that contains of all my socks?

And, again, if you are not using infinite to mean "everything" then how are you using it? I've skimmed through the thread looking for a definition and couldn't find it. Can anybody help me?

By Feynmaniac (not verified) on 20 Mar 2010 #permalink

Can anybody help me?

Sorry, I think I missed that lecture at university where we were discussing the implications of infinity. Or I went to one of those Godless secular universities where my mathematics professor brainwashed me and my class by omitting particular information about the nature of infinity that proves that Jesus died on the cross for my sins.

I've said before that I've got no problem with people constructing abstract philosophical arguments to support the possible existence of a god; however, I have a huge problem with them taking the idea that a possible god may exist and conflating it with the idea that their specific god must exist - with nothing more than presupposition to link the two.

Unless you can find a way to explain it in such a way that the words 'Christian god' can't be crossed out and 'Zeus' written in its place then don't bother; you're wasting your time and ours.

By WowbaggerOM (not verified) on 20 Mar 2010 #permalink

Has it been said yet? Ex nihilo nihil fit is empirically wrong, as predicted by quantum physics.

So wrong, in fact, that if you put 2 metal plates into a vacuum, 100 nm from each other, nothing exerts an entire atmosphere of pressure to push them together!

"All philosophy written before the Industrial Revolution is best forgotten."
Opinion 12

By David Marjanović (not verified) on 20 Mar 2010 #permalink

I'd suggest you start here: [Contra Gentiles 13]

Why?

Aquinas pulls the same garbage logic trick of simply asserting that the unmoved mover is God:

In his Metaphysics [XII, 7], therefore, Aristotle goes on from the mover who is a part of the self-moved mover to seek another mover—God—who is absolutely separate. For, since everything moving itself is moved through appetite, the mover who is part of the self-moving being moves because of the appetite of some appetible object. This object is higher, in the order of motion, than the mover desiring it; for the one desiring is in a manner a moved mover, whereas an appetible object is an absolutely unmoved mover. There must, therefore, be an absolutely unmoved separate first mover. This is God.

This is the same nonsense he pulls in the Summa. Why am I supposed to be impressed when he waves his rhetorical hands and pulls his conclusion from nowhere and nothing using a logical fallacy?

=======

A prime mover is 100% actual, 0% potential.

You mean like everything else that is actually real?

This means it is unchanging.

Then the "prime mover" cannot be the God of the bible, or of any religion.

Thus you disprove Aquinas.

If you are correct, there are a number of things that meet that definition. Can you name any?

Why do they have to be named? Calling them names -- like "God", "Zeus", "Apollo", "Athena", "Ra", "Odin" -- suggests that they are people, when there's no reason to posit such a thing.

The point is that they are not logically impossible. Call them M′1-n, where n is simply unknown.

I just checked, and I see that one estimate for the number of protons in the universe is 4x1079. According to your argument -- that all matter needs this "prime mover" in order to do anything -- each of those protons could share a prime mover, or each could have its own. That doesn't even take into account the number of other fundamental particles, of course. But n is potentially enormous.

=======

Because God is "infinite" does not necessarily mean that he is "everything".

Not necessarily "everything", but the "ground" of everything; the prime mover of everything that moves, and of course, everything is continually in motion. That was your point, right?

=======

Not even close. I'd love to continue talking about them as well.

Well, Aquinas failed as miserably with those as with the first three, so there's not a whole lot of use in doing so.

=======

If there could possibly be more than one infinite being (and infinite = everything) then these infinite beings would each be everything - making them indistinguishable from each other.

Just because they are indistinguishable does not mean that they are not separate and individual.

All of the protons in your body -- and in the universe -- are indistinguishable from each other, but it does not follow that they are the same proton.

By Owlmirror (not verified) on 20 Mar 2010 #permalink

Daniel, forget your sophistry, what is the evidence. That is where your argument fails. Your argument and reality do not agree. Therefore, your argument, which is not based on reality, but rather sophistry and presupposition, fails.

The metaphysical arguments I've been making rest on simple premises based on observed phenomena which are largely not in dispute. For example, the premise that everything that exists is actual, that is - it is real and not imaginary. Liquid water is actually liquid water - it exists, it is not imagined.

Secondly there is the premise that everything that changes must have the potential to change and can only change in a way that it has the potential to do so. Liquid water has the potential to change to steam or ice, it cannot change into a rock or a car.

Thirdly, everything that changes, changes because something else that actually exists acts upon it. Liquid water changes to steam when exposed to the right amount of heat. It is something that is actually hot that makes the water actually hot and so it realizes its potential to become steam.

Are we together so far?

This is where it gets interesting.

Everything that is presently changing is simultaneously being changed by something else acting upon it. The water that is presently boiling and changing to steam is being heated by something that is simultaneously hot; that heat is simultaneously generated by an energy source - say an electric stove; that effect (the heat of the burner) is simultaneously caused by electricity flowing through a resistive load; that effect is simultaneously based on the properties of electrons in the molecular structure of the burner; which is simultaneously caused by the properties of the atoms themselves - and so on. The point being that everything in this simultaneous causal chain is dependent upon something else for its effect - if any one of these things is not there the chain falls apart.

Still with me?

This type of chain of causality - one that is simultaneous and dependent - cannot be infinite. If everything in the chain is dependent upon something else, the chain must end (actually begin) with something that is not dependent upon something else.

This brings us back to actuality and potentiality.

Everything in the chain that we've examined thus far has been a mixture of actuality and potentiality. The water has the potential to become hot; the electricity has the potential to flow; the molecular structure of the burner has the potential to be excited; etc. It is when all these things are exercising their potential (changing from potential to actual) at the same time, that the water realizes its potential to become steam (changes from potential steam to actual steam.) If the water does not have the potential to become steam, or if the burner did not have the potential to carry electrical current, or if the molecules did not have the ability to give up and take on electrons, the chain stops there and the water never changes.

So, every chain of causality of this type (that is - simultaneous causes) eventually (fairly quickly actually) arrives at the basic foundations of matter and energy in the universe. But these foundations are themselves a mixture of actuality and potentiality and hence only actualize their potential when something else acts upon them.

This thing that sustains these forces must itself be something that is actual and unchanging - something that does not have the potential to change - or else the whole thing falls apart - placing us right back at the start, again looking for something that is not dependent upon anything else. And, since matter, time, space, energy - all these things - have the potential to change, the thing that is purely actual must be immaterial and not itself subject to these things.

So, to answer your question, the evidence leads us to the conclusion that the universe is, at every moment, sustained by something immaterial and unchanging. These are, of course, two of the attributes we ascribe to God. Therefore it is entirely reasonable (as my initial post stated) to believe in God.

By Daniel Smith (not verified) on 21 Mar 2010 #permalink

Are we together so far?

No, I smell sophistry coming, not evidence. Try putting the evidence before the rationalization. It will make you look less foolish. You do that by citing the peer reviewed scientific literature as a basis for your inane point.

This is where it gets interesting.

No, this is where you prove yourself a sophist philosopher.

the evidence leads us to the conclusion that the universe is, at every moment, sustained by something immaterial and unchanging.

Nope, you fail again. Science, and the scientific evidence, has no need for imaginary deities, or sophist philosophers or their inane and wrong conclusions attempting to force an imaginary deity upon it. Get real. Failure again. Time to give up.

By Nerd of Redhead, OM (not verified) on 21 Mar 2010 #permalink

However, to assume that the first member must be caused (which is what ex nihilo nihil fit amounts to) is to assume that a necessary being must exist. This is the very issue in question.

No theological argument that I know of (of course there may be some) assumes that the first member "must be caused". All the arguments I've heard (and certainly Aquinas') arrive at the conclusion that the first member must be uncaused.

By Daniel Smith (not verified) on 21 Mar 2010 #permalink

No theological argument

All theology is sophistry. End of story.

By Nerd of Redhead, OM (not verified) on 21 Mar 2010 #permalink

Daniel Smith #440 wrote:

And, since matter, time, space, energy - all these things - have the potential to change, the thing that is purely actual must be immaterial and not itself subject to these things.

The "unchanging" thing you're talking about is simply the concept of "reality" itself -- or "existence" -- which does not change to non-real, or to non-exist. Everything else, even a hypothetical God, is a form (or possible form) of the underlying 'bedrock' of reality.

So, to answer your question, the evidence leads us to the conclusion that the universe is, at every moment, sustained by something immaterial and unchanging. These are, of course, two of the attributes we ascribe to God. Therefore it is entirely reasonable (as my initial post stated) to believe in God.

God is not unchanging: it's supposed to make choices, and act on those choices. Personhood is a collection of complicated attributes, and they simply won't all fit into the regression to Ultimate Simple you're going for.

The argument then fails even on its own terms, because it doesn't follow its chain of reasoning far enough.

Yeah, "the uncaused cause" is the "first cause" that they assume. As best as I can recall . . .

By Menyambal (not verified) on 21 Mar 2010 #permalink

Thirdly, everything that changes, changes because something else that actually exists acts upon it.

this is factually wrong. go read David's post and link at #438, since you evidently failed to do so the first time around. Also read up on virtual particles and radioactive decay.

can't be bothered to read the rest or your argument, since your premises are already wrong.

By Jadehawk, OM (not verified) on 21 Mar 2010 #permalink

Congratulations, Daniel; you've expressed an excellent argument - for deism. But you aren't a deist, are you?

By WowbaggerOM (not verified) on 21 Mar 2010 #permalink

Me: If there could possibly be more than one infinite being (and infinite = everything) then these infinite beings would each be everything - making them indistinguishable from each other.

Owlmirror: Just because they are indistinguishable does not mean that they are not separate and individual. All of the protons in your body -- and in the universe -- are indistinguishable from each other, but it does not follow that they are the same proton.

Apples and oranges. Individual protons are distinguishable by location. If we are talking about infinite individuals, and we are saying that infinite = everything, then each infinite individual is not like an individual proton but is instead "all the protons in the universe" - making them indistinguishable in every respect (and inseperable) from each other.

Try again.

Me: A prime mover is 100% actual, 0% potential.

Owlmirror: You mean like everything else that is actually real?

No - not at all. Everything else is a mixture of actuality and potentiality.

Try again.

Me: If you are correct, there are a number of things that meet that definition. Can you name any?

Owlmirror: Why do they have to be named? Calling them names -- like "God", "Zeus", "Apollo", "Athena", "Ra", "Odin" -- suggests that they are people, when there's no reason to posit such a thing.

This is a silly objection. Does calling a proton a "proton" suggest that a proton is a person? When I asked "Can you name any?", I was not implying that these "multiple prime movers" were people or gods. I was merely asking him to tell me what they are.

Me: Because God is "infinite" does not necessarily mean that he is "everything".

Owlmirror: Not necessarily "everything", but the "ground" of everything; the prime mover of everything that moves, and of course, everything is continually in motion. That was your point, right?

Your dispute on this point is not with me, it's with aratina cage, Nerd of Redhead, and Jadehawk. They are the ones who claimed that "infinite" necessarily means "everything".

By Daniel Smith (not verified) on 21 Mar 2010 #permalink

I don't think it's a good argument even for deism, since a deistic God that lacks any anthropomorphic qualities loses its god-hood. Take away intentional will, mind, values, awareness, goals, consciousness, and emotional content from "god," and, whatever you have left, it's not worthy of worship. That's not even a watered-down deism (unless someone is being poetic, of course.)

Nerd of Redhead: No, I smell sophistry coming, not evidence. Try putting the evidence before the rationalization. It will make you look less foolish. You do that by citing the peer reviewed scientific literature as a basis for your inane point.

Hiding behind "science" is cowardly. If you can actually refute my arguments - by showing the premises to be untrue, or by showing that the conclusions don't rationally follow - then do so.

Otherwise you're just avoiding the issue because (one can only presume) you have no coherent counter-arguments.

By Daniel Smith (not verified) on 21 Mar 2010 #permalink

I don't think it's a good argument even for deism, since a deistic God that lacks any anthropomorphic qualities loses its god-hood.

Which isn't a problem for us - but it's a heck of a problem for Daniel Smith because he's eventually going to make the leap that the possible god he's alluding to must exist and be his specific god; he's just hoping we won't notice.

By WowbaggerOM (not verified) on 21 Mar 2010 #permalink

Otherwise you're just avoiding the issue because (one can only presume) you have no coherent counter-arguments.

where there is no coherent, evidence-based argument, there cannot be a coherent counter-argument. there is no sensible argument to be had with empty sophistry.

but in any case, your premises have already been shown to be incorrect, so stop rambling.

By Jadehawk, OM (not verified) on 21 Mar 2010 #permalink

I don't think it's a good argument even for deism, since a deistic God that lacks any anthropomorphic qualities loses its god-hood. Take away intentional will, mind, values, awareness, goals, consciousness, and emotional content from "god," and, whatever you have left, it's not worthy of worship. That's not even a watered-down deism (unless someone is being poetic, of course.)

I am not saying that God lacks will, mind, awareness, goals, consciousness, etc.

I am, for the moment anyway, only dealing with the existence of God as proven by Aquinas' five ways. If you want to move beyond that to his divine attributes, then you must be ceding the first point. Is that the case?

By Daniel Smith (not verified) on 21 Mar 2010 #permalink

Wowbagger OM #451 wrote:

Which isn't a problem for us - but it's a heck of a problem for Daniel Smith because he's eventually going to make the leap that the possible god he's alluding to must exist and be his specific god; he's just hoping we won't notice.

No; my point was that his regression doesn't lead to deism either: it leads to a very basic concept of "reality" or "existence."

Even a deistic God, has to have attributes. Once it has attributes, then there's a hypothetical 'potential' for change.

Danile makes this claim: "This type of chain of causality - one that is simultaneous and dependent - cannot be infinite. If everything in the chain is dependent upon something else, the chain must end (actually begin) with something that is not dependent upon something else."

No.

Loops, Daniel. Loops. Everything in the "chain" is an interaction. Every arrow points both ways. Third Law, Daniel: to every action an equal and opposite reaction. This whole "chain" business is based on a completely wrong view of physics. It's as if you're relying on arguments formulated by people who did not have the faintest clue what they were talking about; who mistook their own mental habits for deep truths about reality.

Borromean rings: all three rings are bound together by all three rings. They don't have to be bound together by an unbound binder or a meta-ring.

By Stephen Wells (not verified) on 21 Mar 2010 #permalink

Daniel Smith #453 wrote:

I am not saying that God lacks will, mind, awareness, goals, consciousness, etc.
I am, for the moment anyway, only dealing with the existence of God as proven by Aquinas' five ways. If you want to move beyond that to his divine attributes, then you must be ceding the first point. Is that the case?

No. To state it again, your regression argument only gets to Reality or Existence. Since atheists have no problem with "reality is real" or "existence exists," then Reality/Existence is not God, and I'm not going to label it that way.

God requires attributes (and person-hood attributes at that!); this takes it out of the 'fundamental and unchanging' category you want to place it into.

Hiding behind "science" is cowardly.

Wrong sophistry breath. Hiding behind inane logic is cowardice. Show physical evidence for your arguments. Welcome to science, where inane arguments without evidence are considered false. If there is nothing wrong with your sophistry, an eternally burning bush should be in the offing for science to examine...

By Nerd of Redhead, OM (not verified) on 21 Mar 2010 #permalink

Sastra wrote:

No; my point was that his regression doesn't lead to deism either: it leads to a very basic concept of "reality" or "existence."

Whatever he's avoiding by purporting such a nebulous god-concept, he's going to be making a leap eventually. Even proponents of the weakest form of deism don't bother arguing with atheists; it's only theists wearing deist's clothing who do - and those clothes come off like velcro stripper-pants at the first opportunity.

By WowbaggerOM (not verified) on 21 Mar 2010 #permalink

Wowbagger OM #458 wrote:

Whatever he's avoiding by purporting such a nebulous god-concept, he's going to be making a leap eventually.

Ah, but you see, I used to nebulously embrace the nebulous-god concept. I don't care so much about the leaping, gussied-up versions, because that's not where the critical leap is, for the atheist. ;)

Daniel Smith #450

Hiding behind "science" is cowardly.

Hear that, guys. Daniel has determined that rationality and the real world are cowardly.

If you can actually refute my arguments - by showing the premises to be untrue, or by showing that the conclusions don't rationally follow - then do so.

In my post #429 I showed your "uncreated creator" depends on begging the question.

Otherwise you're just avoiding the issue because (one can only presume) you have no coherent counter-arguments.

Sorry, guy, but your arguments have been refuted. Now it's your turn to try to refute Nerd's demand for actual, physical evidence of The Big Guy In The Sky. You know, the one who worries about how often you masturbate.

By 'Tis Himself, OM (not verified) on 21 Mar 2010 #permalink

I am, for the moment anyway, only dealing with the existence of God as proven by Aquinas' five ways

this thing still going? You have not done anything of the sort. If you demonstrate that a unicorn has one horn and only one horn, it doesn't mean that unicorns actually exist or that you have proved them. It certainly doesn't mean that Julius Ceasar rode into battle on one.

Even if Aquinas' arguments were valid (they are evidentially not), you are not doing anything to show the existence of God. The best you could say is that the arguments show that the concept of God is not inconsistent, and even then you've got a vague abstract entity with no attributes or anything that we would recognise as being godly. To even call it God is to either diminish the word or (more likely) to play a game of wolf in sheeps clothing.

Whatever he's avoiding by purporting such a nebulous god-concept, he's going to be making a leap eventually.

It's the sad reality of these arguments. They are arguing for the logical necessity of God (funnily enough using inductive logic), then as soon as that's conceeded it turns into that God is an omnipotent omniscient entity which cares about the lives and affairs of humanity, is a source of morality, judges us and rewards / punishes us eternally based on those choices, and came down to earth in human form in order to give us a chance to vicariously atone for our transgressions.

But still, the real problem is that he thinks he's logically proven the existence of God and no matter how much he's had his arguments destroyed he's persisting with that line of inquiry.

Hmm, it looks like that shorter version of Daniel Smith's (eventual) position is something like this: 'stuff is + Jesus died for your sins; ergo, Christianity roolz!

By WowbaggerOM (not verified) on 21 Mar 2010 #permalink

The short, basic version of the argument is that Thought is not material, not dependent on anything, not inside of space or time, not subject to physical laws, not complicated, not in motion, and foundational to the existence and movement of everything else: therefore, God exists.

Aquinas' 5 Ways looks like a folk version of intuitive cosmology -- but I think it actually reduces down to a folk version of intuitive psychology.

Hiding behind "science" is cowardly.

No, pretending that an a posteriori axiom can be used to logically prove anything is wrong. You're making inductive arguments on matters that you can't prove inductively (see: problem of induction), if you don't use science then you're not ever going to be right.

Thirdly, everything that changes, changes because something else that actually exists acts upon it. Liquid water changes to steam when exposed to the right amount of heat. It is something that is actually hot that makes the water actually hot and so it realizes its potential to become steam.

I don't agree. As has been pointed out to you several time reality doesn't follow your medieval prejudices. Particles can come into existence without anything "acting" on them (see here). Aquinas was limited by the knowledge of the 13th century. You're not.

This type of chain of causality - one that is simultaneous and dependent - cannot be infinite. If everything in the chain is dependent upon something else, the chain must end (actually begin) with something that is not dependent upon something else.

Prove it. It certainly is counter-intuitive, but it doesn't make it wrong.

No theological argument that I know of (of course there may be some) assumes that the first member "must be caused". All the arguments I've heard (and certainly Aquinas') arrive at the conclusion that the first member must be uncaused.

Then why not assume the universe itself is "uncaused"? At least we know it exists.

By Feynmaniac (not verified) on 21 Mar 2010 #permalink

Feynmaniac #465 wrote:

Then why not assume the universe itself is "uncaused"? At least we know it exists.

There's a trick here: "universe" can mean the particular version of space/time which we find ourselves in, or it can mean something like cosmos (or Universe) -- everything. If you allow the Thomists to narrow the term down, they start to bring in the kalam argument, that the universe had a beginning, and we get bogged down in that whole mess.

The cosmos -- the whole shebang -- is uncaused. By definition. Even a hypothetical (and probably impossible) "nothingness" would have to be a form of the cosmos (reality).

Modern science seems to have undermined poor Aquinas and his 5 Intuitive Ways You Can Come Up With While Sitting in your Armchair. But I also think the argument fails even without physics, on its own terms. It tries to establish a God without any of the truly relevant godly attributes. But that only gets us to the concept of the cosmos.

I am, for the moment anyway, only dealing with the existence of God as proven by Aquinas' five ways. If you want to move beyond that to his divine attributes, then you must be ceding the first point. Is that the case?

No. But for the sake of argument let's assume you have proven that X exists and is unique (where X is first cause, or whatever). Show that X has the properties of intentionality, mind, etc. and that X is indeed the Christian God (or whatever version of God you believe in).

By Feynmaniac (not verified) on 21 Mar 2010 #permalink

Feynmaniac wrote:

No. But for the sake of argument let's assume you have proven that X exists and is unique (where X is first cause, or whatever). Show that X has the properties of intentionality, mind, etc. and that X is indeed the Christian God (or whatever version of God you believe in).

If Daniel is game we can expect the usual tapdance routine; I predict it will include sidesteps, shuck-and-jives and the frequent use of jazz hands.

By WowbaggerOM (not verified) on 21 Mar 2010 #permalink

Wowbagger OM #468 wrote:

If Daniel is game we can expect the usual tapdance routine; I predict it will include sidesteps, shuck-and-jives and the frequent use of jazz hands.

Sung to the tune of #463...

#440:

So, to answer your question, the evidence leads us to the conclusion that the universe is, at every moment, sustained by something immaterial and unchanging. These are, of course, two of the attributes we ascribe to God.

For pity's sake. That is as stupid as saying that apples have the attributes of being fruit, and invisible purple obsequious metallic square snozzberries have the attribute of being fruit, therefore, snozzberries exist because apples do.

Therefore it is entirely reasonable (as my initial post stated) to believe in God.

Reasoning from fallacies and using fallacies to reason is the opposite of reasonable.

=====

#448:

Apples and oranges.

No, that's your fallacy.

If we are talking about infinite individuals, and we are saying that infinite = everything, then each infinite individual is not like an individual proton but is instead "all the protons in the universe" - making them indistinguishable in every respect (and inseperable) from each other.

Who is talking about "individuals"? I'm talking about "prime movers", which you have not even established are "individuals".

And since prime movers are by your reasoning immaterial, there's no physical or logical impossibility that prevents a multiplicity of them being simultaneously everything, now is there?

This is a silly objection.

Your entire argument that a putative unique prime mover should be called "God" is much more silly.

Does calling a proton a "proton" suggest that a proton is a person? When I asked "Can you name any?", I was not implying that these "multiple prime movers" were people or gods. I was merely asking him to tell me what they are.

Why should they be anything besides prime movers? In fact, by your own arguments, they cannot be anything else besides prime movers. M′1, M′2, M′3, M′4, and M′5 are members of the set "all members of M′". But precisely because they are as fundamental as you insist, they cannot be anything else, any more than your putative unique prime mover can.

Your dispute on this point is not with me, it's with aratina cage, Nerd of Redhead, and Jadehawk. They are the ones who claimed that "infinite" necessarily means "everything".

Are you saying that my clarification is indeed correct by your own definitions?

=========

#450:

Hiding behind "science" is cowardly. If you can actually refute my arguments - by showing the premises to be untrue, or by showing that the conclusions don't rationally follow - then do so.

It's been done repeatedly. You simply refuse to acknowledge it -- which is unreasonable and cowardly.

=========

#453:

I am not saying that God lacks will, mind, awareness, goals, consciousness, etc.

Actually, you are. As soon as you commit to your unchanging prime mover as being "God", regardless of the simple fact that you have no reason to do so, you utterly destroy any possibility of "God" having will, mind, awareness, goals, consciousness, etc -- because all of those qualities involve change. They are utterly contradictory with being unchanging in their very essence.

Too bad for you.

I am, for the moment anyway, only dealing with the existence of God as proven by Aquinas' five ways.

Using fallacious logic, whose lack of reason which you simply refuse to acknowledge. This failure to acknowledge the fallacies is itself unreasonable, meaning that your belief in God is unreasonable, and your belief that it is reasonable to believe in God is even more unreasonable.

If you want to move beyond that to his divine attributes, then you must be ceding the first point.

How can it be "ceded" when it is entirely unreasonable in the first place?

By Owlmirror (not verified) on 21 Mar 2010 #permalink

It tries to establish a God without any of the truly relevant godly attributes. But that only gets us to the concept of the cosmos.

Yeah.Even granting his entire argument is correct (I don't think it is), as it currently stands all it shows is that X (where X is the first cause or whatever) exists and is unique. At best this is incomplete. God means something. At minimum the term implies intentionality and consciousness. It might also encompass intervention in human affairs, omnipotence, omniscience, omnibenevelont, etc.

He needs to either stop calling whatever he is allegedly proving exists 'God' (I suggest X) or finish the argument and show that X is indeed a being with whatever attributes he can show.

By Feynmaniac (not verified) on 21 Mar 2010 #permalink

If Daniel is game we can expect the usual tapdance routine;

While wearing clown shoes.

I predict it will include sidesteps, shuck-and-jives and the frequent use of jazz hands.

And a few pratfalls.

There may also be custard pie.

By Owlmirror (not verified) on 21 Mar 2010 #permalink

It tries to establish a God without any of the truly relevant godly attributes.

But by virtue of calling it God, you take along the cultural attributions of what we called God and thus conclude that from the notion of there being an uncaused cause means that Jesus died on the cross for your sins.

Completely unjustified reasoning of course, but what else can a theist do in the face of believing in such an absurdity? Otherwise you end up like Ken Ham, proudly defending the obviously irrational. The best solution is to find God in the nebulous and unknown, then you can proudly believe in the same obviously irrational entity but with the pretence of intellectual merit.

But that only gets us to the concept of the cosmos.

A new proof of God: Argument From Carl Sagan

  1. The laws of physics are God
  2. Therefore God exists

Owlmirror wrote:

There may also be custard pie.

Which is yet another supporting argument for what I started claiming ages ago when dealing with a similarly inane argument from the wretched fool facilis - that the only true god is Sideshow Bob.

By WowbaggerOM (not verified) on 21 Mar 2010 #permalink

A new proof of God: Argument From Carl Sagan Baruch Spinoza Marcus Aurelius Heraclitus (etc)

Fixed...

  1. The laws of physics are God
  2. Therefore God exists

("And", add the Christians, "created the world with a garden and a talking snake, and two trees, and the first people, and got mad when the people ate the fruit of one of the trees, and kicked them all out, and then later fucked a little girl and became his own son, and died on a cross and turned into a zombie came back to life for reals three days later. Yay!")

By Owlmirror (not verified) on 21 Mar 2010 #permalink

Daniel Smith @440, your first two premises are tautologies. However, you lost me with the third, namely that: "Thirdly, everything that changes, changes because something else that actually exists acts upon it."

There is every evidence that nuclear decays, atomic transistions and most other quantum events take place spontaneously--that is, uncaused by any outside event.

And, in your #445, yes, protons can be distinguished by statiotemporal coordinates--photons, pions, etc. not so much. So the question for you is whether your deity is a boson.

By a_ray_in_dilbe… (not verified) on 21 Mar 2010 #permalink

So the question for you is whether your deity is a boson.

Ironically, not only does God play dice but he obeys Bose-Einstein statistics.

By Feynmaniac (not verified) on 21 Mar 2010 #permalink

There is every evidence that nuclear decays, atomic transistions and most other quantum events take place spontaneously--that is, uncaused by any outside event.

Oh, it only looks uncaused. But since it changes, some fundamental unchanging aspect of the universe must be causing the change. That we call the prime mover. And if we're dishonest and/or given to using fallacious logic, that we also call God.

</metaphysical logic>

So the question for you is whether your deity is a boson.

Or even a Bozon.

<*Honk!*>

By Owlmirror (not verified) on 21 Mar 2010 #permalink

Fixed...

Probably should have put it like this:

  1. "The idea that God is an oversized white male with a flowing beard who sits in the sky and tallies the fall of every sparrow is ludicrous. But if by 'God' one means the set of physical laws that govern the universe, then clearly there is such a God."
  2. Therefore God exists

William Lane Craig makes a living out of dressing the same arguments up slightly differently and calling them novel...

Sigh. Only a few scraps of bone and gristle remain, and I'm not that desperate.

Ah well.

By John Morales (not verified) on 22 Mar 2010 #permalink

Or even a Bozon.

win

By Jadehawk, OM (not verified) on 22 Mar 2010 #permalink

As soon as you commit to your unchanging prime mover as being "God", regardless of the simple fact that you have no reason to do so, you utterly destroy any possibility of "God" having will, mind, awareness, goals, consciousness, etc -- because all of those qualities involve change. They are utterly contradictory with being unchanging in their very essence.

The problem I see with your argument is that it assumes that all consciousness is like human consciousness, that all will is like human will, that all intellect is like human intellect, that all reason is like human reason. There is no reason to assume that.

Beyond that, the very fact that human reason, will, intellect, and consciousness both exist and change, points to an uncaused cause of all reason, will, intellect, consciousness, and etc. that does not change.

One of Aquinas'/Aristotle's premises is that any agent of change must itself have the ability to cause said change. It cannot give what it does not have to give. A block of ice cannot cause a fire because it does not contain that potentiality. A burning ember can - it is itself on fire. It is said to contain that feature formally. A cigarette lighter can also cause a fire - though it is not itself on fire. It is said to contain that feature eminently. For their to be mind, will, intellect and all the higher forms of consciousness, their ultimate cause must have all these features either formally or eminently.

The further you take this, the more it points to an eternal being of infinite unchanging will, power, mind, life...

There is every evidence that nuclear decays, atomic transistions and most other quantum events take place spontaneously--that is, uncaused by any outside event.

So says the science of today. Even so, these things are caused by the ever changing properties or natures of the particles themselves. Change requires cause.

By Daniel Smith (not verified) on 22 Mar 2010 #permalink

the very fact that human reason, will, intellect, and consciousness both exist and change, points to an uncaused cause of all reason, will, intellect, consciousness, and etc. that does not change.

lolwut?

this makes no sense at all.

Even so, these things are caused by the ever changing properties or natures of the particles themselves.

wrong

By Jadehawk, OM (not verified) on 22 Mar 2010 #permalink

So says the science of today.

You say that like its a bad thing. While you're playing sophist, the science of today is able to predict quantum behaviour to an amazing accuracy. The current models work, and if you're going to talk in anything that is relevant you have to acknowledge that fact.

Change requires cause.

Bare assertion, you haven't explained why this is so.

Change requires cause.

I though several people JUST explained to you, giving examples, why this is not so.

I'm sure you're absolutely stuck in a catch 22 of teleology, but you can, in fact, get change without teleological cause.

whatever. Obvious to me you're a waste of time.

A block of ice cannot cause a fire because it does not contain that potentiality.

Not correct. While the probability is extremely small it is not zero.

By 'Tis Himself, OM (not verified) on 22 Mar 2010 #permalink

Even so, these things are caused by the ever changing properties or natures of the particles themselves.

yes, you don't get to redefine natural processes simply because they don't fit your personal notion of all things teleological.

Daniel Smith wrote:

Beyond that, the very fact that human reason, will, intellect, and consciousness both exist and change, points to an uncaused cause of all reason, will, intellect, consciousness, and etc. that does not change.

Emphasis mine. You keep making this assertion but have yet to support it with anything but wishful thinking. Why does the existence of changing human reason point to the existence of unchanging divine reason?

By WowbaggerOM (not verified) on 22 Mar 2010 #permalink

points to an uncaused cause of all reason, will, intellect, consciousness, and etc. that does not change.

Kinda reminds me of another presupposition fool (F. the fallacious fool). Tried to play gotcha with his definitions, but couldn't show any evidence either.

By Nerd of Redhead, OM (not verified) on 22 Mar 2010 #permalink

I though several people JUST explained to you, giving examples, why this is not so.

This is the kind of thing that puts people off philosophy. It's like someone insisting that Achilles cannot outrun the tortoise, it's pure sophistry with no relevance to reality. It doesn't matter how many times one observes the race, it doesn't matter how much the fundamentals of nature are explored, it doesn't matter about the mathematics showing otherwise - Zeno here is refusing to acknowledge the apparent results because it really does make sense that there's an infinity of distances in between two distances.

I wonder when Daniel Smith is going to question why the leading philosophers of the 21st century don't use Aquinas' arguments. Perhaps all those theist philosophers just don't know about the awesome proofs that Aquinas did...

A block of ice cannot cause a fire because it does not contain that potentiality.

last i checked, ice is made of hydrogen and oxygen, both of which are flammable, given the right conditions.

seriously, the "potentiality" shit about things only having potential to do certain things completely breaks down at the atomic level and below:everything is made of the same stuff, therefore stuff has the potential to be everything there is.

By Jadehawk, OM (not verified) on 22 Mar 2010 #permalink

Daniel Smith #482 wrote:

The problem I see with your argument is that it assumes that all consciousness is like human consciousness, that all will is like human will, that all intellect is like human intellect, that all reason is like human reason. There is no reason to assume that.

The problem I see with this argument is that if God's consciousness, will, intellect, and reason is not like anything we're familiar with, then why would we use those terms to designate something which is apparently nothing like our experience? You might as well say that God is pink, but not like the color pink; pink in an entirely new way.

You cannot appeal to our experience to try to make a case for something that is then going to avoid contradictions and inconsistencies by simultaneously being unlike anything in our experience.

One of Aquinas'/Aristotle's premises is that any agent of change must itself have the ability to cause said change. It cannot give what it does not have to give.

In other words, Like Comes Only From Like. Otherwise known as the Genetic Fallacy.

The amazing thing about science is that it has discovered that novelty builds from simpler components. Life does not come only from life: vitalism, which seemed so intuitively true, turned out to be factually false. Mind evolves from things that are not at all mind-like. There is no "essence of wet" that gets into water. Archaic talk of "potential" residing within things, is leading to a reification of abstractions.

For their to be mind, will, intellect and all the higher forms of consciousness, their ultimate cause must have all these features either formally or eminently.

No.

"Mental things, brains, minds, consciousnesses, things that are capable of comprehending anything -- these come late in evolution, they are a product of evolution. They don’t come at the beginning. So whatever lies behind the universe will not be an intellect. Intellects are things that come as the result of a long period of evolution." (Richard Dawkins)

Like does not have to come, from like. Science has passed this argument by.

Nerd wrote:

Kinda reminds me of another presupposition fool (F. the fallacious fool). Tried to play gotcha with his definitions, but couldn't show any evidence either.

Daniel isn't quite as stupid; Facilis made the fatal mistake of demanding we accept both his unsupported assertions: i.e. that a) his presup nonsense was valid and b) it only supported the Christian god and no other.

Since Daniel hasn't tried to bridge the vast gap between potential philosophical god and necessarily existing god he's only wrong on one level.

By WowbaggerOM (not verified) on 22 Mar 2010 #permalink

Beyond that, the very fact that human reason, will, intellect, and consciousness both exist and change, points to an uncaused cause of all reason, will, intellect, consciousness, and etc. that does not change.

No it doesn't, this is just anthropomorphising reality. Human reason, will, intellect and consciousness are evolved traits. The amount of order in our physical brains required for any of those processes has come through hundreds of millions of years of natural selection. And even then our will, reason, intellect and consciousness are not fixed - they are forever changing. Can one have an unchanging will? Can one have an unchanging intellect? Can one have an unchanging consciousness?

If you don't think that these cognitive faculties are physical in any form, why not put your money where your mouth is and get your brain removed? While scientists are looking into how the brain works, seeing how brain patterns relate to processing and conscious experience - it's jsut the science of today. Sophists can ignore all this and pretend that the mind is an abstract as opposed to an emergent property of information processing (which by definition cannot be unchanging).

So much order is needed for thought, meaning that any explanation of order involving order in turn needs an explanation, and so on ad infinitum. Science in its theories proposes how order emerges from chaos and how complexity emerges from simplicity, the theist on the other hand tries to explain order with an even greater order, thus begging the question and still somehow concluding Q.E.D.

Facilis was making a presuppositional argument (or trying to); Daniel is making an evidential one (we can reason from the evidence of nature, to God). It may seem similar because they both appeal to simplistic intuitions about mind. The Thomists at least engage in apologetics. Presupps just end up being an extended insult.

The problem I see with your argument is that it assumes that all consciousness is like human consciousness, that all will is like human will, that all intellect is like human intellect, that all reason is like human reason.

In other words, you're going to assert, with neither logic nor evidence that there is such a magically different consciousness, will, and intellect, just because you want to, right?

Special pleading is ALSO a logical fallacy.

And you are continuing to be ever more unreasonable.

There is no reason to assume that.

Actually, there is. As I pointed out, consciousness, will, and intellect demand change, and changelessness is a direct contradiction of how consciousness, will, and intellect work.

There is indeed reason to assume that there can be no such thing as numerical answer to dividing by zero.

There is indeed reason to assume that there can be no such thing as a square circle.

There is indeed reason to assume that there can be no such thing as something that is simultaneously invisibly colorless and pink.

There is indeed reason to assume that a complete contradiction cannot possibly exist.

Beyond that, the very fact that human reason, will, intellect, and consciousness both exist and change, points to an uncaused cause of all reason, will, intellect, consciousness, and etc. that does not change.

Sorry, no. You may get a provisional pass for the metaphysics of physics, but not for consciousness -- unless you can demonstrate that consciousness has no relationship whatsoever with the physical.

Of course, doing so would also demonstrate that consciousness does not actually exist, which destroys your argument again.

One of Aquinas'/Aristotle's premises is that any agent of change must itself have the ability to cause said change. It cannot give what it does not have to give. A block of ice cannot cause a fire because it does not contain that potentiality. A burning ember can - it is itself on fire. It is said to contain that feature formally. A cigarette lighter can also cause a fire - though it is not itself on fire. It is said to contain that feature eminently.

What a pathetically ignorant summary. Come on, this is back to real physics and chemistry again, which Aquinas and Aristotle did not know.

And it looks like you don't know either, and don't know that you don't know.

For pity's sake, look up things like kindling temperature and spontaneous combustion.

For their to be mind, will, intellect and all the higher forms of consciousness, their ultimate cause must have all these features either formally or eminently.

False analogy. Consciousness is not a fire. You're also contradicting your previous argument -- you wanted to claim that the ultimate cause of all motion does not have either the formal or eminent "feature" of motion.

The further you take this, the more it points to an eternal being of infinite unchanging will, power, mind, life...

The further you take this, the more it points to no being at all.

And the further you take this, the more ridiculously unreasonable you look.

By Owlmirror (not verified) on 22 Mar 2010 #permalink

Sastra wrote:

Facilis was making a presuppositional argument (or trying to); Daniel is making an evidential one (we can reason from the evidence of nature, to God).

True, but the underlying flaws in the logic are the same, since they both boil down to the same assumption, which is: something, therefore (necessarily) something else and that something else must be god.

Yes, there are differences. Facilis looks at a cat and says 'cats must come from somewhere - therefore god'; Daniel Smith looks at a cat and says 'there must be a perfect cat and that perfect cat is god'.

It's the 'musts' that bother me, rather than the steps involved.

By WowbaggerOM (not verified) on 22 Mar 2010 #permalink

Facilis was making a presuppositional argument (or trying to); Daniel is making an evidential one (we can reason from the evidence of nature, to God). It may seem similar because they both appeal to simplistic intuitions about mind. The Thomists at least engage in apologetics. Presupps just end up being an extended insult.

I agree with you about presupposition being an extended insult -- but I would argue that an apologetic is a defense of presupposition.

Daniel presupposes that God exists, and he presupposes that logical fallacies made to support this presupposition are not logical fallacies, even when they are pointed out as being logical fallacies.

He's not reasoning from the evidence; he's unreasoning from the evidence back to the presupposition that he starts from.

Speaking of extended insult, note that Daniel started off with an insult (@#222):

It's funny that you think he cannot have any reasonable support for the notion of God when - in reality - your position (that there is no God) is the unreasonable one.

By Owlmirror (not verified) on 22 Mar 2010 #permalink

(And that insult (@#222) looks very much like the sort of insult presuppositionalists use)

By Owlmirror (not verified) on 22 Mar 2010 #permalink